Está en la página 1de 114

Cálculo Diferencial e Integral Docente: Carsten Holtfort

Notas principales del curso

NOTAS PRINCIPALES DEL CURSO


CÁLCULO DIFERENCIAL
CON UNA INTRODUCCIÓN A LA INTEGRACIÓN
PARA EL CURSO CÁLCULO INTEGRAL

Carsten Holtfort

INSTITUTO DE FÍSICA DE LA UNIVERSIDAD DE GUANAJUATO (UG)


DIVISIÓN DE CIENCIAS E INGENIERÍAS, CAMPUS LEÓN, UG

Página 1 de 114
1. SUCESIONES ................................................................................. 3
1.1. Los conjuntos de los números....................................................................................... 3
1.2. Sucesiones..................................................................................................................... 3
1.2.1. Límites de sucesiones y reglas para sucesiones convergentes .................................. 4
1.2.2. Sucesiones crecientes y decrecientes ......................................................................... 9
1.2.3. Sucesiones acotadas................................................................................................. 10
1.3. Series infinitas ............................................................................................................ 13
1.3.1. Suma y criterios de convergencia ............................................................................ 13
1.3.2. Series infinitas de términos no negativos ................................................................ 18

2. FUNCIONES ................................................................................. 24
2.1. Definición e interpretación geométrica ...................................................................... 24
2.2. Operaciones algebraicas con funciones ...................................................................... 25
2.3. Límites y continuidad ................................................................................................. 26
2.3.1. Límite de una función al infinito ............................................................................. 26
2.3.2. Límite finito de una función .................................................................................... 27
2.3.3. Continuidad de una función..................................................................................... 33
2.4. Derivada y diferenciación ........................................................................................... 45
2.4.1. Definición y reglas................................................................................................... 45
2.4.2. Derivadas de las funciones trigonométricas ............................................................ 54
2.5. Valores máximos y mínimos de funciones ................................................................. 64
2.5.1. La diferenciación (derivación) implícita ................................................................. 66
2.6. Teorema de Rolle y teorema del valor medio ............................................................. 67
2.7. Derivadas de orden superior ....................................................................................... 72
2.8. Inversa de una función ................................................................................................ 74
2.9. Antiderivación ............................................................................................................ 78
2.10. Integral definida e integración .................................................................................... 82
2.11. Teoremas fundamentales del Cálculo ....................................................................... 105

3. FUNCIONES ESPECIALES ............................................................ 112


3.1. Definición de una función potencia con exponentes racionales ............................... 112

Página 2 de 114
1. SUCESIONES

1.1. LOS CONJUNTOS DE LOS NÚMEROS

Conjunto de los números naturales N  1 , 2 , 3 ,  , n ,  , N0   0,1, 2, 3,, n,, (números


positivos (con cero)).
Conjunto de los números enteros Z   ,  n ,  ,  3 ,  2 ,  1 , 0 , 1 , 2 , 3 ,  , n ,   (números negativos,
positivos con cero).
 p 
Conjunto de los numeros racionales Q   x  p , q  Z , q  0  (números negativos, positivos
 q 
y cero, las fracciones positivas y negativas; entonces números decimales finitos y números
decimales infinitos periódicos positivos y negativos).
Conjunto de los números reales R: Los números racionales Q con los números irracionales I.
Éstos son los números decimales infinitos no periódicos positivos y negativos.
(También se define R como el conjunto de los números racionales completados, eso significa que
R es un conjunto completo: Cada sucesión de Cauchy*1) tiene un límite en el conjunto de los
números reales, R  R , la linea arriba indica que el conjunto R contiene todos los límites de
sucesiones de Cauchy de números reales! El conjunto de los números racionales no es completo
con respecto a sus límites. R  Q  Q  I (Q unión I). R consiste de todas las clases de
equivalencia de sucesiones de Cauchy de los números racionales. Hay que definir y explicar todo
esto en un curso de Cálculo más avancado, pero queremos al menos mencionarlo en este
manuscrito.
*1) Una sucesión an se llama sucesión de Cauchy, si (y sólo si) vale: Para cualquier   0 existe
un n0  N tal que an  am   m, n  n0 . (Vale el teorema: Una sucesión es convergente si y
sólo si es una sucesión de Cauchy). (S. Barner-Flohr Bd.I, Seite 118-120! Für den Beweis der Konvergenz mit
Intervallschachtelung auf der dortigen Seite 119 (vgl. hier Beweis zu teorema 2.10 -> teorema 1.5 -> axioma 1.1,
Vollständigkeitsaxiom!) wird gerade die Vollständigkeit von R benötigt!)

1.2. SUCESIONES
Por ejemplo, los números 2, 4, 6, 8, 10 forman una sucesión. Esta sucesión se denomina finita,
porque tiene un último número. Si un conjunto de números que forman una sucesión no tiene
último número, se dice que la sucesión es infinita, por ejemplo 1/2, 1/3, 1/4, 1/5, ...
Definición 1.1: Función sucesión:
Una función sucesión a : N  R es una correspondencia del conjunto de los números naturales
N al conjunto de los números reales R.

A cada n N le corresponde un único valor an  R : an :n an .

Se escribe también  an nN .

Página 3 de 114
1 1 1 1 1
Ejemplos: a n  : a1  1, a 2  , a3  , a 4  , ... , a n  , ... .
n 2 3 4 n
n 1 2 3 4 n
an  : a1  , a 2  , a3  , a 4  , ... , a n  , ... .
2n  1 3 5 7 9 2n  1

1.2.1. LÍMITES DE SUCESIONES Y REGLAS PARA SUCESIONES CONVERGENTES

Definición 1.2: Límite de una sucesión


Una sucesión annN tiene el límite L si para cualquier   0 (   R  ) existe un número
N  0 ( N R ), tal que para todo n  N ( n N ) entonces an  L   .
Se escribe: lim an  L o a n  L . Es decir: annN es convergente.
n  n

Si no existe un número L R así, se dice que la sucesión es divergente.


lim an  L es equivalente a lim ( a n  L )  0 lo cual es equivalente a lim an  L  0 porque en la
n  n n

definición se puede escribir: an  L    (an  L)  0    a n  L  0   .


El límite de una sucesión siempre es único, ver Apéndice 2 en la página 22.
De otra forma:
lim an  L   c.q.  0  N tal que an  L   n  N  .
n 

 an  L , an  L   an  L   , an  L 
an  L    an  L     
 (an  L) , an  L   (an  L)   , an  L 
a    L , an  L 
an  L     n  , por eso se escribe L    an  L  
 an  L   , an  L 

1 1
Ejemplo 1: an  . Sospechamos que L=0, porque decrece más y más con n creciente.
n n
Demostración:
Tenemos que mostrar que es correcto lo siguiente:
1 1 1
lim  0   c.q.  0  N tal que  0    n  N  . n  N , por eso vale siempre n  0 .
n  n n n
1 1
Entonces    n  : N . ─
n 
1 1
Entonces para cualquier  existe un N   tal que para todo n  N  vale
 
1 1 1 1
   ( n  0) :     0   . Eso significa lim  0 . q.e.d.
n n n n  n

Página 4 de 114
n n 1
Ejemplo2: an  . L=? Truco: an   Porque 1 n
1 , 2 n
 2 y
2n  1 2n  1 2  1  
n
1 1 1
 0 vale an n
  . (La demostración se verá más adelante).
n n  
20 2

Definición 1.3: Suma, diferencia, producto y cociente de dos sucesiones:


Dadas las dos sucesiones an nN y bn nN (ver Def. 1.1)
(i) su suma, denotada por a+b, es la sucesión definida por
a  bn : an  bn (su diferencia: a  b n : an  bn ).
(ii) su producto, denotado por a·b, es la sucesión definida por
a  b n : an  bn .
(iii) su cociente, denotado por a/b, es la sucesión definida por
a / b n : an / bn ., bn  0 n  N

Teorema 1.1: Reglas para Límites de Sucesiones:

Sea c  R . Si lim an  A y lim bn  B , vale que:


n  n

1. liman  bn   lim an  lim bn  A  B


n  n  n 

2. limc  an   c  lim an  c  A
  
n  n 

3. lim an  bn   lim an  lim bn  A  B


n  n  n

 a  lim an A
4. lim  n   n    , si bn  0 n  N  B  0
n b
 n  nlim 
bn B

Demostración:
1) Las hipótesis de la regla 1 son:
 1c.q.  0  N  1 tal que an  A   1 n  N 1 y  2 c.q.  0  N  2 tal que bn  B   2 n  N 2 .
Tenemos que mostrar lo siguiente:
 c.q.  0  N tal que (an  bn )  ( A  B )   n  N .
Lo mostramos así: Dado un arbitrario: Para cualquier  1  0 (  2  0 ) existe un N 1 ( N 2 ),
 
entonces también para 1  y para  2  . Además vale:
2 2

Página 5 de 114
(an  bn )  ( A  B )  (an  A)  (bn  B )  an  A  bn  B   1   2   (i). an  A   1 vale
para todo n  N 1 , bn  B   2 vale para todo n  N 2 , entonces vale la relación (i) para todo
n  max(N 1 , N 2 ) : N .
Entonces  c.q.  0  N tal que (an  bn )  ( A  B )   n  N  , ésto demuestra la regla 1.

2) La hipótesis de la regla 2 es:



 1c.q.  0  N  1 tal que an  A   1 n  N 1 , entonces también para  1  .
c
c  an  c  A  c  ( an  A)  c  ( an  A)  c   1 n  N  1 .

Entonces, si ponemos   c  1   1  y además N : N 1 vale el siguiente:
c
 c.q.  0  N tal que c  an  c  A   n  N lo cual demuestra la regla 2.

3) Las hipótesis de la regla 3 son:


 1c.q.  0  N  1 tal que an  A   1 n  N 1 y  2 c.q.  0  N  2 tal que bn  B   2 n  N 2 .
Tenemos que mostrar lo siguiente:
 c.q.  0  N tal que ( an  bn )  ( A  B )   n  N .
(an  bn )  ( A  B )  an  bn  an  B  an  B  A  B  an  (bn  B )  B  (an  A) 
an bn  B  B an  A
Porque an converge a A, existe una cota superior K1 para toda an (ver definición 1.4 más adelante),
entonces an  K1 n  N . Además vale bn  B   2 n  N 2 por hipótesis, B es un número
finito y an  A   1 n  N 1 por hipótesis. Entonces
( an  bn )  ( A  B )  an bn  B  B an  A  K1   2  B   1 . Si ponemos
 : K1   2  B   1 , se puede elegir un 1 y un 2 apropiados, tal que vale   K1   2  B   1 para
cualquier , porque las hipótesis de la regla 3 valen para todo 1 y 2. Además podemos elegir un
N así: N : max(N 1 , N 2 ) . Entonces vale el siguiente:
 c.q.  0  N tal que ( an  bn )  ( A  B )   n  N , que demuestra la regla 3.

4)
Las hipótesis de la regla 4 son:
 1c.q.  0  N  1 tal que an  A   1 n  N 1 y  2 c.q.  0  N  2 tal que bn  B   2 n  N 2 ,
además vale: bn  0 n  N  B  0 .
Para demostrar ésto, nos ayuda otra regla (4A):
1 1
Si vale cn n
 C , entonces

n  bajo la suposición cn  0 n  N  C  0

cn C
 c   1c.q .  0  N  1 tal que cn  C   1 n  N 1 .
Demostración: Hipótesis: cn n


Página 6 de 114
1 1 C  cn C  cn 1
     cn  C . Porque cn converge a C por hipótesis, existe una
cn C cn  C cn  C cn  C
cota superior K1 para toda cn (ver definición 1.4 más adelante) , entonces cn  K1 n  N ,
tambien existe una cota inferior K2 para toda cn, entonces
1 1
cn  K 2 n  N  K :  n  N ( K 2  0 porque cn  0 n  N  cn  0 n  N . Eso
K 2 cn
1
significa que la sucesión tiene una cota superior K para todo n. Además C es un número
cn
finito.
1 1 K
Por eso existe una cota superior para    cn  C y cn  C   1 n  N 1 para
cn C C
1 1 K C
cualquier  1  0 . Por eso vale    1 n  N 1 y podemos poner  1    . La
cn C C K
C
hipótesis cn  C   1 n  N 1 vale para cualquier  1  0 , entonces también para  1   .
K
1 1
Entonces para cualquier >0 se tiene que    n  N 1 . Si definimos N : N 1 , ésto es
cn C
1 1
equivalente a n . q.e.d.

cn C
Con esta regla (4A) podemos utilizar la regla 3 para demostrar que vale la regla 4. q.e.d.

Arriba ya utilizamos la definición siguiente:


Definición 1.4: Sucesión acotada
El número KI es cota inferior de la sucesión an nN si K I  an n  N .
El número KS es cota superior de la sucesión an nN si an  K S n  N .
Una sucesión es acotada si y sólo si tiene una cota superior y una cota inferior.
Además tenemos que demostrar que vale el siguiente:
Teorema 1.2: Cada sucesión constante cn  k n  N , k  R converge a k.

Demostración: cn  k  0 n  N   c.q .  0  N  tal que cn  k   n  N  lim cn  k .


n 
q.e.d.

Ahora todo está demostrado para calcular el límite del ejemplo 2:

Página 7 de 114
n 1 1
an   . Porque 1 n 1 , 2 n
  2 y n
 0 (como demostrado en
2n  1 2  1    
n 
n
1 1
ejemplo 1) vale an n 

 con las reglas 4 y 1.
20 2

1 1 1 1
Ejemplo: an  0 n  N , bn 
n  N y cn  2 n  N . Ya que 0  2  n  N
n n n n
1 1
sospechamos que 2 n
 0 porque n

 0 .

n n

Teorema 1.3: Dadas las tres sucesiones an nN , bn nN y cn nN . Los dos sucesiones an nN
y bn nN convergen: A  R tal que lim an  A y B  R tal que lim bn  B . Además existe un
n  n 

índice N1 tal que an  cn  bnn  N1 . Sea A=B.


Entonces converge la sucesión cn nN , eso significa que C  R tal que lim cn  C y este
n 
límite es igual a A=B.
Demostración: Por hipótesis vale el siguiente:  1 N  1 tal que A   1  an  A   1 n  N  1 y
c .q .

2 c .q .
N  2 tal que A   2  bn  A   2 n  N  2 porque B=A.
Dado un  c.q.  0 podemos elegir 1   2   y definimos N gen : max(N 1 , N 2 , N1 ) . Entonces
vale el siguiente: A    an  cn  bn  A   n  N gen  A    cn  A   n  N gen
 cn  A   n  N gen .
Entonces dado un  c.q.  0 existe un N : N gen tal que  cn  A   n  N  . Eso significa que
cn nN tiene un límite y este límite es igual a A. q.e.d.

1 c .q.
Con base en el teorema 1.3. podemos demostrar que todas las sucesiones , k  N tienen el
nk
1 1 1
límite 0, porque vale 0  k
 n  N y n
 0 !

n n n
Ejemplo 1:
n 2  3n
an  2
. Simplificamos an dividiendo el numerador y el denominador por n2 :
n
2
n 2  3 n 2 1 3 1
an  n 2 n  n  1  3  1 . Porque 1 
 0 vemos con las reglas 1 y 2 del
n 2 1 n n n
n
teorema 1.2 que an n 1  3  0  1 .


Página 8 de 114
Ejemplo 2:
6n 3  n 2  n  1
an  . Otra vez simplificamos an dividiendo el numerador y el denominador por n3
3n 3  2n 2
1 1 1
6  2  3
: an  n n n . Utilizando las reglas 1, 2 y 4 del teorema 1.2 y el hecho de que lim 1  0
1 n  n
3 2
n
1 1 6
, lim 2  0 y lim 3  0 vemos que vale an n   2 .

n  n n  n 3

Con estos ejemplos vemos que el límite de una sucesión con polinomios en el numerador y
denominador converge a los factores de la mayor potencia de n en el numerador y en el
denominador. La mayor potencia de n en el numerador tiene que ser menor o igual a la mayor
potencia de n en el denominador para tener convergencia!

Ejemplo 3:
1 1 1
6  2 3
6n 3  n 2  n  1 n n n . El denominador converge a cero, por
an  . Simplificación: an 
 2n 2 1
2
n
6
eso este cociente tendría el límite “ ”, lo cual no está definido. Entonces esta sucesión es
0
divergente, ésto significa que no tiene límite.

1.2.2. SUCESIONES CRECIENTES Y DECRECIENTES

Definición 1.5: Sucesión creciente y decreciente


Una sucesión an nN es
(i) creciente si an  an 1 para toda n  N (estrictamente creciente si an  an 1 para toda n  N );
(ii) decreciente si an  an 1 para toda n  N . (estrictamente decreciente si an  an 1 para toda n  N
)
Una sucesión es monótona si es creciente o decreciente.
1 1 1
Ejemplo 1: an  es estrictamente decreciente: ( n  0) ,   n  1  n  1  0 , eso es
n n n 1
correcto. q.e.d.

n n 1 n 1
Ejemplo 2: an  es estrictamente creciente: an 1   . Demostremos que
2n  1 2(n  1)  1 2 n  3
n n 1
vale an  an 1 : Esto es cierto si y solo si  para todo n  N ,
2n  1 2n  3
 n  ( 2n  3)  ( n  1)  ( 2n  1)  2n 2  3n  2n 2  3n  1  0  1 , eso es correcto (ver el
ejemplo en capítulo 1.2). q.e.d.

Página 9 de 114
( 1) n 1 1 1 1 1
Ejemplo 3: an  : a1  1, a2   , a3  , a4   , a5  , . Esta sucesión no es
n 2 3 4 5
monótona, pero es convergente con el límite 0: Con el teorema 1.3 podemos demostrar que el
1 1
límite es cero: Elegimos las sucesiones de la hipótesis del teorema 1.3 asi: an   , bn  y
n n
n 1 n 1
(1) 1 (1) 1 1  1
cn  . Vale    n  N . Porque lim  0 y lim     0 entonces
n n n n n  n n
 n
 (1) 
n 1
lim   0 .
n
 n 
( 1) n 1
También podemos demostrar que converge an  a cero exactamente como lo demostramos
n
1
para la sucesión an   (ver capítulo 1.2.1, ejemplo 1).
n

1.2.3. SUCESIONES ACOTADAS


En el capítulo 1.2.1 ya vemos la definición 1.4 de una sucesión acotada.
Teorema 1.4. Cada sucesión convergente es acotada.
Demostración: Porque existe un límite L ( lim an  L ) vale:  c .q.  0  N  tal que
n

L    an  L   n  N  . Entonces para todo n  N existe una cota inferior L   y una cota


superior L   . La definición de una sucesión dice que para todo n existe un único valor an  R .
Por eso todos los valores an con n  N son números reales finitos y el contenido
a n n  N con n  N   es un contenido finito! Por eso existe una cota K tal que
an  K  n  N    K  an  K  n  N  . Entonces existe una cota inferior
K I : min( L   , K ) y una cota superior K S : max(L   , K ) para todo n  N . q.e.d.

Teorema 1.5. Una sucesión monótona converge si y sólo si es acotada.


Demostración: Tenemos que demostrar que valen los dos enunciados: 1) Cada sucesión monótona
convergente es acotada y 2) Cada sucesión monótona acotada es convergente.
El enunciado 1) queda demostramos con el teorema 1.4 porque una sucesión monótona es un caso
especial de una sucesión general y el teorema 1.4 vale por todas sucesiones! Entonces tenemos
que demostrar solamente el enunciado 2):
Hipótesis: La sucesión an nN es monótona y acotada. Tenemos que demostrar que esa sucesión
tiene un límite. Eso podemos demostrar con el axioma siguiente:
Axioma 1.1 de completez:
Todo subconjunto M no vacío del conjunto de los números reales R que tiene una cota inferior
tiene un ínfimo (el ínfimo de un conjunto M es la máxima cota inferior).

Página 10 de 114
También, todo subconjunto M no vacío del conjunto de los números reales R que tiene una cota
superior tiene un supremo (el supremo de un conjunto M es la mínima cota superior).
(Con esta axioma 1.1 se puede demostrar que los números reales son un conjunto completo R  R
(ver capítulo 1.1)!)
Aqui utilizamos el símbolo an  por el conjunto de todos los valores de la sucesión an.
Demostramos el enunciado 2) del teorema 1.5 por el caso en que la sucesión an sea creciente y
tiene una cota superior. Con esta suposición el conjunto an  (el conjunto de todos los valores de
la sucesión an) tiene un supremo S. El supremo S es la mínima cota superior, esto significa:
1.) S es cota superior: an  S n  N
2.) S es la mínima cota superior: Cada número K<S no es cota superior de an  . Esto es
equivalente a el enunciado siguiente:
(Definición: El conjunto A tiene todas las cotas superiores posibles del conjunto an  .)
Entonces: K c.q.  S  K  A  K c.q .  A  K  S .
Además necesitamos el teorema siguiente:
Teorema 1.6 Enunciado (i): S es el supremo del conjunto an   enunciado (ii): Para cualquier
  0 existe un aN tal que aN  S   .
Primero demostramos la dirección “enunciado (i)  enunciado (ii)” que es equivalente a
“non[enunciado (ii)]  non[enunciado (i)]”.
Si no vale el enunciado (ii) entonces no vale el enunciado (i). non[enunciado (ii)] significa: “Existe
al menos un  0  0 tal que “no existe un an con an  S   0 ” que es equivalente a “existe al
menos un  0  0 tal que “para todo an  an nN vale an  S   0 ”. Pero eso significa que S   0
es una cota superior del conjunto an  ! Entonces existe una cota K : ( S   0 )  A que es menor
que S ( K  S   0  S ) que es equivalente a la negación de la condición 2.) [ K c.q.  A  K  S ]
de la definición “S es un supremo de an  ”. Eso significa que no vale la condición 2.) de la
definición del supremo S de an  y por eso S no es el supremo del conjunto an  ! Pero eso es la
negación del enunciado (i). Asi demostramos el enunciado “Si no vale el enunciado (ii) entonces
S no es el supremo del conjunto an  ” o de otra forma: “non[enunciado (ii)]  non[enunciado
(i)]”. Entonces vale el enunciado “enunciado (i)  enunciado (ii)! q.e.d.
Segundo demostramos la otra dirección: “enunciado (ii)  enunciado (i)”. Eso también lo
demostramos vía negación “non[enunciado (i)]  non[enunciado (ii)]”: non[enunciado (i)] es
equivalente a “S no es el supremo del conjunto an  ”, eso significa que al menos una de las dos
condiciones 1.) o 2.) de la definición del supremo no vale. Por ejemplo no vale condición 2.), eso
significa non[ K c.q.  A  K  S ]  existe al menos un K  A con K  S , o en otras palabras
existe al menos un  0  0 con S   0  A . S   0  A  an  S   0 n  N . Eso es la negación
del enunciado (ii)! q.e.d.
De la misma manera se puede demostrar el caso en que la sucesión an sea decreciente y tiene una
cota inferior.
q.e.d. Entonces ya hemos demostrado el teorema 1.6.

Página 11 de 114
Con la ayuda del teorema 1.6. podemos demostrar que la sucesión creciente acotada tiene un
límite: Ya que la sucesión an es acotada, por hipótesis existe un supremo del conjunto an  . Eso
significa según el teorema 1.6. que para cualquier   0 existe un aN tal que aN  S   (i).
También vale aN  an n  N (ii) porque an es creciente por hipótesis. Además vale
an  S n  N (iii) porque S es una cota superior. Con (i), (ii) y (iii) obtenemos:
S    aN  an  S  S   n  N  S    an  S   n  N . Resumiendo se tiene: Para
cualquier   0 existe un indice N tal que S    an  S   n  N  an  S   n  N  ,
lo que significa que existe un límite L de la sucesión an (L=S). Entonces an es convergente y el
límite es igual al supremo de an  ! q.e.d.
Asi demostramos el teorema 1.5.!

De los teoremas 1.4. y 1.5. podemos concluir:


1a) an nN convergente  an nN acotada. (esto último es condición necesaria, no suficiente!)
1b) an nN no acotada  an nN divergente.
1a)  1b).
Si an nN es acotada, no necesariamente se tiene que an nN es convergente! Ejemplo:
n 1
an  (1) es acotada ( an  1 n  N ) pero no tiene límite! Pero si la sucesión además es
monótona entonces si an nN es acotada siempre se tiene la convergencia!
2a) an nN monótona: an nN convergente  an nN acotada (en este caso esto ultimo es una
condición necesaria y suficiente:)
2b) an nN monótona: an nN acotada  an nN convergente.
También es posible que una sucesión es acotada y convergente, pero no es monótona (ver ejemplo
( 1) n 1
3 arriba: an  ).
n
El enunciado 2) es equivalente a:
3a) an nN monótona: an nN no acotada  an nN divergente.
3b) an nN monótona: an nN no convergente  an nN no acotada.
3c) an nN no acotada  an nN divergente o no monótona.
3d) an nN divergente  an nN no acotada o no monótona.

Página 12 de 114
1.3. SERIES INFINITAS

1.3.1. SUMA Y CRITERIOS DE CONVERGENCIA

Definición 1.6: Serie infinita


Si an nN es una sucesión y sn : a1  a2  a3    an entonces sn nN es una sucesión de

sumas parciales denominada serie infinita y se denota por a
n 1
n  a1  a2  a3    an   .

Los números a1 , a2 , a3 ,, an , son los términos de la serie infinita.



1 1 1 1 1 1 1 1 1 1
Por ejemplo:  4n
1n 1
2
  
3 15 35
. s1 
3
, s2   ,
3 15
s3    ,
3 15 35
...

1 1 1 1
sn       2 ,... La sucesión sn nN de las sumas parciales la podemos escribir
3 15 35 4n  1
1 1 1 1  1 1 
así:       , porque la expresión entre paréntesis es
4 n  1 2n  1 2n  1 2  2n  1 2 n  1 
2

 1 1  2n  1 2n  1 2n  1  2 n  1 2
      .
 2 n  1 2n  1  ( 2n  1)  ( 2 n  1) ( 2 n  1)  ( 2n  1) ( 2n  1)  ( 2 n  1) (2 n  1)  ( 2 n  1)
1  1 1 1 1 1 1 1 1  1 1 
Entonces podemos escribir sn   1                   
2  3 2 3 5 2 5 7 2  2n  1 2 n  1 
1  1 1 1 1 1 1 1 1 1 
y obtenemos sn   1            
2  3 3 5 5 7 2(n  1)  1 2(n  1)  1 2n  1 2n  1 
1  1 1 1 1 1 1 1 1 1 
sn    1            , equivalente a
2  3 3 5 5 7 2n  3 2n  1 2 n  1 2n  1 
1  1 
sn    1  .
2  2n  1 

Definición 1.7: Suma de una serie infinita



Considere que a n denota una serie infinita dada para la cual sn nN es la sucesión de sumas
n 1

parciales. Si lim sn existe y es igual a S, entonces la serie es convergente y S es la suma de la


n 

serie. Si lim sn no existe, entonces la serie es divergente y la serie no tiene suma.


n 

Calculamos lim sn del ejemplo anterior:


n 

1
1  1  1 1 1 1 1 1 1 1 n  1  1 0  1
lim  1    nlim  lim     lim    lim
n  2
 2n  1    2 n   2 2n  1 2 2 n   2n  1 2 2 n   2 1 2 2 2
n
.

1 1
Según la definición 1.7. vale:  2  .
n 1 4 n  1 2

Página 13 de 114

Una serie infinita de la forma a r
n 1
n 1
 a  a  r  a  r 2  a  r 3   a  r n 1   se denomina

serie geométrica. Vale que a1 : a y an 1  an  r n  N !


La n-ésima suma parcial de la serie geométrica está dada por:
 
sn  a  a  r  a  r 2  a  r 3   a  r n 1  a  1  r  r 2  r 3    r n 1 . Demostramos que vale
rn 1
sn  a  , r 1:
r 1
rn 1
 
sn  a  1  r  r 2  r 3    r n 1  a 
r 1
 r  1

 
 a  1  r  r 2  r 3    r n 1  r  1  a  r n  1 
 a  r  r 2
  
 r 3  r 4   r n  1  r  r 2  r 3    r n 1  a  r n  1 . q.e.d.
Entonces podemos utilizar:
rn 1
a  1  r  r 2  r 3    r n 1   a  , r  1!
r 1

Además mostramos que se tiene que: lim r n  0 si r  1 . Primero se considera r=0. En este caso
n

tenemos una sucesión constante que converge a cero (ver teorema 1.2). Para r  1 y r  0 debe-
mos mostrar que vale lo siguiente: Para cualquier   0 existe un número N  R  tal que
r n  0   n  N . La última expresión es equivalente a r    ln r
n
   ln( ) (adelanto:
n

ln(x), el logaritmo natural, es una función estrictamente creciente, ln(x) está definido para todo
x  0 , aqui vale r  0 y 0  r  1 ),  n  ln  r   ln( ) . Como 0  r  1 vale ln  r   0 , entonces
ln( ) ln( )
la proposición anterior r n  0   n  N es equivalente a n  . Por tanto, si N :
ln  r  ln r 
, se cumple r n  0   n  N . N   0 para 0    1 . Si   1 vale r n  0   n  N porque
n
r n  0  r  1 y r  1 por hipótesis, entonces en este caso también existe un número N   0 tal
que r n  0   n  N , por ejemplo N  0.1 , porque r n  0   vale para todo n  N !
Entonces para cualquier   0 existe un número N  R  tal que r n  0   n  N
 lim r n  0 si r  1 . q.e.d.
n 

Ahora podemos calcular el límite de la n-ésima suma parcial de la serie geométrica con r  1 :

lim sn  lim a 
 rn 1
  a 
1

 lim r n  lim1  a  
1
 0  1 
a
. Entonces vale:
n  n 
 r 1  r  1 n  n  r 1 1 r

a

n 1
a  r n 1 
1 r
!

Página 14 de 114

Teorema 1.7: Si la serie infinita a
n 1
n es convergente, entonces lim an  0 .
n

Demostración: Sea sn nN la sucesión de sumas parciales de la serie dada y denote la suma por S.
De la definición 1.7. vale lim sn  S y lim sn 1  S . sn : a1  a2  a3    an 1  an , entonces
n n  
 s n 1

an  sn  sn 1 . Ya vemos que vale lim an  lim sn  sn 1   lim sn  lim sn 1  S  S  0 . q.e.d.


n  n  n  n 

El teorema 1.7. proporciona un criterio simple para la divergencia de una serie: Si lim an  0 ,
n

entonces a
n 1
n es divergente!

Por ejemplo encontramos que la serie geométrica es divergente por r  1 porque la sucesión de
 
las sumas parciales de la serie sn  a  1  r  r 2  r 3    r n 1  a  n (r  1) en este caso no es
cero (es divergente)! También para el caso r  1 este límite no es igual a cero.

Este criterio lim an  0 es una condición necesaria, no es una condición suficiente! Un ejemplo
n
 
1
de una serie infinita divergente con lim an  0 es la serie harmónica:
n

n 1
a n  
n 1 n
.

Página 15 de 114
Teorema 1.8.:
 
1) Sea c  R \ 0 . Si la serie  an es convergente y su suma es S, entonces la serie c  a n
n 1 n 1

también es convergente y su suma es c  S :


  

 an  S
n 1
  c  a n  c   an  c  S
n 1 n 1
  
2a) Si  an  S y
n 1
 bn  T , entonces
n 1
 a
n 1
n  bn  es una serie convergente y su suma es S  T :
    

a n S, b n T   an  bn    an   bn  S  T .
n 1 n 1 n 1 n 1 n 1
  
2b) Si a
n 1
n S y b
n 1
n  T , entonces  a
n 1
n  bn  es una serie convergente y su diferencia es

S T :
    

a n S, b n T   an  bn    an   bn  S  T .
n 1 n 1 n 1 n 1 n 1


Demostración: 1) sn : a1  a2  a3    an sea la n-ésima suma parcial de la serie an 1
n . La n-
 
ésima suma parcial de la serie  c  an es c  (a1  a2  a3    an )  c  sn . Porque
n 1
a
n 1
n  S , por

hipótesis vale lim sn  S . Por tanto, lim c  sn  c  lim sn  c  S (ver teorema 1.1, regla 2).
n n  n

2a) sn : a1  a2  a3    an sea la n-ésima suma parcial de la serie a
n 1
n ,

tn : b1  b2  b3    bn sea la n-ésima suma parcial de la serie b
n 1
n . La n-ésima suma parcial

de la serie  a
n 1
n  bn  es (a1  b1 )  (a2  b2 )  (a3  b3 )    (an  bn ) 

(a1  a2  a3    an )  (b1  b2  b3    bn )  sn  tn . Por hipótesis vale lim sn  S y lim tn  T


n n 

. Por tanto, lim ( sn  tn )  lim sn  lim tn  S  T (ver teorema 1.1, regla 1). De la misma manera
n  n  n 
podemos demostrar 2b). q.e.d.

Página 16 de 114
Teorema 1.9.:
 
1) Sea c  R \ 0 . Si la serie  an es divergente, entonces la serie c  a n también es
n 1 n 1
divergente.
  
2) Si la serie a
n 1
n es convergente y la serie b
n 1
n es divergente, entonces la serie  a
n 1
n  bn 

es divergente.
Demostración:

1) Si la serie a n 1
n es divergente, entonces lim sn no existe. Ahora suponemos que la serie
n 


sn
c  a
n 1
n es convergente. Entonces existe el límite lim c  sn . Pero sn  c 
n  c
; de modo que

1
lim sn  lim c  sn   1  nlim c  sn  . Asi podemos ver que existe el límite nlim sn porque existe el
n  n  c c  

límite lim c  sn , lo cual es una contradicción. En consecuencia, la serie
n 
c  a
n 1
n es divergente.

2) Suponemos que la serie  a
n 1
n  bn  es convergente y su suma es S. Sea T la suma de la serie

a
n 1
n (es convergente por hipótesis). Entonces, como
    

 b  (a
n n  bn )  an   an  bn    an  S  T se concluye del teorema 1.8., 2b) que b n
n 1 n 1 n 1 n 1 n 1

es convergente y su suma es S-T. Pero esto es una contradicción a la hipótesis de que b
n 1
n es

divergente. En consecuencia,  a
n 1
n  bn  es divergente.

Teorema 1.10.:
 
Si a
n 1
n y b
n 1
n son dos series infinitas, que difieren únicamente en sus primeros m términos (es

decir, ak  bk k  m (m  N , m arbitrario pero fijo) entonces las dos series son convergentes o
ambas son divergentes. (Sin demostración)

Página 17 de 114
1.3.2. SERIES INFINITAS DE TÉRMINOS NO NEGATIVOS

Teorema 1.11.:
Una serie infinita de términos positivos es convergente si y solo si su sucesión de sumas
parciales tiene una cota superior.
Demostración: La sucesión de sumas parciales sn nN de una serie infinita de términos positivos
es creciente y tiene una cota inferior 0. Si la sucesión de sumas parciales también tiene una cota
superior, entonces la sucesión sn nN es monótona y acotada. Como el acotamiento y la
convergencia de una sucesión monótona son propiedades equivalentes (ver teorema 1.5.),
entonces, la serie es convergente. sn nN es monótona y tiene una cota inferior porque todos los
terminos de la serie son positivos, el acotamiento de la sucesión, se dice la existencia de una cota
superior, es equivalente a la convergencia de la sucesión y tambien es equivalente a la
convergencia de la serie por la definición 1.7.

Teorema 1.12.: Criterio de comparación:



Sea la serie a
n 1
n una serie de términos positivos.

(i) Si b
n 1
n es una serie de términos positivos que es convergente, y an  bn n  N , entonces

a
n 1
n es convergente.

(ii) Si c
n 1
n es una serie de términos positivos que es divergente, y an  cn n  N , entonces

a
n 1
n es divergente.

Demostración:

(i) Sea sn nN la sucesión de sumas parciales de la serie a n y tn nN la sucesión de sumas
n 1
 
parciales de la serie  bn . Puesto que
n 1
b
n 1
n es una serie de términos positivos que es

convergente, del teorema 1.10. se infiere que la sucesión tn nN tiene una cota superior; sea B
esta cota. Entonces vale: an  bn n  N  sn  tn  B n  N . Por tanto B es una cota superior

de la sucesión sn nN , lo cual es la sucesión de sumas parciales de la serie a n de términos
n 1

positivos. Por eso la existencia de una cota superior de sn nN es equivalente a la convergencia

de la serie a
n 1
n (ver teorema 1.11.).

Página 18 de 114
  
(ii) Suponemos que  an es convergente. Entonces, como
n 1
 an y
n 1
c
n 1
n son series infinitas de

términos positivos, y an  cn n  N , se deduce del inciso (i) que c
n 1
n es convergente. Esto

contradice la hipótesis; de modo que la suposición es falsa. Por tanto a
n 1
n es divergente.

Teorema 1.13.: Criterio de la raiz



Sea a
n 1
n una serie infinita para la cual cada an es positivo:

(i) Si lim n an  L  1 , entonces la serie es convergente;


n 

(ii) si lim n an  L  1 o si lim n an   , la serie es divergente;


n  n 

(iii) si lim n an  1 , no se puede concluir nada acerca de la convergencia a partir de este criterio.
n

Demostración:
(i) lim n an  L  1  N  tal que n an  L   c.q. n  N  L    n an  L   n  N  .
n 

Porque L  1 se puede eligir   0 tal que vale: L    1 (propiedad de los números reales).

q : L   con n an  q  1 n  an  q n  1 . La serie q
n 1
n
es convergente (ver ariba: La serie


a
geométrica es convergente: a r
n 1
n 1

1 r
, aqui tenemos a=1 y q  r con 0  q  1 ) y segun el

teorema 1.12. (i) también a
n 1
n es convergente.

( (ii) y (iii) sin demostración.)

32 n  1

Ejemplo: Aplicación del criterio de la raíz para determinación si la serie 
n 1 n
2n
es convergente
1
 2 nn1   2  1n 
3 2 n 1
 n 3  3 
o divergente: lim an  lim 2 n
n   lim 2 n   lim 2   0  1 , entonces la serie dada es
n n
 n  n
 n n  n   n 
   
convergente.

Teorema 1.14.: Criterio de la razón



Sea a
n 1
n una serie infinita para la cual cada an es positivo:

Página 19 de 114
an 1
(i) Si lim  L  1 , entonces la serie es convergente;
n a
n

a a
(ii) si lim n 1  L  1 o si lim n 1   , la serie es divergente;
n a n a
n n

a
(iii) si lim n 1  1 , no se puede concluir nada acerca de la convergencia a partir de este criterio.
n a
n

Demostración:
an 1 an 1 a n 1
(i) lim  L 1  N  tal que  L   c.q. n  N L    L   , q : L   .
n a
n an an
Porque L  1 se puede eligir   0 tal que vale: L    1 (propiedad de los números reales).
a aN   2
Entonces vale n 1  q  1 n  N  : Por ejemplo:  q  1 n  N  . Además vale el
an aN  1
an 1 aN   2 aN   3 aN   4 a
siguiente:       n 1 . Este producto tiene n  ( N  1)  1  n  N
aN  1 aN  1 aN   2 aN   3 an
an 1
factores, cada factor es menor q. Entonces vale  q n  N   an 1  aN  1  q  N   q n . El factor
aN  1
a N  1  q  N  no depende de n, es un constante k : a N  1  q  N  . Entonces vale an 1  k  q n n  N
 
. La serie  qn es convergente (ver ariba) y segun el teorema 1.12. (i)
n 1
a
n 1
n 1 es convergente y


entonces segun el teorema 1.10. también an 1
n es convergente.

a n 1 a
(ii) lim  L  1 o lim n 1   , entonces en cualquier caso existe un número entero N 0  0
n a n a
n n

a a
tal que si n  N 0 entonces n 1  1 (porque n 1  L   , L  1 podemos elegir   0 asi que vale
an an
a n 1
 1 , propiedad de los números reales). Si n toma los valores N 0 , N 0  1, N 0  2, , y así
an
sucesivamente, se obtiene aN 0 1  aN 0 , aN 0  2  aN 0 1  aN 0 , aN 0  3  aN 0  2  aN 0 1  aN 0 ,  De
esta manera, si n  N 0 , entonces an  aN 0 . En consecuencia, lim an  0 ; por lo que la serie dada
n
es divergente.


1
(iii) Si se aplica el criterio de la razón a la serie p ( p
) tiene
n 1 n

1 p
a ( n  1) p  n 
lim n 1  lim  lim    1 . Puesto que la serie p diverge si p  1 y converge si
n a
n
n 1 p n n  1
 
n

Página 20 de 114
p  1 , se ha mostrado que es posible tener series tanto convergentes como divergentes para las
a
cuales lim n 1  1 . q.e.d.
n a
n

n 1 1
1

n a n  1 (n  1)  2 n
n 1 a n  1  1.
Ejemplo 1:  n ; n 1  2   . De modo que lim n 1  lim
n 1 2 an n n2 n 1
2n n   an n   2 2
n
2
Por tanto, por el criterio de la razón (teorema 1.14.), la serie dada es convergente.


n2 n
Ejemplo 2: Para cuales valores de x converge la serie infinita   x , x  0 ?
n 1 n!

n2 n 3 3 n2 n
Solución: 
n 1 n!
x  x  2 x 2

2
x   
n!
x   . Utilizamos el criterio de la razón:

(n  1) 2 n 1
x
an 1 (n  1)! (n  1) 2  n! x n 1 (n  1) 2 n 1 1 1 
    x  2  x    2   x n
 0 .

Este
an 2
n n (n  1)!n  x
2 n
(n  1)  n 2
n  n n 
x
n!
a
vale para todo x  R  . Entonces encontramos: lim n 1  0  1  la serie es convergente para
n a
n

todo x  R  (teorema 1.14.).

Definición 1.8: Convergencia absoluta


 
La serie infinita  an es absolutamente convergente si la serie
n 1
a
n 1
n es convergente.

 
Teorema 1.15.: Si la serie  an es convergente, entonces la serie
n 1
a
n 1
n es convergente.

Demostración: Vale  an  an  an (ver apéndice 1). A cada miembro de la desigualdad se le



suma an y se obtiene: 0  an  an  2 an . Como la serie a
n 1
n es convergente por hipótesis,

también la serie 2a
n 1
n (ver teorema 1.8., regla 1). Entonces con el criterio de comparación
 
(teorema 1.12. (i)) se concluye que la serie  a
n 1
n  an  es convergente. La serie a
n 1
n puede
    
escribirse como  an   an  an  an  y segun teorema 1.8. vale
n 1 n 1
 an   an  an    an
n 1 n 1 n 1

Página 21 de 114
 
. Ya demostramos que la serie  a
n 1
n  an  es convergente y la serie a
n 1
n es convergente por

hipótesis, entonces segun teorema 1.8. la serie a
n 1
n también es convergente! q.e.d.

Con el teorema 1.15. podemos modificar el criterio de la razón:


Teorema 1.14b.: Criterio de la razón

Sea a
n 1
n una serie infinita para la cual cada an es diferente de cero:

an 1
(i) Si lim  L  1 , entonces la serie es absolutamente convergente;
n  an
an 1 a
(ii) si lim  L  1 o si lim n 1   , la serie es divergente;
n  a n a
n n

an 1
(iii) si lim  1 , no se puede concluir nada acerca de la convergencia a partir de este criterio.
n  an
Podemos demostrar este teorema como demostramos el teorema 1.14., solamente escribimos an
en lugar de an .
También podemos modificar (y demostrar similarmente) el criterio de la raiz:
Teorema 1.13b.: Criterio de la raiz

Sea a
n 1
n una serie infinita para la cual cada an es diferente de cero:

(i) Si lim n an  L  1 , entonces la serie es absolutamente convergente;


n 

(ii) si lim n an  L  1 o si lim n an   , la serie es divergente;


n  n 

(iii) si lim n an  1 , no se puede concluir nada acerca de la convergencia a partir de este criterio.
n 

Página 22 de 114
Apéndice 1:
 a si a  0
a   a  a  a  a :  a  a  a .
  a si a  0
a, b  R , entonces a  b  a  b .

Demostración:  a a a y  b  b  b entonces   a  b   a  b  a  b (i).

Además vale x  a   a  x  a donde a  0 (ii). (Demostración de (ii): Porque x  x si x  0


entonces x  a  x  a si x  0 y x   x si x  0 entonces
x  a   x  a  x   a si x  0 ).

Con (i) y (ii) podemos decir a  b  a  b . q.e.d.

Además vale: a, b  R , entonces a  b  a  b y a  b  a  b

Demostración: a  b  a  ( b )  a  ( b )  a  b y a  a  b   b  a  b  b b
 a  b  a  b . q.e.d.

Apéndice 2:
Demostramos en general que el límite de una sucesión convergente es único:
Hipótesis: an  L1 y an  L2 con L1  L2 . Esto significa que para para cualquier  1  0 existe
n n 

un número N  1  0 , tal que para todo n  N 1 vale an  L1   1 y que para para cualquier  2  0
existe un número N 2  0 , tal que para todo n  N  2 vale an  L2   2 .
Entonces para cualesquier 1  0 y 2  0 tenemos la desigualdad
L1  L2  ( L1  an )  ( an  L2 )  L1  an  an  L2   1   2 n  max( N  1 , N  2 ) , y porque la
suma de dos números tan pequeños como sean puede ser tan pequeño como sea resulta que debe
valer L1  L2  0  L1  L2 . Entonces no puede ser que L1  L2 , siempre sigue que L1  L2 para
cualquier par de números L1 , L2 , por eso sigue que el límite de cualquier sucesión convergente es
único.

Página 23 de 114
2. FUNCIONES

2.1. DEFINICIÓN E INTERPRETACIÓN GEOMÉTRICA


Definición 2.1: Función:
Una función f : X  R  Y  R es una correspondencia del subconjunto X (dominio) de los
números reales R al subconjunto Y (contradominio) de los números reales R.
A cada x  X le corresponde un único valor f ( x )  Y : f : x  f ( x ) .
(comparar definición 1.1), (Leithold p. 2)
A cada x  X le corresponde solamente un único valor y  f ( x )  Y , pero es posible que a unos
valores diferentes x corresponde el mismo valor y.
El dominio X puede ser el conjunto de todos números reales R o solamente un subconjunto de los
números reales. Ejemplo: f ( x)  x  2 : X  x  R | x  2 , Y  y  R | y  0  R0 .
Otros ejemplos:
f ( x)  3x 2  5 x  2 , X  R
5x  2
f ( x)  , X  x  R | x  4
x4
f ( x )  4  x 2 , X  x  R | x  2:
Tenemos que considerar la condición: 4  x 2  0  4  x 2  2  x  2  x  2 .

 x, x0
f ( x )  x : f ( x)   , X  ( , ), Y  [0, )
 x , x  0
 1 , x  0

sgn( x)   0 , x  0 (función signo), X  (  ,  ), Y   1, 0 ,1
 1, x  0

 x  3 , x  5

f ( x)   25  x 2 ,  5  x  5
 5 x , x  5

Definición 2.2: Gráfica de una funcion:


Si f es una función, entonces la gráfica de f es el conjunto de todos los puntos (x,y) del plano R2
 
para los cuales (x,y) es un par ordenado de f: gráfica de f: (x, y)  R | x  X  y  f (x) .
2

Página 24 de 114
2.2. OPERACIONES ALGEBRAICAS CON FUNCIONES

Definición 2.3: Suma, diferencia, producto y cociente de dos funciones:


Dadas las dos funciones f : X f  Y f y g : X g  Yg (ver Def. 2.1, comparar Def. 1.3)
(ia) su suma, denotada por f+g, es la función definida por
 f  g ( x) : f ( x)  g ( x) (su diferencia:  f  g ( x) : f ( x)  g ( x) ). f  g : X f  X g  Y f  g .
(ib) producto con una constante c  R , denotado por c  f , es la función definida por
(c  f )( x ) : c  f ( x ) . c  f : X f  Yc  f .
(ii) su producto, denotado por f·g, es la función definida por
 f  g ( x) : f ( x)  g ( x) . f  g : X f  X g  Y f  g
(iii) su cociente, denotado por f/g, es la función definida por
 
 f / g ( x ) : f ( x ) / g ( x ) . f  g: x  X f  X g | g(x)  0 Yf / g
(Comparar definición 1.3)

Definición 2.4: Función compuesta:


Dadas las dos funciones f : X f Yf y g : X g Yg (ver Def. 2.1).
La función compuesta, denotada por f  g , está definida por
f  g ( x ) : f  g ( x )  . f  g :  x  X g | g(x)  X f   Yf  g .
Ejemplos:
 3
1) f (x)  x , g ( x )  2 x  3 . Xg  R, X f  R . X f  g  x  R | 2 x  3  0  x  R | x   .

 2
5 Xg  R, X f  x  R \ x
2) f ( x )  , g ( x)  2 x  1 .  2 .
x2
 1
X f  g  x  R | 2x  1  2  x  R | x  
 2

Página 25 de 114
2.3. LÍMITES Y CONTINUIDAD

2.3.1. LÍMITE DE UNA FUNCIÓN AL INFINITO

La definición del límite de f(x) al infinito es similar :


Definición 2.5a: Límite de f(x) cuando x crece sin límite:
Sea f una función f :(a, )  Yf . El límite de f(x) cuando x crece sin límite es L, lo que se
escribe como lim f ( x )  L ,
x

si para cualquier   R  existe un número x  R , x  0 tal que
si x x entonces f ( x)  L   .

(comparar definición 1.2)


2x2 2 x2 2
Ejemplo: f (x)  2 , lim f ( x)  lim 2  lim 2
x 1 x  x  x 1 x  1
1 2
x
(comparar teorema 1.1, ejemplo 2)

Definición 2.5b: Límite de f(x) cuando x decrece sin límite:


Sea f una función f :(, a)  Yf . El límite de f(x) cuando x decrece sin límite es L, lo que se
escribe como lim f ( x)  L ,
x  

si para cualquier   R  existe un número x  R , x  0 tal que
si x x entonces f ( x)  L   .

2 1 5
2x2  x  5  
f ( x)  2x2  x  5 x x 2 x3  0 .
Ejemplo: , lim f ( x )  lim  lim
4x  1
3
x   x   4x3  1 x  
4 3
1
x

Definición 2.6a: Límite positivo infinito de f(x) cuando x crece sin límite:
Sea f una función f :(a, )  Yf . El límite de f(x) cuando x crece sin límite es positivo
infinito, lo que se escribe como lim f ( x)   ,
x 
 
si para cualquier número y0  R , y0  0 existe un número x0  R , x0  0 tal que
si x x0 , entonces f (x)  y0 .

Página 26 de 114
Definición 2.6b: Límite negativo infinito de f(x) cuando x crece sin límite:
Sea f una función f :(a, )  Yf . El límite de f(x) cuando x crece sin límite es negativo
infinito, lo que se escribe como lim f ( x )   ,
x 
 
si para cualquier número y0  R , y0  0 existe un número x0  R , x0  0 tal que
si x x0 , entonces f (x)  y0 .

Definición 2.6c: Límite positivo infinito de f(x) cuando x decrece sin límite:
Sea f una función f :(, a)  Yf . El límite de f(x) cuando x decrece sin límite es positivo
infinito, lo que se escribe como lim f ( x )   ,
x  
 
si para cualquier número y0  R , y0  0 existe un número x0  R , x0  0 tal que
si x x0 , entonces f (x)  y0 .

Definición 2.6d: Límite negativo infinito de f(x) cuando x decrece sin límite:
Sea f una función f :(, a)  Yf . El límite de f(x) cuando x decrece sin límite es negativo
infinito, lo que se escribe como lim f ( x )   ,
x  
 
si para cualquier número y0  R , y0  0 existe un número x0  R , x0  0 tal que
si x x0 , entonces f (x)  y0 .

2
1
2 x  x3 x 2
Ejemplo 1: lim  lim ( x  0 ) . El numerador tiene límite -1, el denominador
x 3x  5 x 3 5

x2 x3
tiene límite 0. El límite del cociente es   .

2.3.2. LÍMITE FINITO DE UNA FUNCIÓN

2 x2  x  3
Ejemplo ilustrativo: f ( x) 
x 1

x f(x) x f(x)
0 3 1.00001 5.00002
0.25 3.5 1.0001 5.0002
0.5 4 1.001 5.002
0.75 4.5 1.01 5.02
0.8 4.6 1.1 5.2

Página 27 de 114
0.9 4.8 1.25 5.5
0.95 4.9 1.5 6.0
0.99 4.98 1.75 6.5
0.99999 4.99998 2 7.0
Proporcionado cualquier número positivo   R  puede lograrse que f (x)  5   tomando
x  1 lo suficientemente pequeño, es decir existe un número positivo   R  lo suficientemente
pequeño tal que si x 1   entonces f (x)  5  
o
para cualquier   0 existe un número positivo   R  , seleccionado adecuadamente, tal que si
x 1   y x 1  0 entonces f (x)  5   .  depende de , por eso escribimos .

Definición 2.7a: Límite de una función:


Sea f una función definida en cada número de algún intervalo abierto que contiene a x0, excepto
posiblemente en el número x0 mismo: f : X f Yf , X f  (c, d ) \  x0  con x0 (c, d) . f puede
ser definida, pero no debe ser definida en x0.
El límite de f(x) conforme x se aproxima a x0 es L, lo que se escribe como
lim f ( x )  L
x  x0

si la siguiente proposición es verdadera:


Dada cualquier   0 ( R ) , no importa que pequeño sea, existe una  , x  0 ( , x  R )
0 0
tal
que f ( x)  L    x con x  x0   ( x  x0 ) (  ,x0 depende de  y puede depender de x0 ).
Este definición es equivalente a la siguiente:
Definición 2.7b: Límite de una función:
Sea f una función definida en cada número de algún intervalo abierto que contiene a x0, excepto
posiblemente en el número x0 mismo: f : X f Yf , X f  (c, d ) \  x0  con x0 (c, d) . f puede
ser definida, pero no debe ser definida en x0.
El límite de f(x) conforme x se aproxima a x0 es L, lo que se escribe como
lim f ( x )  L
x  x0

si la siguiente proposición es verdadera:


Para cualquier sucesión xn (c, d) con lim xn  x0 resulta lim f ( xn )  L .
n  n

lim xn  x0 es equivalente a lim ( xn  x0 )  0 y a lim x n  x0  0 , porque según la def. 1.2 se


n  n n 

puede escribir xn  x0    ( xn  x0 )  0    xn  x0  0   .

Demostración:(x)

Página 28 de 114
El límite de una función lim f ( x )  L es único, eso significa: Si lim f ( x)  L1 y lim f ( x)  L2 ,
xa xa xa

entonces L1  L 2 .
(Para la demostración ver Apéndice 2) (x)

2 x2  x  3
Entonces la función f ( x)  tiene el límite lim f ( x )  5 ! Pero la función f no está
x 1 x 1

definida en el número x  1 .

Teorema 2.1: Límite de una función lineal


Si m y b son dos constantes cualesquiera ( m , b  R ), entonces
lim(mx  b)  ma  b .
xa

Demostración: Se debe demostrar que para cualquier   0 existe una   0 , tal que si x  a  
entonces (mx  b)  (ma  b)   .
Caso 1: m  0 : b  b  0   vale x  R . Entonces en este caso vale que para cualquier   0
existe una   0 (puede ser cada número), tal que si x  a   (vale para todo x) entonces
(mx  b)  (ma  b)  0   .
Caso 2: m  0 : Como (mx  b)  (ma  b)  m  x  a , se desea encontrar una   0 para
cualquier   0 , tal que
si x  a   entonces m  x  a   o como m  0 ,

si x  a   entonces x  a  .
m
 
Esta proposición se cumplirá si   : Por lo que se puede concluir que si x  a   y  
m m

entonces (mx  b)  (ma  b)  m  x  a  m    . q.e.d.
m

Teorema 2.2: Límite de la suma y de la diferencia de dos funciones


Si lim f ( x )  L y lim g ( x)  M entonces lim f ( x)  g ( x)  L  M .
xa xa xa

Además vale limc  f ( x )  c  L  c  R .


xa

Por la demostración comparar la demostración del teorema 1.1!


Demostración: Se debe demostrar que para cualquier   0 existe un   0 , tal que
 f ( x)  g( x)  L  M     x  a   . Por hiotesis vale: 1
c.q.
 0 1  0 , tal que
f ( x)  L  1  x  a  1 y  2  0  2  0 , tal que g( x)  M   2  x  a  2 . Para
c.q.

Página 29 de 114
 
culaquier   0 podemos elegir  1 : y  2 : , porque la hipótesis vale para cualquier  1  0
2 2
y 2  0 , entonces también para estos valores especiales. Entonces vale
 f ( x)  g( x)  L  M    f ( x)  L  g( x)  M   f ( x)  L  g ( x)  M (ver Apéndice 1).
Con los hipótesis ya existen 1  0 y  2  0 , tal que f ( x)  L  1  x  a  1 y
g( x)  M   2  x  a  2 ; entonces si definimos  : Min ( 1 ,  2 ) existe un   0 , tal que
vale el siguiente: f (x)  g(x)  L  M   1   2    x  a  Min(1,2 ) . Resumiendo:
Para culaquier   0 existe un   0 , tal que  f ( x)  g ( x)  L  M     x  a   que es
equivalente a lim f ( x)  g ( x)  L  M . El inciso con la constante c se demuestra de manera
xa

semejante. Q.e.d.

Esta regla se puede generalizar a causa de la ley asociativa de la adición de números reales que
vale también para la suma de funciones: Hipótesis: lim f1 ( x)  L1 , lim f 2 ( x )  L2 y lim f3 ( x)  L3
xa xa x a

. Se define g : f2  f3 . Según las hipótesis y según el teorema 2.2 (aplicado para un par de
funciones) vale lim g ( x )  lim  f 2 ( x)  f3 ( x)  L2  L3 : Lg . De nuevo resulta del teorema 2.2
xa xa

(para un par de funciones) lim  f1 ( x )  g ( x )   L1  Lg y esto es equivalente a


xa

lim  f1 ( x )   f 2 ( x )  f 3 ( x )    L1   L2  L3  . Por lo tanto resulta aplicando la ley asociativa de la


xa

adición: lim  f1 ( x )  f 2 ( x )  f 3 ( x )   L1  L2  L3 . De esta manera se puede seguir y demostrar que


xa

 n  n
vale lim  f i ( x)   Li para cualquier n N , n finito. Las hipótesis en este caso son:
x a
 i 1  i 1
lim f i ( x)  Li  i  1,2,, n . De una manera semejante se puede demostrar la generalización de esta
xa

regla para el signo “-“ y para casos mixtos de signos.


n  n
Junto con constantes ci  R , i  1,2,, n , resulta que lim  ci fi ( x)   ci Li .
xa
 i 1  i 1
Esto es un ejemplo para el hecho que muchas veces en las matemáticas se puede generalizar
teoremas o definiciones de un par de elementos para cualquier número finito de elementos.

Teorema 2.3: Límite del producto de dos funciones


Si lim f ( x )  L y lim g ( x)  M entonces lim f ( x )  g ( x )  L  M .
xa xa xa

Por la demostración comparar la demostración del teorema 1.1 y la demostración del teorema
2.2!
De una manera semejante se puede generalizar este enunciado como para el teorema 2.2 para un
 n  n
producto de n funciones: lim  f i ( x)   Li con las hipótesis lim fi ( x)  Li  i  1,2,, n ,
xa
 i1  i1 x a

Página 30 de 114
para cualquier n  N , n finito. Esto se puede demostrar con la ayuda de la ley asociativa de la
multiplicación.

Ejemplo: limx(2 x  1)  lim x  lim(2 x  1)  4  9  36


x 4 x 4 x4

Porque vale el ley asociativo vale el siguiente:


Teorema 2.4: Límite del producto de n funciones
Si lim f ( x)  L y n es cualquier número natural ( n  N ) entonces lim  f ( x)   Ln .
xa
n
xa
 

Ejemplo: lim (5 x  7) 4  lim (5 x  7)  (3) 4  81
x  2 x  2

4

Teorema 2.5: Límite del cociente de dos funciones


f ( x) L
Si lim f ( x)  L y lim g ( x)  M entonces lim  si M  0 y g ( x )  0  x  V (a ) para
xa xa x  a g ( x) M
algún   0 . La “delta-vecindad” del punto a está definida como: V ( a ) :  x  R x  a   .
Por la demostración comparar la demostración del teorema 1.1!
De una manera semejante se puede generalizar este enunciado como para el teorema 2.3 para un
 n  n
producto de n funciones dividido por un producto de m funciones: lim  f i ( x)    Li con las
x a
 i 1  i 1
 m  m
hipótesis lim fi ( x)  Li  i  1,2,, n , para cualquier n  N , n finito y lim  g j ( x)   K j
x a xa
 j 1  j 1

con las hipótesis lim g j ( x)  K j  j  1,2,, m , para cualquier m  N . Entonces si se define


x a
n n n m
f ( x ) :  f i ( x ) con lim f ( x)  lim  f i ( x )   Li y g ( x) :  g j ( x) con
xa xa
i 1 i 1 i 1 j 1
m m
lim g ( x)  lim  g j ( x)   K j (ver teorema 2.3) y se aplica el teorema 2.5 para f y g en esta
x a xa
j 1 j 1
n n

 f ( x)  L
i i
forma resulta la generalización del teorema 2.5: lim i 1
m
 i 1
m
.
x a
g
j 1
j ( x) K
j 1
j

x( x  1) lim x  lim( x  1) 45 5 5


Ejemplo: lim  x4 x4
   .
x4 (7 x  1) x 2
lim(7 x  1)  lim x 2
 27  16 27  4 108
x4 x4

Página 31 de 114
1
Demostramos teorema 2.5 adelantando con la continuidad de la función : Sea h la función
x
1 1
definida por h( x)  . La función compuesta h  g está definida por h  g ( x )   . h es
x g ( x)
continua en todo número x excepto x  0 . En consecuencia
lim
1
x  a g ( x) xa xa
 
 lim h g ( x)   h lim g ( x )  h ( M ) 
M
1
. Segun teorema 2.3 y segun esta ecuación vale

f ( x)  1  1 1 L
el siguiente: lim  lim f ( x)    lim f ( x)  lim  L  .
x  a g ( x) g ( x)  g ( x) M M
xa
 x  a x  a

En la demostración siguiente necesitamos el límite de una función compuesta:


¡Porque necesitamos la definición de función continua en la demostración del proximo teorema
por favor ver la definición 2.9 en el capítulo 2.3.3.!

Teorema adicional 2.6: Límite de una función compuesta:


Si lim g ( x)  b y si f es continua en b, entonces lim f  g ( x)   f (b) .
x a x a

Demostración: Puesto que f es continua en b: Para cada 1  0  1  0 , tal que


c .q .

  0   2  0 , tal que
c. q.
f ( y )  f (b )   1  y  b   1 (i). Como lim g ( x)  b 1
x a

g ( x )  b  1  x  a   2 (ii).
Si x  a   2 se sustituye y por g(x) en el enunciado (i) obteniéndose lo siguiente:
Para cada 1  0  1  0 , tal que f  g ( x )   f (b)   1  g ( x)  b  1 (iii)
c .q .

Para cualqier 1  0 existe un 1  0 , tal que vale (iii), y para cualquier 1  0 (entonces en
c .q .

particular para esto) existe un  2  0 , tal que g ( x )  b  1  x  a   2 ; entonces vale:


Para cualqier 1  0 existe un  2  0 , tal que f  g ( x )   f (b )   1  x  a   2 , y ésto es
c .q .

equivalente a lim f  g ( x)   f (b) ! Porque b  lim g ( x) vale además lim f  g ( x)   f lim g ( x) .


x a x a x a
 x a

Q.e.d.

Definición 2.8a: Límite por la derecha:


Sea f una función definida en cada número de algún intervalo abierto ( a, c ) .
Entonces el límite de f(x), conforme x tiende a a por la derecha, es L, lo que se denota por
lim f ( x)  L
xa

si para cualquier   0 (  R  ) , no importa que pequeño sea, existe una   0 (  R  ) tal que
f ( x )  L    0  x  a   ( depende de  y del punto a).

Observa que en la última línea de la definición, no se colocaron barras de valor absoluto alrededor
de x  a ya que se consideran únicamente valores de x para los cuales x  a .
Página 32 de 114
Ejemplo: lim x  4  0 .
x 4

Definición 2.8b: Límite por la izquierda:


Sea f una función definida en cada número de algún intervalo abierto (d , a ) .
Entonces el límite de f(x), conforme x tiende a a por la izquierda, es L, lo que se denota por
lim f ( x)  L
xa

si para cualquier   0 (  R  ) , no importa que pequeño sea, existe una   0 (  R  ) tal que
f ( x )  L    0  a  x   ( depende de  y del punto a).

Se referirá al lim f ( x)  L como el límite bilateral para distinguirlo de los límites laterales. Los
x a

teoremas 2.1 a 2.6 de límites siguen siendo válidos si “ x  a ” se sustituye por “ x  a  ” o “


x  a  ”.

Teorema 2.7: El lim f ( x)  L existe y es igual a L si y sólo si lim f ( x) y lim f ( x) existen y


x a xa xa

son iguales a L.
(Para la demostración ver la definición del límite de una función, definición 2.7)

2.3.3. CONTINUIDAD DE UNA FUNCIÓN

Definición 2.9: La continuidad de una función en un número:


Se dice que la función f es continua en el número a si y solo si se satisfacen las tres condiciones
siguientes:
(i) f(a) existe
(ii) lim f ( x) existe (ver definición 2.7)
x a

(iii) lim f ( x)  f (a)


x a
Si una o más de estas tres condiciones no se cumplen en a, entonces se dice que la función f es
discontinua en a.

2x2  x  3
Ejemplo: f ( x )  (ver principio del capítulo 2.3.2.): Observa que el numerador de la
x 1
2( x  3 2)  ( x  1) (2 x  3)  ( x  1)
fracción puede factorizarse de modo que f ( x)   , porque el
x 1 x 1
polinomio del númerador tiene las dos raíces reales diferentes x1   3 2 y x2  1 . Existe un factor
común del polinomio del númerador y del polinomio del denominador: ( x  1) . Si x  1 , entonces
el numerador y el denominador pueden dividirse entre x  1 para obtener f ( x )  2 x  3 , x  1 . Es
evidente que puede lograrse que f(x) esté tan cerca de 5 como se desee, tomando x suficientemente
cerca de 1, por lo que esta propiedad de la función f no depende de que f esté definida cuando x  1
. Ningún valor de x hace que f(x) tenga el valor 5.

Página 33 de 114
Por lo tanto esta funciona tiene el límite lim f ( x)  5 , pero la función f no está definida en el
x 1

número x  1 , se dice f(1) no existe. Por eso no se cumple la condición (i) de la definición 2.9,
~
entonces f es discontinua en x  1 . Pero podemos definir una nueva función f , la cual sí es
continua en x  1 :
 2x2  x  3
~  f ( x)  , x 1 ~ 2 x  3 , x  1 ~
f ( x)   x 1  f ( x)    f ( x)  2 x  3 , definido para todo
 lim f ( x) , x  1  5 , x 1
 x 1
~ ~
x. Para está función vale lim f ( x)  5 , el valor en 1 existe, f (1)  5 , y además este valor es igual
x 1
~ ~ ~
al límite: lim f ( x)  f (1) . Entonces está función f sí es continua en x  1 . Esta definición
x 1
practicamente corresponde a “pintar un pequeño punto (infinitesimal)” en el agujero de la gráfica
2x2  x  3
de f ( x )  en el punto (1,5), es decir “reparar” la discontinuidad en la gráfica y generar
x 1
una gráfica continua. La función f no diverge a   o   en x  1 porque existe el límite finito
lim f ( x)  5 y la gráfica de esta función es una recta ( 2 x  3 ) con un pequeño agujero en el punto
x 1
(1,5). En un plot con cualquier software este agujero no se puede resolver, entonces practicamente
no es visible, pero sí existe. Además vale para una integral sobre una función con una
discontinuidad de este tipo en un punto, que el valor de la función en un punto aislado no afecta el
valor de la integral (ver el ejemplo 3.) abajo del comentario 2.10.1): Se tiene por ejemplo
2 2
~
0
 f ( x) dx   f ( x) dx . Por esta razón, la discontinuidad se denomina discontinuidad removible (o
0
eliminable). En algunos libros, por ejemplo para la ingieniería (ver el curso “Sistemas Lineales”),
se argumenta que “una función racional no tiene polo si para este polo existe un factor común”.
Estrictamente esto no es correcto, porque el polo sigue existiendo, si hay un factor común o no.
Pero con respecto a las aplicaciones para esta función, como la gráfica o integrales sobre la función
(junto con otras funciones), este polo que corresponde a una discontinuidad removible no tiene
efecto, es practicamente despreciable. En este sentido se puede seguir la argumentación de estos
libros. Pero claro, estríctamente hay que definir una nueva función con el límite de la función
original en este punto, para trabajar “sin polo”.

Si una discontinuidad no es removible, entonces se llama discontinuidad esencial.


2x2  x  3
Por ejemplo: f ( x )  . La gráfica de esta función se rompe en el punto donde x  2 ,
x2
por lo que se investigarán en ese punto las condiciones de la definición 2.9: (i) f ( 2) no está
definida, por eso f es discontinua en 2. La discontinuidad es esencial porque lim f ( x) no existe
x 2

como un número finito. La función diverge a   y   en x  2 . No existe un factor común


( x  2) en la factorización del númerador y del denominador como en el ejemplo arriba! Por eso
esta discontinuidad también recibe el nombre discontinuidad infinita.

Otro ejemplo:

Página 34 de 114
 x  3 si x  3
f ( x)   . Se investigarán las tres condiciones de la definición 2.9 en el punto 3:
 2 si x  3
(i) f (3)  2
(ii) lim f ( x)  lim (3  x)  0 , lim f ( x)  lim ( x  3)  0 . Por tanto, lim f ( x)  0 (ver
x 3 x 3 x 3 x 3 x 3

teorema 2.7).
(iii) lim f ( x)  f (3) , f no es continua en 3. Esta discontinuidad es removible porque si se redefine
x 3
~
f(3) como 0, entonces la nueva función correspondiente f será continua en 3.

Otro ejemplo:
x 2
f ( x)  es discontinua en 4. Si la discontinuidad parece removible, especula sobre cuál
x4
sería el valor de f(4) de modo que la discontinuidad sea eliminada:

lim f ( x )  lim
x 2
 lim
 
x 2  x 2
 lim
 x4
 lim
1 1
 (la funcón
x4 x4 x  4 
x  4 x  4  x 2 
x  4 x  4    
x  2 x4 x  2 4
f ( x)  x es continua para todo x  0 , ver más adelante(x)). Por tanto, se redefine la función f
~
en 4 y se obtiene una nueva función f , lo cual es continua en 4, definida por:
 x 2  1
~  si x  4 ~  x  2 si x  4 ~ 1
f ( x)   x  4  f ( x)    f ( x)  , x 0.
 lim f ( x ) si x  4  1 x  2
 x4 si x  4
 4

Teorema 2.8: Si f y g son dos funciónes continuas en el número a, entonces


(i) f  g es continua en a
(ii) f  g es continua en a
(ib)) c f es continua en a  c  R , c constante.
(iii) f  g es continua en a
(iv) f / g es continua en a, considerando que g ( a )  0 y g ( x)  0  x  V (a) para algún   0 .
(Para la demostración ver las reglas para límites de funciones, teoremas 2.2, 2.3 y 2.5) y
adicionalmente la definición de la continuidad, def. 2.9, y la def. 2.3!
Por ejemplo: lim( f  g )( x )  lim( f ( x)  g ( x))  lim f ( x)  lim g ( x)  L  M  f ( a )  g (a )
x a x a xa xa

 ( f  g )( a ) . Asi resulta qu el producto f  g es continua en x  a .


Aplicando la ley asociativa de la adición de funciones se puede generalizar el resultado de la regla
(i) y (ib) (comparar la generalización correspondiente del teorema 2.2): Si fi , i  1,2,, n , son
n
funciones continuas en x  a y si ci  R , i  1,2,, n , son constantes, entonces resulta que c f
i 1
i i

es continua en x  a . La regla (ii) se puede generalizar con contantes ci  R , i  1,2,, n ,

Página 35 de 114
negativas. La continuidad de cualquier combinación lineal de funciones continuas resulta también
con la ayuda de la generalización del teorema 2.2: Con lim f i ( x)  Li  f i (a )  i  1,2,, n el
xa

n  n n
teorema 2.2 dice lim  ci fi ( x)   ci Li   ci fi (a) lo que es equivalente con
xa
 i 1  i 1 i 1

 n
  n
 n
lim   ci f i     ci f i  . Esto es equivalente a  ci f i es continua en x  a .
xa
 i1 ( x )  i1 ( a ) i 1

De manera semejante se puede generalizar la regla (iii): Con la ayuda de la ley asociativa de la
multiplicación de funciones o con la ayuda de la generalización del teorema 2.3 resulta que: Con
la hipotesis lim f i ( x)  f i (a)  i  1,2,, n , es decir cada función fi es continua en a, se sigue
x a

 n
 n n
lim  fi ( x)   fi (a) lo que es equivalente a f i es continua en x  a . Con constantes
xa
 i 1  i 1 i 1
n
resulta que c f
i 1
i i es continua en x  a .

De manera semejante se puede generalizar finalmente también la regla (iv): Con la ayuda de la ley
asociativa de la multiplicación y división de funciones o con la ayuda de la generalización del
teorema 2.5 resulta que: Con la hipotesis lim f i ( x)  f i (a)  i  1,2,, n , y lim g j ( x)  g j (a) ,
x a xa

g j (a)  0 y g j ( x)  0  x  V (a) para algún   0 ,  j  1,2,  , m , es decir cada función fi y gj


n n

 ci fi ( x)  c f (a )
i i
es continua en a, con constantes ci y dj se sigue lim i 1
m
 i 1
m
lo que es
x a
d g
j 1
j j ( x) d g
j 1
j j (a)
n

c f i i
equivalente a i 1
m
es continua en x  a .
d g
j 1
j j

Ver la definición 2.9. (iii): Segun los teoremas 2.1 – 2.5 o directamente con la ayuda del
teorema 2.8 para funciones continuas del tipo f ( x )  x (continuas según el teorema 2.1)
podemos concluir que las funciones polinomiales f ( x)  an x n  an 1 x n 1  an  2 x n  2   a1 x  a0
son continuas para todo xR y que funciones racionales
n 1 n2
a x  an 1 x  an  2 x  a1 x  a0
n
f ( x)  n m , n, m  N son continuas en todo x  R a
bm x  bm 1 x m 1  bm  2 x m  2  b1 x  b0
exepción de los raíces del polinomino del denominador:

Teorema 2.9a: Una función polinomíal es continua en todo número.

Teorema 2.9b: Una función racional es continua en todo número de su dominio.

Página 36 de 114
x3  1
Ejemplo: f ( x )  : El dominio de f es X f  R \ x1 , x2  , x1 , x2 son los dos raíces del
x2  9
denominador: x 2i  9  0  x1  3 , x2  3 . Por el teorema 2.9b f es continua en todos los
números del conjunto X f  R \  3,3 .
Teorema 2.9c: Una función compuesta de dos funciones continuas también es continua:
Si g es continua en x  a , es decir lim g ( x)  g (a) , (se define b : g (a ) ) y si f es continua en b,
x a

es decir lim f ( y )  f (b) , entonces lim( f  g )( x)  lim f g ( x)   f  g (a)   ( f  g )(a) , según la


y b x a x a

def. 2.9 resulta que f  g es continua en x  a .


Demostración: Puesto que g es continua en a: Según la def. 2.9 se tiene: lim g ( x)  g (a) . Se
x a

define b : g (a ) . Puesto que f es continua en b  g (a ) : Se tiene: lim f ( y )  f (b) para cualquier


y b

sucesión y  b . Entonces resulta: 1.) Existe ( f  g )(a)  f  g (a)  . 2.) También existe
lim( f  g )( x)  lim f  g ( x)  porque según la hipótesis vale lim f ( y )  f (b)  f  g (a)  para
x a x a y b

cualquier sucesión y  b , entonces también para la sucesión y : g ( x )  b  g ( a ) que converge


de esta manera para x  a , porque g es continua en a por hipótesis. Entonces resulta x  a
 y  g ( x )  b  g (a )  f ( y )  f  g ( x)   f (b)  f  g (a)  . Es decir, se tiene:
lim f g ( x)   f  g (a)  . 3.) Por eso el límite existe y es igual al valor de la función compuesta en
x a

a. Según la def. 2.9 resulta que f  g es continua en x  a . Q.e.d.

En ocasiones se necesita emplear una definición de continuidad en la que se utilice la notación


 ,  . Segun la definición 2.9 una función es continua en el punto a si vale lim f ( x)  f (a) si
x a

existe el límite y si existe f (a ) . Al aplicar la definición 2.7a del límite de una función, donde L
es igual a f(a), se cumple si para cualquier   0 existe una   0 tal que si x  a   ( x  a )
entonces f ( x)  f ( a )   . Si f es continua en a, debe existir f(a); por tanto, la condición de que
x  a no es necesaria en la proposición, debido a que cuando x  a , f ( x )  f ( a ) será 0 y así,
menor que . Entonces tenemos una definición alternativa y equivalente de la continuidad:

Definición 2.10a: La continuidad de una función en un número:


La función f es continua en el número a si f está definida en algún intervalo abierto que contenga
a a y si para cualquier   0 existe una   0 tal que f ( x)  f ( a )    x con x  a   (o más
corto:  x  V (a) .  depende de  y puede depender de a.
También se puede formular la definición de esta manera:
La función f es continua en el número a si f está definida en algún intervalo abierto que contenga
a a y si para cualquier   0 existe una   0 tal que f ( x)  V  f (a)   x  V (a) (o más
compacto: f (V (a))  V  f (a)  ).  depende de  y puede depender de a.

Página 37 de 114
Si f(a) existe (condición (i) de la definición 2.9) las otras dos condiciones se encuentran asi: La
existencia del límite se obtiene de la def. 2.7a con  y , la igualdad con f(a), el valor de la función
en el punto a, se encuentra en el cambio del límite L con f(a) en la definición del límite de la
función. Entonces esta definición es equivalente a la definición 2.9.

Aplicando la def. 2.7b resulta una definición de la continuidad equivalente:


Definición 2.10b: La continuidad de una función en un número:
La función f es continua en el número a si f está definida en algún intervalo abierto (c, d ) que
contenga a a y si para cualquier sucesión xn  (c, d ) con lim xn  a resulta lim f ( xn )  f (a) .
n  n 

Ejemplo 1.) La demostración de la continuidad de la función f ( x)  x con la definición 2.10a:


x : R0  R0 . Por eso se demuestra la continuidad en cualquier punto (fijo, positivo pero
arbitrario) a  0 . Hay que demostrar que para cualquier   0 existe una   0 tal que
x  a    x con x  a   . Investigamos la desigualdad x  a   y transformamos
esta desigualdad a una condición para x  a . Investigamos el límite lim x para lo cual siempre
x a

x a
vale x  a  x  a  0: Por eso podemos escribir x a  ( x  a)
xa
x a xa 1
 ( x  a) 
 x a 
x a  x a
. Ahora buscamos una cota superior de
x a
:

1 1
x 0 x a a   x  0 porque x  a  0 y a  0 ( a  0 por
a x  a
xa xa 1 1 1 1
hipótesis). Resulta que  y    porque
x a x a x a a x a a
1 1 xa xa
todas las cantidades son positivos.    porque x  a  0 .
x a a x a a
xa xa
Por lo tanto resulta x a   y de esto se puede concluir: Si x  a   y
x a a
xa   a
si se define  :  a , entonces x a      . Es decir para cualquier
a a a
  0 dada resulta que x  a   para todo x con x  a   , y esto vale para a fijo, positivo
pero arbitrario, a  0 . Esto significa que f ( x)  x es continua en cualquier punto a  0 . Q.e.d.

Ejemplo 2.) La demostración de la continuidad de las funciones trigonométricas sen y cos con la
definición 2.10b:

Página 38 de 114
1.) Empezamos con la demostración que sen(x) es continua en a  0 . Según la def. 2.10b hay que
demostrar: Para cualquier sucesión xn con lim xn  0 resulta lim sen ( xn )  sen (0)  0 .
n  n 

Aplicamos la desigualdad 2.14b.(3) (pág. 54), 0  sen ( xn )  xn  xn  (   2 ,  2 ) , y


aplicamos el teorema 1.3. La sucesión xn  0 no debe estar en el intervalo (   2 ,  2 ) para
n

todo índice n natural, es suficiente si vale esto  n  N1 con algún número positivo finito N1 ,
porque esto es suficiente para la aplicación del teorema 1.3. Y esta hipótesis la cumple cualquier
sucesión xn  0 : Los primeros valores de la sucesión pueden estar en cualquier intervalo, los
n 

últimos, para todo n mayor que un número finito N1 , todos satisfacen la condición
xn  (   2 ,  2 ) , porque a causa de la convergencia a cero siempre se puede encontrar un N1
para lo cual vale la condición. Entonces se puede aplicar el teorema 1.3 para todas las sucesiones
xn  0 . Porque xn  0 es equivalente a xn  0 (ver el comentario abajo de la def. 1.2) y
n  n  n 

porque 0  0 resulta con la ayuda del teorema 1.3 que sen( xn )  0 lo cual es equivalente a
n n 

sen ( xn )  0 . Porque esto vale para cualquier sucesión xn  0 (aquí no hemos necesitado alguna
n n 
propiedad particular de las sucesiones menos que convergen a cero), según la def. 2.10b resulta
que sen(x) es continua en a  0 . Q.e.d.
2.) Seguimos con la demostración que también cos(x) es continua en a  0 . Se puede expresar el
cos como función de sen: Según el teorema 2.14a,(ii) vale cos( x)  cos( x 2  x 2)
 cos2 ( x 2)  sen 2 ( x 2) . Según el teorema de Pitágoras en el círculo unitario vale
sen 2 ( x)  cos2 ( x)  1  x . De esto resulta cos2 ( x)  1  sen 2 ( x)  x , entonces se puede escribir
cos2 ( x 2)  1  sen 2 ( x 2) . Por lo tanto se tiene cos( x)  1  2sen 2 ( x 2) . Según el teorema 2.1 y
según la def. 2.9 resulta que g ( x)  x 2 es una función continua en todo x  R . Porque ya hemos
demostrado en el inciso 1.) que f ( x )  sen ( x ) es continua en a  0 resulta con la ayuda del
teorema 2.9c que la función compuesta f  g ( x)   sen ( x 2) es continua en a  0 . Con la ayuda
del teorema 2.8(iii) resulta que sen 2 ( x 2) es continua en a  0 . Y finalmente la combinación
lineal de la función continua constante 1 y de sen 2 ( x 2) es continua en a  0 según el
teorema 2.8(ii) y (ib). Por lo tanto hemos demostrado que cos( x)  1  2sen 2 ( x 2) es continua en
a  0 . Según la def. 2.10b esto significa que para cualquier sucesión xn con lim xn  0 resulta que
n 

lim cos( xn )  cos(0)  1 . Q.e.d.


n 

3.) Seguimos con la demostración que sen(x) es continua en todo punto a  R . Según la def. 2.10b
hay que demostrar: Para cualquier sucesión xn con lim xn  a resulta lim sen ( xn )  sen (a) .
n  n 

lim xn  a es equivalente a lim ( xn  a)  0 (ver el comentario posterior a la def.2.7b y después


n  n 

de la def.1.2). Definimos con hn : xn  a  xn  hn  a  n  N una sucesión que converge a cero


( lim hn  0 ) y calculamos lim sen ( xn )  lim sen (hn  a) aplicando el teorema 2.14a,(i):
n  n  n 

lim sen (hn  a)  lim sen (hn ) cos( a)  cos( hn )sen( a)  . Ya hemos demostrado con los incisos 1.) y
n  n 

Página 39 de 114
2.) que vale para cualquier sucesión hn con lim hn  0 resulta que lim sen (hn )  sen (0)  0 y resulta
n  n 

que lim cos(hn )  cos(0)  1 . Por lo tanto se tiene lim sen ( xn )


n  n 

 cos( a)  lim sen (hn )  sen( a)  lim cos( hn )  sen( a) . Esto vale para cualquier a  R . Q.e.d.
n  n 

4.) Terminamos esta demostración demostrando que cos(x) es continua en todo punto a  R . Esto
vamos a hacer de manera análoga como en el inciso 3.). Según la def. 2.10b hay que demostrar:
Para cualquier sucesión xn con lim xn  a resulta lim cos( xn )  cos( a) . Definimos con
n  n 

hn : xn  a  xn  hn  a  n  N una sucesión que converge a cero ( lim hn  0 ) y calculamos


n 

lim cos( xn )  lim cos( hn  a) aplicando el teorema 2.14a,(ii):


n n 

lim cos( hn  a)  lim cos( hn ) cos( a)  sen (hn )sen( a )  . Ya hemos demostrado con los incisos 1.) y
n  n 

2.) que vale para cualquier sucesión hn con lim hn  0 resulta que lim sen (hn )  sen (0)  0 y resulta
n  n 

que lim cos(hn )  cos(0)  1 . Por lo tanto se tiene lim cos( xn )


n  n 

 cos(a )  lim cos(hn )  sen( a)  lim sen (hn )  cos(a) . Esto vale para cualquier a  R . Q.e.d.
n  n 

Ya hemos demostrado en cuatro pasos que las dos funciones trigonométricas sen y cos son
continuas en todo su argumento.

De esto resulta también que la función tan(x) es continua en todo su argumento, menos para los
xz  (2 z  1)  2 , z  Z , para los cuales vale cos( xz )  0 , porque se puede aplicar el
teorema 2.8(iv) a tan( x)  sen ( x) cos( x) . En xz  (2 z  1)  2 , z  Z , la función tan(x) no está
definida y por eso es discontinua en todo xz  (2 z  1)  2 para todo z  Z .

De manera semajante resulta que la función cot(x) es continua en todo su argumento, menos para
los xz  z  , z  Z , para los cuales vale sen ( xz )  0 , porque se puede aplicar el teorema 2.8(iv)
a cot( x)  cos( x) sen ( x) . En xz  z  , z  Z , la función cot(x) no está definida y por eso es
discontinua en todo xz  z  para todo z  Z .

Definición 2.11: La continuidad de una función en un intervalo abierto:


Se dice que una función, cuyo dominio contiene al intervalo abierto ( a, b) , es continua en el
intervalo abierto ( a, b) si y sólo si es continua en cada número x del intervalo abierto ( a, b) ; f
continua (segun definición 2.9) x  (a, b) .
Según la definición 2.9 es necesario que podamos calcular el límite bilateral, que significa el
límite por la derecha y por la izquierda en todo punto del intervalo abierto ( a, b) . Porque se trata
de un intervalo abierto, cada punto en este intervalo abierto tiene un -vecindad
 
V ( x0 ) : x x  x0   que es un subconjunto de este intervalo abierto: U  ( x0 )  (a, b) . Eso

Página 40 de 114
significa que para cada punto x0  (a, b) existe un  tal que vale U  ( x0 )  (a, b) . Entonces en un
intervalo abierto para todo x0  (a, b) podemos calcular el límite bilateral lim f ( x) lo cual
x x 0

necesitamos para demostrar la continuidad de la función f en el punto x0. Entonces en un intervalo


abierto no tenemos problemas en los puntos x0 cerca a ó b.
Pero si trabajamos con un intervalo cerrado [ a, b] , si tenemos un problema si queremos calcular
el límte lim f ( x) o lim f ( x ) . En este intervalo cerrado [ a, b] no podemos calcular el límite por la
x a x b

izquierda en a lim f ( x) lo cual necesitamos para el límite bilateral lim f ( x ) y tampoco podemos
xa x a

calcular el límite por la derecha en b lim f ( x) lo cual necesitamos para el límite bilateral lim f ( x )
x b x b

. Por eso necesitamos dos definiciones nuevas:

Definición 2.12a: La continuidad por la derecha:


La función f es continua por la derecha en el número a si y solo si se cumplen las tres
condiciones siguientes:
(i) f(a) existe
(ii) lim f ( x) existe (ver definición 2.8a)
xa

(iii) lim f ( x)  f (a)


xa

Definición 2.12b: La continuidad por la izquierda:


La función f es continua por la izquierda en el número a si y solo si se cumplen las tres
condiciones siguientes:
(i) f(a) existe
(ii) lim f ( x) existe (ver definición 2.8b)
xa

(iii) lim f ( x)  f (a)


xa

Ahora podemos definir la continuidad de una función en un intervalo cerrado:


Definición 2.13: La continuidad de una función en un intervalo cerrado:
Se dice que una función, cuyo dominio contiene al intervalo cerrado [ a, b] , es continua en el
intervalo cerrado [ a, b] si y sólo si es continua en el intervalo abierto ( a, b) , así como
continua por la derecha en a y continua por la izquierda en b.
Muchos teoremas posteriores necesitarán la hipótesis de que f(x) es continua en un intervalo
cerrado. Por eso la definición 2.13 es importante.

Página 41 de 114
Teorema 2.10: Valor intermedio:
Si la función f es continua en el intervalo cerrado [ a, b] y si f ( a )  f (b) , entonces (para el caso
f (a )  f (b) ) para cada valor k  R con f ( a )  k  f (b) existe al menos un número c  R con
a  c  b tal que f (c)  k . (Para el caso f (a )  f (b) esto vale para cada valor k  R con
f ( a )  k  f (b) .)

Demostración:
Primeramente demostramos un corolario que es un caso especial de este teorema: f ( a )  0 y
f (b)  0 . Entonces  c  [ a, b] con f (c )  0 :
Trabajamos con un método que utiliza sucesiones de intervalos:
a b ab
1) Empezamos con a0  a , b0  b y definimos c0 : 0 0  . En el caso f (c0 )  0 el punto
2 2
a b
buscado c tiene que estar entre a0 y c0. Por eso definimos a1 : a0 , b1 : c0  0 0 . En el caso
2
f (c0 )  0 el punto buscado c tiene que estar entre c0. y b0. Por eso definimos
a b
a1 : c0  0 0 , b1 : b0 .
2
a b
2) En un segundo paso definimos c1 : 1 1 . En el caso f (c1 )  0 el punto buscado c tiene que
2
a b
estar entre a1 y c1. Por eso definimos a2 : a1 , b2 : c1  1 1 . En el caso f (c1 )  0 el punto
2
a b
buscado c tiene que estar entre c1. y b1. Por eso definimos a2 : c1  1 1 , b2 : b1 .
2

...(y asi sucesivamente, ver la figura 2.1)...

an 1  bn 1
n) En el paso n-ésimo definimos cn 1 : . En el caso f (cn 1 )  0 el punto buscado c tiene
2
a b
que estar entre an-1 y cn-1. Por eso definimos an : an 1 , bn : cn 1  n 1 n 1 . En el caso
2
f (cn 1 )  0 el punto buscado c tiene que estar entre cn-1. y bn-1. Por eso definimos
a b
an : cn 1  n 1 n 1 , bn : bn 1 .
2

Página 42 de 114
Figura 2.1: El procedimiento del encajonamiento de intervalos

De esta manera obtenemos dos sucesiones: an nN y bn nN con an  an 1 y bn  bn 1 . También
vale a  an  b n  N y b  bn  a n  N . Entonces an nN es una sucesión creciente y
acotada y bn nN es una sucesión decreciente y acotada. Según el teorema 1.5 son sucesiones
b  a0 b a b a
convergentes. Ademas vale en cada caso: b1  a1  0 , b2  a2  1 1  0 2 0 ,
2 2 2
n
b2  a2 b0  a0 b a 1 1
b3  a3   3
, ... , bn  an  0 n 0 . Porque lim n  lim   0 (ver la
n  2
2 2 2 n   2  
demostración para el cálculo de la suma de la serie geométrica abajo de la def. 1.7: lim r  0 si
n
n 

r  1 , en este caso r  1 2  1 ) vale lim (bn  an )  0  lim bn  lim an  0  lim bn  lim an .


n n  n  n  n 

an 1  bn 1
También vale cn 1 : n  N  an  cn  bnn  N . Según el teorema 1.3 vale
2
lim cn  lim an  lim bn . Según teorema 1.5 este límite existe, lo denotamos con lim cn : L .
n  n  n  n

Arriba definimos los puntos an y bn asi que siempre vale f (an )  0 y f (bn )  0 . Por hipótesis f
es continua en el intervalo [ a, b] , por eso vale lim f (an )  f ( lim an )  f ( L) (ver definición 2.7b
n  n

y 2.10b) y también lim f (bn )  f ( lim bn )  f ( L) . Porque lim bn  lim an  lim cn  L podemos
n  n n  n  n

escribir lim f (an )  f ( lim an )  f ( lim cn )  f ( L)  f ( lim bn )  lim f (bn ) . Cada valor f (an ) es
n  n  n  n  n 

Página 43 de 114
negativo, cada valor f (bn ) es positivo, es decir esto vale para cada n natural. Como hemos
mostrado arriba vale lim f (an )  lim f (bn )  f ( L) , entonces resulta que
n n

lim f (an )  lim f (bn )  lim f (cn )  f ( L)  0 . El límite lim f (an )  lim f (bn )  K debe ser igual
n  n  n  n  n 

a cero, porque si suponemos que K  0 , entonces con K  lim f (an ) según la def. 1.2 existe un
n 

número infinito de valores n para los cuales se tiene K    f (an )  K   para cualquier   0 ,
entonces también para K    0 . Pero esto es equivalente a f (an )  0 para estos índices n, lo
que es una contradiccón al hecho que f (an )  0  n  N . De menera semejante resulta una
contradicción si suponemos que K  0 . Con K  lim f (bn ) según la def. 1.2 existe un número
n

infinito de valores n para los cuales se tiene K    f (bn )  K   para cualquier   0 , entonces
también para K    0 . Pero esto es equivalente a f (bn )  0 para estos índices n, lo que es una
contradiccón al hecho que f (bn )  0  n  N . Por eso el límite K  lim f (an )  lim f (bn ) debe
n  n

ser igual a cero. Junto con lim f (an )  lim f (bn )  K  0  lim f (cn )  f ( L) resulta que
n  n  n 

lim cn  L con lim f (cn )  f ( L)  K  0 . Asi hemos demostrado que existe un número
n  n 

c  L  [ a, b] con f (c )  0 .
Para el otro caso f ( a )  0 y f (b)  0 se puede repetir la demostración con los signos opuestos (
f (an )  0 y f (bn )  0 ) pero con exactamente la misma argumentación, talque también resulta
lim f (an )  lim f (bn )  K  0  lim f (cn )  f ( L) tal que existe un número c  L  [a, b] con
n  n  n

f (c )  0 . Q.e.d.

El caso más general (para cada valor k  R con f ( a )  k  f (b) existe al menos un número c  R
con a  c  b tal que f (c)  k ) se puede reducir a este corolario que ya demostramos: Podemos
definir una nueva función g ( x ) : f ( x )  k que es continua, por que f es continua. Además vale:
g ( a )  f ( a )  k  0 y g (b)  f (b)  k  0 . Para el caso “igual a” ya se tien la solución, es decir
si g ( a )  0  c  a y si g (b)  0  c  b . Entonces se puede suponer que vale g ( a )  0 y
g (b)  0 . Asi se cumplen todas las hipótesis del corolario anterior. Entonces existe un punto
c  [ a, b] tal que g (c )  0 . Esto es equivalente a f (c)  k  0  f (c )  k .
Para el otro caso (para cada valor k  R con f ( a )  k  f (b) existe al menos un número c  R
con a  c  b tal que f (c )  k ) se define también g ( x ) : f ( x )  k y resulta que
g ( a )  f ( a )  k  0 y g (b)  f (b)  k  0 y para los casos c  a y c  b resulta que g ( a )  0 y
g (b)  0 tal que se puede aplicar el corolario para este caso y resulta también que existe un punto
c  [ a, b] tal que f (c )  k . Q.e.d.
a b
Este método que hemos aplicado con los intervalos [ an , bn ] y cn : n n se llama
2
encajonamiento de intervalos.

Página 44 de 114
Ejemplo a): f ( x)  x 2  1 . f es continua para todo x  R . Según el teorema del valor intermedio
existe al menos un punto c con f (c )  1 : f (c)  c 2  1  1  c 2  2  c1, 2   2 .
x  1 , 0  x  2
Ejemplo b): f ( x)   2 . La función f es discontinua en 2, el cual está en el intervalo
 x , 2 x3
[0,3] . f (0)  1 , f (3)  9 . Si k es cualquier número entre 1 y 4, entonces no hay ningún valor de
c tal que f (c )  k porque no existen valores de la función entre 1 y 4.

2.4. DERIVADA Y DIFERENCIACIÓN

2.4.1. DEFINICIÓN Y REGLAS


Definición 2.14: La derivada de una función:
La derivada de la función f es aquella función, denotada por f ' , tal que su valor en un número x0
del dominio de f está dada por
f ( x0  x)  f ( x0 )
f ' ( x0 ) : lim si este límite existe.
x  0 x
En el caso que existe este límite (el límite siempre es único) para un conjunto de puntos x0 y si
definimos el conjunto de todos los puntos x0 para los cuales existe este límite como Z, entonces
se tiene una correspondencia única de Z en R, es decir se puede definir una función que se denota
como f ' . Observar que el dominio Z de f ' es un subconjunto del dominio X de f (es decir Z  X
) y este dominio Z debe ser un conjunto abierto, porque asi x0  x siempre pertenece a Z y por
eso también a X, es decir x0  x  Z para x  0 y para x  0 , talque f ( x0  x) siempre es
bien definido (ver las explicaciones abajo de la def.2.11).

La recta secante que toca la gráfica de la función f en los dos puntos  x0 , f ( x0 )  y

x0  x, f ( x0  x) tiene la pendiente m( x0 )  f ( x0  x)  f ( x0 )  y . Si calculamos el límite


x x
x  0 encontramos la pendiente de la recta tangente a la gráfica de la función en el punto
x0 , f ( x0 )  . La recta tangente toca la gráfica de la función f nada más en un punto, en x0 , f ( x0 )  .
Observa que la pendiente de la recta tangente a la gráfica de la función en el punto  x0 , f ( x0 )  es
precisamente la derivada de f evaluada en x0 .

Ejemplo 1: Derivada de una función lineal:

Página 45 de 114
f ( x )  mx  b , m, b  R .
f ( x0  x)  f ( x0 ) m x0  x   b  mx0  b m  x
f ' ( x0 ) : lim  lim  lim m. Este
x  0 x x  0 x x  0 x

resultado vale para todo x0  R , entonces la derivada de una función lineal es la función constante
f ' ( x)  m .

1
Ejemplo 2: f ( x)  . Si x0 es un número del dominio de f, entonces
x
1 1 x0   x0  x 

f ( x0  x)  f ( x0 ) x  x x0 x   x  x 
f ' ( x0 ) : lim  lim 0  lim 0 0
x  0 x x  0 x x  0 x
 x 1 1
 lim 2  lim 2  2 (ver las reglas para límites y teorema 2.9b). Este
 x  0 x  x  x  x 2
0  x0  x
x  0 x x0
0 0

1 1
resultado vale para todo x0  R \ 0. Entonces la derivada de f ( x)  es f ' ( x)   2 , x  0
x x
(escribimos x en lugar de x0 ).

f ( x0  x)  f ( x0 ) f ( x0  h)  f ( x0 )
f ' ( x0 ) : lim (o f ' ( x0 ) : lim ); considera x0  x : x .
x  0 x h  0 h
Entonces x  0 equivale x  x0 . Asi se obtiene la fórmula siguiente para f ' ( x0 ) :
f ( x)  f ( x0 )
f ' ( x0 ) : lim si este límite existe.
x  x0 x  x0
f ( x0  x)  f ( x0 ) f ( x)  f ( x0 )
Los cocientes y reciben el nombre de cocientes de diferencias
x x  x0
estándar de la función f en el número x0.

Si (x,y) es un punto de la gráfica de f, entonces y = f(x). y' se utiliza también como una notación
para la derivada de f(x). Con la función f definida por la ecuación y = f(x) se considera que
y  f ( x  x )  f ( x ) (i) donde y se denomina incremento de y y denota un cambio en el valor
dy
de la función cuando x varía en x . Al utilizar (i) y escribir en lugar de f ' ( x ) , la fórmula
dx
f ( x0  x)  f ( x0 ) dy y
f ' ( x0 ) : lim se transforma en  lim . Porque se puede escribir
x  0 x dx x  0 x
dy d d
 ( y ) se puede considerar como un operador, se dice una corespondencia entre la
dx dx dx
d
función f y su derivada f ' , cual es una función también: : f  f '.
dx

Página 46 de 114
El uso del símbolo f ' ( x) para la derivada de la función f fue introducido por el matemático francés
dy
Joseph Louis Lagrange (1736-1813). El símbolo fue empleado como notación para la derivada
dx
por primera vez por el matemático alemán Gottfried Wilhelm Leibniz (1646-1716). En el siglo
XVII Leibniz y Sir Isaac Newton (1642-1727), trabajando de manera independiente, dieron a
aconocer casi simultáneamente la derivada.

Definición 2.15: La derivada lateral:


(i) Si la función f está definida en x0, entonces la derivada por la derecha de f en x0, denotada
por f ' ( x0 ) , está definida por
f ( x0  x)  f ( x0 ) f ( x)  f ( x0 )
f ' ( x0 ) : lim  f ' ( x0 ) : lim si existe el límite.
x  0 x x  x0 x  x0
(ii) Si la función f está definida en x0, entonces la derivada por la izquierda de f en x0, denotada
por f ' ( x0 ) , está definida por
f ( x0  x)  f ( x0 ) f ( x)  f ( x0 )
f ' ( x0 ) : lim  f ' ( x0 ) : lim si existe el límite.
x  0 x x  x0 x  x0

A partir de esta definición y del teorema 2.7, se deduce que una función f definida en un intervalo
abierto que contiene a x0 es diferenciable en x0 si y sólo si f ' ( x 0 ) y f ' ( x0 ) existen y son iguales.

Ejemplo: f ( x)  1  x 2 . 1  x 2  0  1  x 2  1  x  1  x ( x  0) o  1  x ( x  0) , en este caso


vale f ( x)  1  x 2 . 1  x 2  0  1  x 2  1  x  1  x ( x  0) o  1  x ( x  0) , en este caso vale
 x 2  1 si x  1 o x  1
f ( x)  x 2  1 . Entonces vale: f ( x)   .
1- x
2
si  1  x  1
f ( x)  f (1) ( x 2  1)  0 ( x  1)( x  1)
f ' (1)  lim  lim  lim  lim  x  1   2 .
x 1 x 1 x 1 x 1 x 1 x 1 x 1

f ( x)  f (1) (1  x )  0
2
(1  x)(1  x)
f ' (1)  lim  lim  lim  lim  (1  x)  2 .
x 1 x 1 x 1 x 1 x 1 x 1 x 1

Debido a que f ' (1)  f ' (1) se concluye que f ' (1) no existe, de modo que f no es diferenciable
en 1. Pero se puede demostrar que f sí es continua en 1. La función f tampoco es diferenciable en
-1, pero es continua en -1.
Es posible que una función sea continua en un punto x0, y sin embargo no sea diferenciable en este
punto. Pero si la función es diferenciable en un punto x0, entonces siempre es continua en este
punto (ver teorema 2.11).

Página 47 de 114
Teorema 2.11:
Si la función f es diferenciable en un número x0, entonces f es continua en x0.
Demostración:
f ( x)  f ( x0 )
Por hipótesis f es diferenciable en x0. Por tanto, existe f ' ( x0 ) : lim . f ( x0 ) existe;
x  x0 x  x0
en otro caso el límite anterior no tendría significado. Por tanto, se cumple la condición (i) de la
definición 2.9.
 f ( x)  f ( x0 ) 
Ahora considera lim  f ( x)  f ( x0 )  lim ( x  x0 )   . Como xlim ( x  x0 )  0 y
x  x0 x  x0
 x  x0 
 x0

f ( x)  f ( x0 )
lim  f ' ( x0 ) se aplica el teorema del límite de un producto (ver teorema 2.3) y se
x  x0 x  x0
 f ( x)  f ( x0 ) 
obtiene lim  f ( x)  f ( x0 )    lim ( x  x0 )   lim   0  f ' ( x0 )  0 . Vale
x  x0  x  x0   x  x 0 x  x0 
lim  f ( x)  f ( x0 )  0  lim f ( x)  f ( x0 ) . Asi se concluye que se cumplen las condiciones (ii)
x  x0 x  x0

y (iii) para la continuidad de f en x0. q.e.d.

Ejemplo 1: La función lineal es diferenciable y continua en cada punto x  R (ver ejemplo 1 más
arriba, así como el teorema 2.1).

Ejemplo 2: La función f ( x )  x es diferenciable y contínua en todo x  R \ 0, pero esta función


es contínua en 0, pero no es diferenciable en 0: Los dos límites por la derecha y por la izquierda
en x0  0 son iguales y son iguales a 0. Entonces f ( x)  x es continua en 0 y en todo otro número,
porque es una función lineal para x  0 y para x  0 .
Pero si calculamos el cociente de diferencias en el punto x0  0 , encontramos que el límite por la
derecha no es igual al límite por la izquierda, lo que significa que no existe el límtie bilateral en
f ( x )  f ( 0)  x0 x f ( x )  f ( 0) x0
x0  0 : lim  lim  lim   1 y lim  lim  1.
x 0 x0 x  x0 x0 x  x0 x x 0 x0 x  x0 x  0

f ( x)  f (0)
lim  lim . Entonces no existe el límite lim y f ( x)  x no es diferenciable en
x 0  x 0 x 0 x0
x0  0 (ver definición 2.15). Entonces f ( x)  x es contínua para todo x  R pero es
diferenciable solamente en todo x  R \ 0. La continuidad en todo x  R \ 0 es una
consecuencia de la diferenciabilidad en estos puntos. La continuidad en x  0 es un caso
particular.

Página 48 de 114
La derivada de f ( x)  x , n  N :
d xn
n
 lim
 x0  h   x0
n n

dx x  x0
h 0 h

n  n  n 1  n  n  2 n n
x0    x0 h    x0 h 2     h n  x0
 lim 1  2 n
h 0 h
 n  n 1  n  n  2 n  n 1
 lim   x0    x0 h     h n 1   n  x0 .
h  0  1   2 n 
d n n 1
Por eso vale x  n  x0 , x0  R , n  N .
dx x  x0

También se puede escribir


d n
dx
'
 
x  x n  n  x n 1 , x  R , n  N .
Aqui necesitamos los coeficientes binomiales y el teorema del binomio:
 n!
n  ,0k n
  :  k!(n  k )! . (Caso especial para cero factorial: 0! : 1 , además vale
k   0 ,0nk

 n  n  n
      1 ,    n ). Los valores de los coeficientes binomiales se pueden obtener
 0  n 1
sucesivamente del triángulo de Pascal.
 4 4! 1  2  3  4 24
Por ejemplo:   :    6.
 2  2!(4  2)! 1  2  1  2 4

Teorema 2.12: Teorema del binomio (binomio de Newton)


Para todo a, b  R \ 0 y para todo n  N vale:

a  b n      a n  k  b k     a n  b0     a n 1  b1     a n  2  b2       a 0  bn .
n n n n n n
k 0  k   0 1  2  n
(sin demostración).

Si f es una función constante (f(x)=c), su derivada es cero:


f ( x)  f ( x0 ) cc
f ' ( x0 )  lim  lim  0 . Eso vale para todo x0  R , entonces vale:
x  x0 x  x0 x  x 0 x  x
0

f ( x )  c (c  R ), f ' ( x )  0 x  R .

Página 49 de 114
Teorema 2.13: Reglas de la diferenciación:
f y g sean funciones diferenciables en el punto x0. c  R sea una constante arbitraria.
1) Producto de una función por una constante: (c  f )' ( x0 )  c  f ' ( x0 ) (Homogeniedad de primer
d
orden de la derivada:  c  f ( x)   c  d f ( x) evaluado en x  x0 ).
dx dx
2) Suma de dos funciones: ( f  g )' ( x0 )  f ' ( x0 )  g ' ( x0 ) (Distributividad de la derivada:
d
 f ( x)  g ( x)  d f ( x)  d g ( x) evaluado en x  x0 ).
dx dx dx

De la regla 1) y 2) se sigue con la ayuda de la ley asociativa de la adición una generalización que
indica la linealidad de la derivada: Sean fi , i  1,2,, n , funciones diferenciables en x0 y
ci , i  1,2,, n , sean constantes reales, fijos pero arbitrarios, entonces vale:
d  n  n d
 
dx  i 1
ci  f i ( x )    ci 
 i 1 dx
f i ( x) evaluado en x  x0 .

3) Producto de dos funciones: ( f  g )' ( x0 )  f ' ( x0 )  g ( x0 )  f ( x0 )  g ' ( x0 ) .



f  f ' ( x0 )  g ( x0 )  f ( x0 )  g ' ( x0 )
4) Cociente de dos funciones ( g ( x0 )  0 ):   ( x0 )  .
g g ( x0 )2
(Regla 3) y 4) son reglas fuera de la linealidad de la derivada)
Demostración:
1)
(cf )( x0  h)  (cf )( x0 ) c  f ( x0  h)  c  f ( x0 )  f ( x0  h)  f ( x0 ) 
(cf )' ( x0 )  lim  lim  lim c  
h 0 h h  0 h h  0
 h
(ver definición 2.3 y las reglas para límites teoremas 2.1-2.4).
f ( x0  h)  f ( x0 )
(cf )'  c  lim  c  f ' ( x0 ) . q.e.d.
h 0 h
Ejemplo ilustrativo: f ( x)  c  x n . f ' ( x)  c  ( x n )'  c  nx n 1 .
2)
( f  g )' ( x0 )  lim
( f  g )( x0  h)  ( f  g )( x0 )
 lim
 f ( x0  h)  g ( x0  h)   f ( x0 )  g ( x0 ) 
h 0 h h 0 h
f ( x0  h)  f ( x0 )  g ( x0  h)  g ( x0 )  f ( x0  h)  f ( x0 ) g ( x0  h)  g ( x0 ) 
 lim  lim    
h 0 h h0
 h h
f ( x0  h)  f ( x0 ) g ( x0  h)  g ( x0 )
 lim  lim  f ' ( x0 )  g ' ( x0 ) . q.e.d.
h 0 h h 0 h
(ver definición 2.3 y las reglas para límites teoremas 2.1-2.4).
Ejemplo ilustrativo: f ( x)  2 x 2  5 x 3 .
f ' ( x)  (2 x 2  5 x 3 )'  (2 x 2 )'(5 x 3 )'  2  2 x  5  3x 2  4 x  15 x 2 .

3) ( f  g )' ( x0 )  lim
( f  g )( x0  h)  ( f  g )( x0 )
 lim
 f ( x0  h)  g ( x0  h)   f ( x0 )  g ( x0 ) 
h0 h h 0 h
Página 50 de 114
f ( x0  h)  g ( x0  h)  f ( x0  h)  g ( x0 )  f ( x0  h)  g ( x0 )  f ( x0 )  g ( x0 )
 lim 
h 0 h
 g ( x0  h)  g ( x0 ) f ( x0  h)  f ( x0 ) 
 lim  f ( x0  h)    g ( x0 ) 
h 0
 h h 
 g ( x0  h)  g ( x0 )   f ( x0  h)  f ( x0 ) 
 lim  f ( x0  h)    lim   g ( x0 ) 
h 0
 h  h  0
 h 
(ver definición 2.3 y las reglas para límites teoremas 2.1-2.4). f es diferenciable en x0 por
hipótesis. Segun teorema 2.11 f también es continua en x0! Por eso vale lim f ( x0  h)  f ( x0 ) ,
h 0

porque segun definición 2.10 y definición 2.7b vale lim f ( xn )  f ( lim xn  x0 ) para cualquier
n n 

sucesión que converge a x0 , entonces también para lim( x0  h)  x0 . Entonces podemos escribir:
h 0

g ( x0  h)  g ( x0 ) f ( x0  h)  f ( x0 )
( f  g )' ( x0 )  lim f ( x0  h)  lim  lim  lim g ( x0 ) 
h 0 h0 h h  0 h h 0

 f ( x0 )  g ' ( x0 )  f ' ( x0 )  g ( x0 ) . q.e.d.



Ejemplo ilustrativo: f ( x)  2 x 3  4 x 2  3x 5  x 2 .  
 
     
f ' ( x)  2 x 3  4 x 2  3x 5  x 2  2 x 3  4 x 2  3 x 5  x 2  
    
 6 x 2  8 x  3x 5  x 2  2 x 3  4 x 2  15x 4  2 x  
 18 x 7  6 x 4  24 x 6  8 x 3  30 x 7  4 x 4  60 x 6  8 x 3  48 x 7  84 x 6  10 x 4  16 x 3 .
  
Prueba: f ( x)  2 x 3  4 x 2  3x 5  x 2  6 x8  2 x 5  12 x 7  4 x 4 .
f ' ( x)  48 x  84 x  10 x  16 x , es correcto.
7 6 4 3

f f ( x0 ) 1
4)  ( x0 )   f ( x0 )  . Entonces vale segun 3):
g g ( x0 ) g ( x0 )
  
f  1  1   1 
  ( x0 )  f ' ( x0 )   f ( x0 )    . Tenemos que calcular   con la definición
g g ( x0 )  g ( x0 )   g ( x0 ) 
1 1 g ( x0 )  g ( x0  h)
 
 1  g ( x0  h) g ( x0 ) g ( x0 )  g ( x0  h)
de la derivada:    lim  lim 
 g ( x0 )  h  0 h h 0 h
 1 g ( x0 )  g ( x0  h)  1  g ( x0  h)  g ( x0 )) 
 lim     lim  lim  
h  0 g ( x )  g ( x  h) h g ( x0 )  g ( x0  h) h
 0 0  h  0 h  0

(ver las reglas para límites teoremas 2.1-2.4). g es diferenciable en x0 por hipótesis. Segun
teorema 2.11 g también es continua en x0. Por eso vale lim g ( x0  h)  g ( x0 ) . Entonces
h 0


 1  1 g ' ( x0 )
podemos escribir:      g ' ( x0 )   . Ahora podemos calcular el
 g ( x0 )  g ( x0 )  g ( x0 ) g ( x0 )2

Página 51 de 114
 
f 1  1  f ' ( x0 )  g ' ( x0 ) 
siguiente:   ( x0 )  f ' ( x0 )   f ( x0 )      f ( x0 )   2

g g ( x0 )  g ( x0 )  g ( x0 )   g ( x0 )  
f ' ( x0 )  g ( x0 ) f ( x0 )  g ' ( x0 ) f ' ( x0 )  g ( x0 )  f ( x0 )  g ' ( x0 )
   . q.e.d.
g ( x0 )2 g ( x0 ) 2 g ( x0 ) 2
 
Ejemplo ilustrativo: f ( x)  2
2 x3  4
. f ' ( x) 
    
2 x3  4  x 2  1  2 x3  4  x 2  1  

x 1 
x2  1
2

f ' ( x) 
2
 2
 
6x  x  1  2x  4  2x 3
 
6 x  6 x  4x  8x
4 2 4

2x  6x2  8x
4
.
x 2

1
2
x 2
1 2
x 2
1 2

Teorema 2.13.i: Propiedad particular de una función diferenciable:


f : X  R  Y  R sea una función diferenciable en el punto interior x0  X .
La diferenciabilidad según def. 2.14 es equivalente a la propiedad siguiente:
Se puede expresar el incremento de la función exactamente como
r ( h)
f ( x0  h)  f ( x0 )    h  r (h) con una función de resta r (h) con la propiedad lim  0 . En
h 0 h

este caso vale   f ' ( x0 ) .

Demostración (Siehe Heuser, Analysis 1, Seite 267 und 268!!!): Si f es diferenciable en el punto
f ( x0  h)  f ( x0 )
x0, entonces existe el límite f ' ( x0 ) : lim . Se puede definir la función de resta
h 0 h
r (h) f ( x0  h)  f ( x0 )
r (h) : f ( x0  h)  f ( x0 )  f ' ( x0 )  h . En este caso resulta que   f ' ( x0 )
h h
r ( h)  f ( x0  h)  f ( x0 ) 
h  0 (como vale para el límite h  0 ) y se sigue lim  lim   f ' ( x0 )
h 0 h h
h 0
 
f ( x0  h)  f ( x0 ) f ( x0  h)  f ( x0 )
 lim  f ' ( x0 )  0 porque por hipótesis vale lim  f ' ( x0 ) .
h 0 h h  0 h
Con eso se puede expresar el incremento de la función de manera siguiente:
r ( h)
f ( x0  h)  f ( x0 )  f ' ( x0 )  h  r (h) con lim  0 . (ver abajo *1) )
h 0 h

La equivalencia se puede demostrar de manera siguiente:


Se supone que se puede expresar el incremento de la función exactamente como
f ( x0  h)  f ( x0 )  f ' ( x0 )  h  r (h) con f ' ( x0 ) igual a un número constante real, o escrito en
una forma más general: f ( x0  h)  f ( x0 )    h  r (h) con   R . Si además existe una función
r ( h) f ( x0  h)  f ( x0 )
de resta r (h) con la propiedad lim  0 , entonces resulta que lim
h 0 h h 0 h
 r ( h)  r ( h) r ( h)
 lim       lim   porque por hipótesis vale lim  0 . Entonces existe el
h 0
 h  h  0 h h  0 h
f ( x0  h)  f ( x0 )
límite lim y es igual al número definido arriba  , es decir resulta que f es
h 0 h
Página 52 de 114
diferenciable en el punto x0 y vale f ' ( x0 )   .  juega el papel del valor de la derivada de f en el
punto x0. Q.e.d.
*1) La función f ya es diferenciable en el sentido más general, por ejemplo con esta
diferenciabilidad ya se sigue que f es continuo en todos los puntos donde f es diferenciable (ver
teorema 2.11). Es decir no es necesario que f es diferenciable con continuidad ( f  C 1 ) como se
necesita para las derivadas parciales en el teorema 1.6 del manuscrito “Cálculo de Varias
Variables, los teoremas y las definiciones” para demostrar que un campo escalar f  C1 es
diferenciable total (o diferenciable de manera general según Fréchet)! Por eso ya es posible
expresar el incremento de esta función f diferenciable en x0 de una manera diferente sin más
r ( h)
hipótesis: Se tiene f ( x0  h)  f ( x0 )    h  r (h) con la propiedad lim  0 (ver arriba). Se
h 0 h

r ( h) r ( h)
define  (h) : , h  0 , es decir  (h) es finito para todo h  0 y lim  (h)  lim  0 por
h h 0 h

  
h 0

hipótesis. Resulta que lim r (h)  lim  (h)  h   lim  (h)   lim h  0  0  0 . Entonces se
h 0 h0 h0 h0

tiene f ( x0  h)  f ( x0 )    h   (h)  h      ( h)   h . Se puede definir s( x0 ; h) :    (h) .


El límite de esta expresión es lim s ( x0 ; h)  lim     (h)   lim   lim  (h)  f ' ( x0 ) porque
h0 h0 h 0 h 0

lim  (h)  0 y   f ' ( x0 ) , ver arriba. Asi se obtiene una expresión del incremento de una
h 0

función diferenciable alternativa: f ( x0  h)  f ( x0 )  s ( x0 ; h)  h con lim s ( x0 ; h)  f ' ( x0 ) .


h 0

Con la ayuda del teorema del valor medio (teorema 2.21) se puede expresar el incremento de una
función unidimensional diferenciable con continuidad también de esta manera siguiente, que se
aplica por ejemplo en el teorema 1.6 del manuscrito “Cálculo de Varias Variables, los teoremas y
las definiciones”: f ( x0  h)  f ( x0 )  f ' (c)  h con una c  ( x0 , x0  h)  c  x0  t  h con
t  ( 0 ,1) . Esto se puede escribir como
f ( x0  h)  f ( x0 )  f ' ( x0 )  h  [ f ' ( x0  t  h)  h  f ' ( x0 )  h] .
Se define  ( x0 ; t  h) : [ f ' ( x0  t  h)  f ' ( x0 )] .
Entonces se tiene f ( x0  h)  f ( x0 )  f ' ( x0 )  h   ( x0 ; t  h)  h   f ' ( x0 )   ( x0 ; t  h)  h . Se
define s( x0 ; t  h) :  f ' ( x0 )   ( x0 ; t  h) y resulta que f ( x0  h)  f ( x0 )  s( x0 ; t  h)  h con
lim s ( x0 ; t  h)  lim  f ' ( x0 )   ( x0 ; t  h)  lim f ' ( x0 )  lim  ( x0 ; t  h)
h 0 h 0 h0 h0

 f ' ( x0 )  lim  ( x0 ; t  h) con lim  ( x0 ; t  h)  lim [ f ' ( x0  t  h)  f ' ( x0 )]


h0 h 0 h 0

 lim f ' ( x0  t  h)  f ' ( x0 )  f ' ( x0 )  f ' ( x0 )  0 , si la primera derivada f ' es continua en x0 .


h 0

Por lo tanto se tiene f ( x0  h)  f ( x0 )  s( x0 ; t  h)  h con lim s ( x0 ; t  h)  f ' ( x0 ) .


h 0

Presentamos este cálculo porque se necesita un cálculo parecido en el teorema 1.6 mencionado
arriba. Pero repetimos que no es necesario que una función unidimensional sea diferenciable con
continuidad para que resulta esta expresión de su incremento alternativa, como hemos demostrado
arriba!

Página 53 de 114
2.4.2. DERIVADAS DE LAS FUNCIONES TRIGONOMÉTRICAS

Teorema 2.14a: teorema de la adición


(i) sen(a  b)  sen(a) cos(b)  cos(a )sen (b)
(ii) cos(a  b)  cos (a) cos(b)  sen (a )sen (b) .
(Sin demostración (x)).

sen(h) cos(h)  1 1 - cos(h)


Teorema 2.14b: Vale lim  1 y lim  lim 0.
h 0 h h  0 h h  0 h
Demostración:

En la Figura 2.2 vemos un segmento de un círculo con radio 1 (en azul). Para el ángulo h indicado
en la gráfica vale: sen(h)  BC r  BC porque vale r  1 . Aplicando el teorema de Pitágoras en
2 2 2 2 2 2 2
el triángulo CAB resulta: AB  BC  AC con AC  0  AC  BC  BC y con la
2 2
ecuación que resulta del teorema de Pitágoras se sigue AB  BC . Porque la función x es
estrictamente creciente para todo x  0 (ver la def. 2.20) y también la función x 2 es estrictamente
2 2
creciente para todo x  0 , AB  BC es equivalente a AB  BC (las dos cantidades son
2
longitudes, por eso son positivos: AB  AB  AB , el mismo vale para BC ). La longitud del
arco entre los puntos A y B, denotada por arco( AB ) , es estrictamente mayor que la longitud del
segmento de recta entre los puntos A y B, denotado por AB . Si el ángulo h está definido en
radianes, vale h  arco( AB) . Por lo tanto se tiene BC  sen(h)  AB  arco( AB)  h y por eso
resulta sen (h)  h ,  h  ( 0 ,  2 ) .

Figura 2.2: Círculo unitario con sen (h)  BC y h  arco( AB ) .

Página 54 de 114
AD
Además vale arco( AB)  AD y tan(h)   AD porque 0 A  r  1. Entonces de
0A
arco( AB)  AD se sigue h  tan(h) . Por lo tanto se tiene sen (h)  h  tan(h) ,  h  ( 0 ,  2 ) ,
demostrado con los argumentos geométricos de arriba. Para el caso h  0 vale
sen (0)  h  0  tan(0) . Asi resulta la cadena de desigualdades
sen (h)  h  tan( h)  h  ( 0 ,  2 ) y
sen (h)  h  tan(h)  h  [ 0 ,  2 ) 2.14b.(1)
Ahora investigamos el caso h  (   2 , 0 ) : Se define k : h con h  0 (más excato
h  (   2 , 0 ) ), por eso vale k  0 (más exacto k  ( 0 ,  2 ) ). Según 2.14b.(1) vale entonces
sen ( k )  k  tan(k ) . Investigamos la primera parte de la cadena de desigualdades: sen (k )  k
 sen (  h)   h  sen ( h)   h (porque la función sen es impar)  sen ( h)  h
h  (   2 , 0 ) . De manera semajante resulta de la segunda parte de las desigualdades:
k  tan(k )   h  tan(h)   h   tan(h) (porque la función tan es impar)
 h  tan(h)  h  (   2 , 0 ) . Si combinamos estos dos resultados se sigue:
sen (h)  h  tan(h)  h  (   2 , 0 ) y
sen (h)  h  tan(h)  h  (   2 , 0 ] 2.14b.(2)
Se pueden escribir estas dos ecuaciones 2.14b.(1) y 2.14b.(2) de una forma más compacta:
sen (h)  h  tan(h)  h  (   2 ,  2 ) 2.14b.(3)
La igualdad solamente vale para h
A causa de la definición del valor absoluto de 2.14b.(3) se sigue 2.14b.(1) para el caso
h  [ 0 ,  2 )  sen ( h)  0  sen ( h)  sen ( h) y h  [ 0 ,  2 )  tan ( h)  0  tan ( h)  tan ( h) .
De 2.14b.(3) se sigue 2.14b.(2) para el caso h  (   2 , 0 )  sen (h)  0  sen (h)  sen (h) y
h  (   2 , 0 )  tan (h)  0  tan (h)   tan (h) .
sen(h) sen (h)
De 2.14b.(3) se sigue: h  tan(h)   cos(h)  lo cual vale para
cos(h) h
h  (   2 ,  2 ) \  0  . Además vale cos(h)  0 y cos(h)  cos(h) h  (   2 ,  2 ) . De
sen (h)
2.14b.(3) se sigue también sen (h)  h   1 para h  (   2 ,  2 ) \  0 . Si
h
combinamos las desigualdades resulta
sen (h)
cos( h)   1  h  (   2 , 2 ) \ 0  2.14b.(4)
h
Con la ayuda de esta ecuación 2.14b.(4) se puede calcular los siguientes límites de funciones:
sen(h) cos(h)  1 sen(h)
lim y lim . Si se aplica la def. 2.7b lim debe existir como el límite único
h 0 h h  0 h h  0 h
sen (hn )
lim para cualquier sucesión lim hn  0 . Estos límites son definidas según la def. 1.2. Se
n  hn n

Página 55 de 114
tiene cos(hn )  cos(0)  1 porque el cos es continua en cero, como hemos demostrado en el
n 

ejemplo 2.) después de las definiciones 2.10a y b, y para hn  (   2 ,  2 ) vale


cos( hn )  cos( hn ) , tal que también el valor absoluto del cos es continua en cero. Alternativamente
se puede argumentar que la función valor absoluto es continua en todo su argumento, también en
cero, y el cos es continua en todo su argumento, también en cero, entonces según el teorema 2.9c
la función compuesta cos(hn ) debe ser continua en todo argumento, también en cero.
sen (hn )
Además se tiene según 2.14b.(4) cos(hn )   1  hn  (   2 ,  2 ) . Adicionalmente
hn
aplicamos el teorema 1.3. La sucesión hn  0 no debe estar en el intervalo (   2 ,  2 ) para
n 

todo índice n natural, es suficiente si vale esto  n  N1 con algún número positivo finito N1 ,
porque esto es la hipótesis del teorema 1.3. Y esta hipótesis la cumple cualquier sucesión hn  0
n 
: Los primeros valores de la sucesión pueden estar en cualquier intervalo, los últimos, para todo n
mayor que un número finito N1 , todos satisfacen la condición hn  (   2 ,  2 ) , porque a causa
de la convergencia a cero siempre se puede encontrar un N1 para lo cual vale la condición.
Entonces se puede aplicar el teorema 1.3 para todas las sucesiones hn  0 . Porque cos(hn )  1
n  n 

sen (hn )
por la izquierda y 1  1 por la derecha la sucesión encerrada según el teorema 1.3
n hn
sen (hn )
también debe converger a 1! Entonces resulta que  1 lo cual es equivalente a
hn n 

sen (hn ) sen (hn ) sen (hn )


 1 . Esto se puede ver facilmente porque para hn  0 se tiene  0 y
hn n   hn hn
sen (hn ) sen (hn ) sen (hn )
para hn  0 se tiene sen (hn )  0 y por eso  0 y de nuevo  . Por lo
hn hn hn
sen (hn )
tanto hemos desmostrado que  1 para cualquier sucesión hn  0 , porque no hemos
hn n   n 

aplicado ninguna condición o especialidad para esta sucesión, menos que debe converger a cero.
sen(h)
Todo esto es equivalente a lim  1 según la def. 2.7b. Q.e.d. Escribimos el resultado de
h0 h
toda la demostración como la ecuación 2.14b.(5):
sen(h)
lim 1 2.14b.(5)
h0 h
cos(h)  1
Con este resultado se puede calcular también el límite siguiente: lim :
h 0 h
lim
cos(h)  1
 lim
 cos(h)  1    cos(h)  1    lim
cos2 (h)  1
 lim
- sen 2 (h)
h 0 h h 0 h   cos(h)  1  h  0 h   cos( h)  1  h 0 h  1  cos ( h) 

Página 56 de 114
sen (h) - sen (h) 0
 lim  lim  1  0 . El primer límite demostramos en la ecuación 2.14b.(5)
h 0 h h  0 1  cos( h ) 11
y el segundo límite resulta según las reglas para funciones continuas (teorema 2.2-2.6) y porque
sen(x) y cos(x) son continuas en 0, como hemos demostrado en el ejemplo 2.) después de las
definiciones 2.10a y b. Entonces resula un segundo límite:
cos(h)  1 1 - cos(h)
lim  lim 0 2.14b.(6)
h 0 h h  0 h
Q.e.d.

Ahora podemos calcular la derivada de sen(x) con la ayuda de la definición 2.14:


sen( x0  h)  sen ( x0 ) sen ( x0 ) cos(h)  cos( x0 )sen(h)  sen( x0 )
sen ' ( x0 )  lim  lim  (ver
h 0 h h0 h
 cos(h)  1  sen (h) 
teorema 2.14a )  lim sen ( x0 )   lim cos( x0 ) . Aplicando las reglas para
h 0
 h  h  0
 h 
 cos(h)  1  sen (h) 
límites y el teorema 2.14b resulta: sen ' ( x0 )  sen ( x0 )  lim    cos( x0 )  lim 
h 0
 h  h 0
 h 
 sen ( x0 )  0  cos( x0 ) 1  cos( x0 ) para todo x0  R (ver y las ecuaciones 2.14b.(5,6))). Entonces
vale sen ' ( x )  cos( x )  x  R .

Calculamos la derivada de cos(x):


cos( x0  h)  cos( x0 ) cos( x0 ) cos(h)  sen ( x0 )sen(h)  cos( x0 )
cos' ( x0 )  lim  lim  (ver
h 0 h h  0 h
 cos(h)  1 sen (h) 
teorema 2.14a y b y las ecuaciones 2.14b.(5,6))  lim cos( x0 )  sen ( x0 )
h 0
 h h 
 cos( x0 )  0  sen( x0 ) 1  sen( x0 ) para todo x0  R . Entonces vale cos' ( x)  sen ( x )  x  R .

Calculamos la derivada de tan(x): tan' ( x)  sec2 ( x) . Demostración:



 sen ( x )  sen' ( x ) cos( x )  sen ( x ) cos' ( x ) cos 2 ( x )  sen 2 ( x ) 1
tan' ( x )     2
 2
 2
 sec 2 ( x )
 cos( x )  cos ( x) cos ( x) cos ( x )
(ver teorema 2.13 (4) ).

Teorema 2.15: regla de la cadena


La función g sea definida en el intervalo X g y la función f en el intervalo X f  g X g  .
Si la función g es diferenciable en x0 y la función f es diferenciable en g ( x0 ) , entonces la
función f  g es diferenciable en x0 , y además vale
( f  g )' ( x0 )  f ' ( g ( x0 ))  g ' ( x0 ) .

Página 57 de 114
Demostración: Por hipótesis g es diferenciable en x0 , entonces según el teorema 2.13.i se tiene
g ( x0  h)  g ( x0 )  g ' ( x0 )  h  r1 (h) (i) con una función de resta r1 (h) con la propiedad
r ( h)
lim 1  0 . Se define y0 : g ( x0 ) . Por hipótesis f es diferenciable en g ( x0 ) y se encuentra según
h0 h

el teorema 2.13.i f ( y0  k )  f ( y0 )  f ' ( y0 )  k  r2 (k ) (ii) con una función de resta r2 (k ) con la


r (k )
propiedad lim 2  0.
k 0 k
Entonces se tiene ( f  g )( x0  h)  ( f  g )( x0 )  f  g ( x0  h)   f  g ( x0 )  (comparar def. 2.4), y
según (i) se sigue ( f  g )( x0  h)  ( f  g )( x0 )  f  g ( x0 )  g ' ( x0 )  h  r1 (h)   f g ( x0 ) . Con la
definición k (h) : g ' ( x0 )  h  r1 (h) y con (ii) se sigue ( f  g )( x0  h)  ( f  g )( x0 ) 
 f  g ( x0 )   f '  g ( x0 )    g ' ( x0 )  h  r1 (h)   r2  g ' ( x0 )  h  r1 (h)   f  g ( x0 ) 
 f '  g ( x0 )    g ' ( x0 )  h  r1 (h)   r2  g ' ( x0 )  h  r1 (h) 
 f '  g ( x0 )   g ' ( x0 )  h  f '  g ( x0 )   r1 (h)  r2  g ' ( x0 )  h  r1 (h)  . El segundo termino se puede
definir como r (h) : f '  g ( x0 )   r1 (h)  r2  g ' ( x0 )  h  r1 (h)  , tal que se obtiene
( f  g )( x0  h)  ( f  g )( x0 )  f '  g ( x0 )   g ' ( x0 )  h  r (h) .
( f  g )( x0  h)  ( f  g )( x0 ) f '  g ( x0 )   g ' ( x0 )  h  r (h)
Finalmente se puede escribir lim  lim
h 0 h h0 h
r ( h) r (h)
 f '  g ( x0 )   g ' ( x0 )  lim . Es decir ( f  g ) ' ( x0 )  f '  g ( x0 )   g ' ( x0 )  lim .
h0 h h0 h

r ( h)
Falta solamente demostrar que vale lim  0:
h 0 h

r ( h) r (h) r2  g ' ( x0 )  h  r1 (h)  r ( h)


 f '  g ( x0 )   1  , y con la hipótesis lim 1  0 resulta
h h h h0 h

r ( h) r ( h) r  g ' ( x0 )  h  r1 (h)  r  g ' ( x0 )  h  r1 (h) 


lim  f ' g ( x0 )   lim 1  lim 2  lim 2 . Primero
h 0 h h 0 h h0 h h  0 h
r ( h)
calculamos el limite lim k (h)  lim  g ' ( x0 )  h  r1 (h)   0 porque por hipótesis vale lim 1 0
h 0 h0 h0 h

  r ( h)  
y con lim h  0 resulta con las reglas para límites que vale lim r1 (h)  lim   1   h   0 .
h0 h0 h0
 h  
Entonces hemos demostrado que vale: Si h  0 entonces k ( h)  0 o simplemente k  0 . Con
todo esto se puede demostrar lo siguiente:
r ( h) r  g ' ( x0 )  h  r1 (h)  r  k ( h)   r (k )   k 
lim  lim 2  lim 2  lim  2    
h 0 h h 0 h h  0 h h  0
 k   h 
 r (k )   k
  lim 2    lim  .
 h 0 k   h 0 h 
k g ' ( x0 )  h  r1 (h) r ( h) r (k )
lim  lim  g ' ( x0 )  lim 1  g ' ( x0 ) . Vale por hipótesis lim 2  0.
h 0 h h 0 h h  0 h k  0 k

Página 58 de 114
r ( h)  r (k )  r (k )
Por eso resulta lim   lim 2   g ' ( x0 )  g ' ( x0 )  lim 2  g ' ( x0 )  0  0 . Q.e.d.
h 0 h  h 0 k  k 0 k

Ejemplo 1: f ( x)  sen ( x ) y g ( x)  x 2  3 . Entonces la función compuesta f  g (ver definición


2.4) está definida por ( f  g )( x)  f  g ( x)   sen ( x 2  3) . f ' ( x)  sen ' ( x )  cos( x ) . En
consecuencia f  g ( x)   cos g ( x)   cos( x 2  3) . Puesto que g ( x)  x 2  3, g ' ( x)  2 x . Por tanto
 
se obtiene ( f  g )' ( x)  f g ( x)   g ' ( x)  cos( x 2  3)  2 x  2 x cos( x 2  3) .

5
 2  2
Ejemplo 2: h( x)    . f ( x)  x , g ( x) 
5
. Y además podemos definir:
 x 1 x 1
2
g ( x)  k l ( x)  , l ( x)  x  1, k ( x)  . Primero calculamos
x
1 2
g ' ( x)  k l ( x)   l ' ( x)  2  1   .
x  12
x  12
Además vale: f ' ( x)  5 x 4 . Entonces podemos calcular la derivada de h(x):
4
 2  2  160
h' ( x)  f  g ( x)   g ' ( x)  5    .
 x 1 x  1 ( x  1)6
2

Teorema 2.16: Diferenciación de una función potencia (con exponentes racionales)


Si f es una función potencia definida por f ( x)  x r , donde r es cualquier número racional (ver
subcapítulo 3.1), entonces f es diferenciable y f ' ( x)  rx r 1 x  R0 , r  Q ( x  0 si r  0 ).
La misma regla también vale para cualquier exponente r de los números reales:
d r
x  r  x r 1 , x  R0 , r  R ( x  0 si r  0 ).
dx
Demostración:
d n
Primero genealizamos el resultado de la derivada x  n  x n 1 , x  R , n  N (arriba del
dx
d z
teorema 2.12), es decir x  z  x z 1 , x  R ( x  0 si z  0) , z  Z , de la manera siguiente: Para
dx
cualquier exponente entero negativo z   n , n  N , podemos aplicar la regla del cociente (ver el
d  n d 1 0  x n  1  n  x n 1
teorema 2.13) y la regla ya demostrada para n  N : x  
dx dx x n x 2n
  n  x n 1 2 n   n  x  n 1  x  R , x  0 ,  n  N . Entonces vale la regla para cualquier número
entero negativo z   n , n  N . Para todos los números enteros positivos (los números naturales)
d 0 1
ya hemos demostrado la regla. Para z  0 vale x  0  x 0 1  0   0 ,  x  R , x  0 . El
dx x

Página 59 de 114
d 0 d
resultado es correcto porque x  1 0. Entonces vale la regla
dx dx
d z
x  z  x z 1 , x  R , z  Z ( x  0 si z  0 ). Q.e.d.
dx
d r
Ahora hay que demostrar que vale x  r  x r 1 , x  R0 , r  Q ( x  0 si r  0 ): Cada r  Q
dx
p
se puede escribir como r  , p, q  Z , q  0 . Una potencia con un exponente racional se puede
q
definir en dos pasos (ver subcapítulo 3.1) (ver “Ordnungsrelationen [13]ff, H. Heuser, Lehrbuch der Analysis
Bd.1, S. 77): Primero se define a
1q
como la solución única  de la ecuación x q  a para cualquier
a  0 ( a  R0 ) para todo q  N . Se puede demostrar que existe una solución única
  sup  y  R y  0  y q  a  con   0 , es decir vale  q  a . Se define entonces:
 : a 1 q  a como la q-esima raiz de a. Esto es al mismo tiempo la definición de la función
q

inversa (ver la def. 2.27) f 1 ( x)  x1 q de la función f ( x)  x q para x  0 : Según esta definición


se puede escribir x q  a  x  a 1 q . Por eso vale a  x q  a 1 q   q
 a  a  0  q  N y además
se puede concluir x  a  ( x )  x  x  0 ,  q  N . Por lo tanto se puede escribir
1q q 1q

a   a 
1q q q 1q
 a q q  a  a  0  q  N , tal que la operación inversa al exponente 1 q es el
exponente q y la operación inversa al exponente q es el exponente 1 q .
p
En el segundo paso se puede definir a r con r  , p, q  Z , q  0 , es decir para cualquier r  Q
q
positivo y para cualquier a  R0 de la manera siguiente: (i) a r : a p   1q
 p, q  N , a  0 , es
p
decir para r   0 . Se tiene: a  0  a p  0  p  N y porque a  0  a 1 q  0  q  N se
q
sigue a 0 a p   1q
 0  p, q  N ; vale  
0r  0 p
1q
 0  p, q  N . Además se define
p
 1 p p
(ii) a  r  a q
:  p, q  N , a  0 , es decir para r   0 y  r    0 . Además se
a 
p 1q q q
define (iii) a : 1 para cualquier a  0 . Con estas tres definiciones (i), (ii) y (iii) hemos definido
0

a r para cualquier a  0 y para cualquier r  Q . Se define a0 : 1 , a  0 . Con la definición final


(iv) 0 r : 0 para r  Q , r  0 (lo que también es un resultado de las definiciones anteriores
porque 0 p  0  p  N y 0 1 q  0  q  N y por eso se sigue 0 r : 0 p  1q
 0 para r  Q , r  0
) hemos definido a r para cualquier a  R0 para todo r  Q ( a  0 si r  0 ), a r  R0 .
Para esta definición de una potencia con exponentes racionales valen las mismas leyes como para
exponentes naturales: Para todos los número reales a, b  0 y para cualquier exponente racional
r , s  Q vale: 1.) a r a s  a r  s 2.) a r a s  a r  s (si r  s resulta ar ar  1  ar r  a0 , a  0 , que
  s
es consistente con la definición a : 1 , a  0 ) 3.) a r  a r  s 4.) a r b r  (ab) r 5.) a r b r  (a b) r
0

. Se puede demostrar por ejemplo la regla 3.) con la ayuda de la definición de un exponente

Página 60 de 114
racional: a r : a p   1q
 p, q  N , r 
p
q
 0 . La regla 3.) vale para exponentes  p, q  N :

(I).
Hay que demostrar que vale la regla también para exponentes como 1 p ,1 q  p, q  N : Según la
definición original vale: a 1 q es definida como la solución única  de la ecuación x q  a
( a 1 q :  ) para cualquier a  0 ( a  R0 ) para todo q  N , aquí la hipótesis es a  0 . Esto vamos
a escribir de la forma x q  a  x  a 1 q . Asi se puede escribir x q   p
b ( x 0b0 )
 x q  b 1 p y además  x  b   1 p 1q
. Porque para exponentes naturales ya vale x q   p
 b  xp   q

(ver (I)) se puede deducir de la misma manera x  b 1 q   1 p


. También vale: x p  q  x q  p  b
1 1
p q q p
 xb b . Por lo tanto con las reglas para exponentes naturales
1 1 1 1

x   x     
p  
p q 1q 1 p
q
 x p q  x q  p  b (ver (I)) resulta que x  b 1 p  b1 q b p q
b q p
b  0
1 1 1 1

   
1q   1 p
1 p ,1 q con p, q  N . Entonces hemos demostrado x 1 p x x  x1 q (II) p q q p

 x  0 1 p ,1 q con p, q  N .
Se puede demostrar que la regla también vale para casos mixtos p ,1 q  p, q  N : Queremos

demostrar que vale x1 q    x p p 1q


(x  0) x
1q
  
p q
 x p . Vale x1 q  
p q
 
 x1 q
pq
 
 x1 q
q p


 x1 q 
q p
  x   x p porque vale regla 3.) para exponentes naturales y se puede aplicar las
p

operaciones inversas (ver arriba). Entonces resulta x1 q    x  p p 1q


 x p  x p lo cual es correcto.

Con la definición (i) vale entonces (III)  x  0 ,  p ,1 q con p, q  N .


Entonces también vale regla 3.) para exponentes naturales y exponentes como valores recíprocos
p k
de números naturales. Finalmente podemos concñuir: Con r  y s  se puede escribir
q l

      
1l 1l 1 1 p k

x   x    x p     x p 
1q k k 1q 
r s p q k l pk
  (ver (III))  x q l
(ver (I) y (II))  x q l
(ver
   
p k

(III) con pk N y q l  N x q l
 x rs . Entonces ya hemos demostrado
x 
r s
 x r  s  r , s  Q , r , s  0 . De manera semejante se puede demostrar esto para
r , s  Q , r  s  0 según la def. (ii) y despúes para cualquier caso mixto con los signos. Todo esto
ya dejamos al lector.

Ahora aplicamos estas definiciones (i)-(iv) a la expresión x r  0 para cualquier x  R0 para todo
p
r  Q ( x  0 si r  0 ), r  , p, q  Z , q  0 , y derivar esta función:
q

Página 61 de 114
p
d  q 
p 1
p d r d q
1.) Empezamos con el caso r   0 , p, q  N . Tenemos x  x  x ( ver (III) )
q dx dx dx  
p 1
 1q  d
1

 p x   x q .según la regla de la cadena (teorema 2.15) y la regla de la potencia con


  dx
 
1
d q
exponentes naturales. Hay que calcular ahora x .
dx
La unicidad de la solución del problema x  y  0 es equivalente con la existencia de una función
q

1
inversa de la función f ( x)  x , q  N : y  f ( x)  x  x  f ( y)  y , q  N . La función
q q 1q

inversa existe en este caso según el teorema 2.29a. f ( x)  x , q  N es contiuna para todo x (ver
q

el teorema 2.9a), también en R0  [ 0 ,  ) . Entonces según el teorema 2.29a resulta que f tiene
1
una función inversa f ( x) definida en R0  [ 0 ,  ) que es estrictmante creciente y continua en
R0  [ 0 ,  ) . Además existe la derivada de esta función inversa según el teorema 2.30:
f ( x)  xq , q  N es continua y monótona en R0  [ 0 ,  ) . Además f es diferenciable para todo x

(ver la demostración de
d n
dx
'
 
x  x n  n  x n 1 , x  R , n  N arriba del teorema 2.12). f es

diferenciable y desigual a cero  x  R  ( 0 ,  ) . Entonces según el teorema 2.30 existe la
1 d 1 1
derivada de la función inversa f ( x)  x y vale:
1q
f ( x)  . Entonces se tiene:
dx d
dx
f
 f ( x)
1

 
x1 q ' 
1
.
q x
1 q q1

q 1

Ahora se necesita la regla 3.) a  


r s
a rs
 
1 q q 1
(ver arriba): x x q
 x11 q . Entonces resulta
1

dx
 
d 1q 1 1 1 1
x  11 q  x  (11 q )  x q .
qx q q
Alternativamente se puede calcular, aplicando las reglas para exponentes, de la manera siguiente:
 
d 1q q d
x  x  x  0  q x1 q  
q 1 d d
 x1 q  1   x1 q 
1

1 1
 x 1 q 1 .
dx dx dx dx q x1 q
q 1
qx 
11 q
q

Con este resultado resulta junto con la fórmula de la derivación de arriba resulta:
d r
dx
 
x  p x1 q
p 1 d

dx
x1 q  
 p x1 q
p 1 1

q
x 1 q 1
1
 p x p q 1 q x 1 q 1
q
(según regla 1.))

p p
 x p q 1 q 1 q 1  x p q 1  r x r 1 para cualquier x  R0 y para todo r  Q con r  0 .
q q

Página 62 de 114
p
2.) Ahora calculamos la derivada para el caso r  0 , p, q  N : Tenemos
q
d r d p q d 1
dx
x  x
dx

dx x p q
 
 (1) x p q
2 d

dx
p
x p q con  0 , p, q  N . Es decir, podemos aplicar
q
el resultado de 1.) para la derivada interior de esta regla de la cadena. Entonces resulta:
 (1)x p q 
d r d p q  2 p p q 1 p p
x  x  x  (1) x p q 1 x  2 p q  (1) x  p q 1
dx dx q q q
p
  x  p q 1  rx r 1 . Q.e.d.
q

Después en el subcapítulo 3.1 vamos a definir la función exponencial a x para cualquier a  R 


fijo pero arbitrario, definida para todo x  R con a x  0  x  R , estrictamente creciente si a  1
y estrictamente decreciente si 0  a  1 , continua para todo x. Para a  e  1 , e  2.718281828...
es un número irracional y se llama número de Euler, se tiene la función exponencial
e x  exp( x) : R  R  con su función inversa loga  e ( x)  ln( x) : R   R tal que
y  exp( x)  x  ln( y )  x  R ,  y  R  . Vale por eso y  expln( y )   y  R  y
x  ln exp( x)   x  R . Con la ayuda de la función exponencial y del logaritmo natural, que
resultan de toda la teoria de las funciones de potencia con exponentes racionales y finalmente con
d
exponentes reales, y con la derivada de la función exponencial exp( x)  exp( x)  x  R y con
dx
d
la derivada del logaritmo natural ln( x)  1 x  x  R  (ver más adelante(x)), se puede calcular
dx
la derivada
d r
dx
x , x  0 , r  R con r fijo pero arbitrario:
d r d
dx dx
 
x  exp ln( x r )  x  R  , esto

vale a causa de y  expln( y )   y  R y aquí se tiene x  0 . A causa de las propiedades de las


 r

funciones de logaritmo (entre otras: ln( x r )  r ln( x)  x  0 ,  r  R ) resulta que


d r d
dx
d
 
x  expr ln( x)   expr ln( x)   r  ln( x)  exp ln( x r )  r  1 x  x r  r  1 x  r  x r 1 . De
dx dx
esta manera ya hemos demostrado la regla más general de este tipo:
d r
x  r  x r 1 , x  R0 , r  R ( x  0 si r  0 ). Q.e.d.
dx

2
 
1
Ejemplo: f ( x)  4 3 x 2  4  x 2 3  4x 3 . Entonces vale
2  1
2  1  8  8 1 8
f ' ( x)  4  x 3 
 x 3   1  3 .
3 3 3 3 3 x
x

Teorema 2.17:
Si f y g son dos funciones tales que f ( x)  g ( x) , donde r es cualquier número real, y si g ' ( x )
r

Página 63 de 114
existe, entonces f es diferenciable, y
f ' ( x)  r g ( x)  g ' ( x) .
r 1

(Ver teorema 2.16 y 2.15 (regla de la cadena) .)

 
1
Ejemplo: f ( x)  4sen 2 ( x)  9 cos2 ( x) . Se escribe f ( x)  4sen 2 ( x)  9 cos 2 ( x) 2 y se aplica el

    
1
1 
teorema 2.17: f ' ( x )  4sen 2 ( x )  9 cos 2 ( x ) 2  4sen 2 ( x)  9 cos 2 ( x ) 
2
8sen ( x) cos( x)  18 cos( x) sen ( x)   10sen ( x) cos( x)
f ' ( x)   . Entonces encontramos
2 4sen 2 ( x)  9 cos 2 ( x) 2 4sen 2 ( x)  9 cos 2 ( x)
 5sen ( x) cos( x)
f ' ( x)  .
4sen 2 ( x)  9 cos 2 ( x)

2.5. VALORES MÁXIMOS Y MÍNIMOS DE FUNCIONES


Una aplicación importante de la derivada es determinar dónde una función alcanza sus valores
máximos y mínimos (extremos). Aqui estudiamos los extemos relativos, extremos absolutos y el
teorema del valor extremo.
Definición 2.16: Definición de valor máximo relativo:
La función f : I  R tiene un valor máximo relativo en el número c si existe una -vecindad
 
V (c) : x x  c    I que contiene a c, tal que f (c)  f ( x)  x V (c) .

Definición 2.17: Definición de valor mínimo relativo:


La función f : I  R tiene un valor mínimo relativo en el número c si existe una -vecindad
 
V (c) : x x  c    I que contiene a c, tal que f (c)  f ( x)  x V (c) .

Teorema 2.18:
Si f : [a, b]  R tiene un extremo relativo en c  ( a, b) , y además f ' (c ) existe, entonces
f ' (c )  0 .

Para la demostración necesitamos un corollario:


Corolario (2):
Dada la función f : I  R .

(i) Si lim f ( x) existe y es positivo, entonces existe una -vecindad V (c) : x x  c    I
x c

tal que f ( x)  0 x  U  (c), x  c .

Página 64 de 114

(ii) Si lim f ( x) existe y es negativo, entonces existe una -vecindad V (c) : x x  c    I
x c

tal que f ( x)  0 x  U  (c), x  c .
Demostración: Sea lim f ( x)  L , donde, por hipótesis, L>0. Al aplicar la definición de límite (ver
x c

L L
def. 2.7a) con   , tiene que existir un   0 tal que si 0  x  c   entonces f ( x)  L 
2 2
L L 3L
. 0  x  c    x  U  (c), x  c y f ( x)  L    f ( x)  .
2 2 2
L 3L
Entonces vale: Si x  U  (c), x  c entonces  f ( x)  . Como L>0, esta proposición significa
2 2
que f ( x)  0 x  U  (c), x  c .
La demostración del inciso (ii) es semejante a la del inciso (i): Sea lim f ( x)  L , donde, por
x c

L
hipótesis, L<0. Al aplicar la definición de límite con    , tiene que existir un   0 tal que si
2
L L L L L
0  x  c   entonces f ( x)  L   ,   0. f ( x)  L    L   f ( x)  L 
2 2 2 2 2
3L L
  f ( x)   0 . Es decir para todo x con 0  x  c   resulta que f ( x )  0 . Entonces
2 2
f ( x )  0  x  U  (c), x  c . Q.e.d.

Demostración del teorema 2.18: Suponga que f ' (c)  0 . Entonces f ' (c)  0 o f ' (c)  0 . Si
f ( x )  f (c)
f ' (c )  0 entonces lim  0 . Por tanto, por el corolario arriba existe una -vecindad
x c xc

V (c) : x x  c    tal que
f ( x )  f (c )
xc
 0  x  V (c) , x  c . Se concluye que si

x  V (c) , x  c entonces f ( x )  f (c ) y x  c tienen el mismo signo; esto es


f ( x )  f (c ) si x  c y f ( x )  f (c ) si x  c . Por eso f no puede tener un valor máximo relativo
ni un valor mínimo relativo en c (ver definiciones 2.16 y 2.17), lo cual contradice la hipótesis de
que f tiene un extremo relativo en c.
f ( x )  f (c )
Si f ' (c )  0 , se obtiene una contradicción semejante: lim  0 por hipótesis y según
xc xc
el corolario existe una -vecindad 
V (c) : x x  c    tal que
f ( x )  f (c )
 0  x  V (c) , x  c . Se concluye que si x  V (c) , x  c entonces f ( x )  f (c ) y
xc
x  c tienen un signo opuesto; esto es f ( x )  f (c ) si x  c y f ( x )  f (c ) si x  c . Por eso f
no puede tener un valor máximo relativo ni un valor mínimo relativo en c lo cual contradice la
hipótesis de que f tiene un extremo relativo en c.
Así, la suposición de que f ' (c )  0 conduce a una contradicción, por tanto, f ' (c )  0 . q.e.d.

Página 65 de 114
Ejemplo: f ( x)  x 4  4 x 3  2 x 2  12 x . Como f es un polinomio, f’(x) existe en todo número. Por
tanto, los unicos números críticos son aquellos valores de x para los que f’(x)=0, esto es, las
coordenadas x de los puntos de la gráfica de f para los que la recta tangente es horizontal. Se calcula
f’(x), se iguala a cero y se despeja xi, los puntos críticos:
3 2 3 2 2
f ' ( xi )  4 xi  12 xi  4 xi  12  0  xi  3 xi  xi  3  0  xi ( xi  3)  ( xi  3)  0
2 2
 ( xi  3)( xi  1)  0  x1  3 y  x2,3  1  x2  1 , x3  1 .

Definición 2.18: Valor máximo absoluto en un intervalo:


La función f : I  R tiene un valor máximo absoluto en un intervalo I si existe algún número
c  I tal que f (c)  f ( x ) x  I . El número f(c) es el valor máximo absoluto de f en el
intervalo I: f (c)  max f ( x) .
xI

Definición 2.19: Valor mínimo absoluto en un intervalo:


La función f : I  R tiene un valor mínimo absoluto en un intervalo I si existe algún número
c  I tal que f (c)  f ( x) x  I . El número f(c) es el valor mínimo absoluto de f en el
intervalo I: f (c)  min f ( x) .
xI

Teorema 2.19: Teorema del valor extremo:


Si la función f : [ a, b]  R es continua en el intervalo cerrado [ a, b] , entonces f tiene un valor
máximo absoluto y un valor mínimo absoluto en [ a, b] .
Sin demostración, solo unas sugerencias: [a, b]  R es un conjunto acotado de los números reales.
El conjunto de todos los valores f ([a, b])  R también es un conjunto acotado si la función f es
continua. Segun el axioma 1.1 existe un infimo y un supremo del conjunto f ([a, b])  R . Con
eso, con la continuidad de la función f y con el teorema del valor intermedio, teorema 2.10, se
puede demostrar que existe un c1  [a, b] tal que f (c1 )  f ( x) x  [a, b] y un c2  [a, b] tal que
f (c2 )  f ( x ) x  [a, b] .

2.5.1. LA DIFERENCIACIÓN (DERIVACIÓN) IMPLÍCITA

 
Si f  ( x, y ) | y  3x 2  5 x  1 entonces la ecuación y  3 x 2  5 x  1 define a la función
explícitamente. Sin embargo, no todas las funciones pueden ser definidas explícitamente mediante
una ecuación. Por ejemplo, no se puede resolver la ecuación x 6  2 x  3 y 6  y 5  y 2 para y en
terminos de x. No obstante, pueden existir una o más funciones tales que la ecuación
x 6  2 x  3 f ( x)   f ( x)   f ( x) se cumple para todos los valores de x en el dominio de f. En
6 5 2

este caso, la función f está definida implícitamente por la ecuación dada. Con la suposición de que

Página 66 de 114
esta ecuación define a y como al menos una función diferenciable de x, la derivada de y con
respecto de x puede determinarse mediante la diferenciación implícita. Sea F la función definida
por el miembro izquierdo y G la función definida por el miembro derecho. Así, F ( x)  x 6  2 x y
G ( y )  3 y 6  y 5  y 2 donde y es una función de x, por decir y  f (x ) . De este modo
podemos escribir F ( x)  G  f ( x)  (por todo x del dominio de f para los cuales G  f (x)  existe).

Entonces, para todo x para los cuales f es diferenciable, se tiene


d 6
dx

x  2x  
d
dx

3 y6  y5  y2 . 

dx

d 6

x  2 x  6 x5  2 y
d
dx
  dy dy
3 y 6  y 5  y 2  18 y 5   5 y 4   2 y 
dx dx
dy
dx
. Entonces se obtiene


6 x 5  2  18 y 5  5 y 4  2 y  
dy
dx

dy

6 x5  2
dx 18 y 5  5 y 4  2 y
.

Ejemplo ilustrativo: Utilizamos la diferenciación implícita para determinar la pendiente de la


dy dy x2
recta tangente a la curva x 3  y 3  9 en el punto (1,2). 3x 2  3 y 2 0   2 . En el punto
dx dx y
dy 1
(1,2) vale  . Una ecuación de la recta tangente es
dx 4
1 1
y  2    x  1  x  4 y  9  0  y  9  x  . Asi encontramos la recta tangente
4 4
 
1
1
y  9  x  a la gráfica de la función y  9  x 3 3 en el punto (1,2). En este caso se puede hacer
4
la prueba, porque aquí se puede despejar la ecuación con respecto a y: x 3  y 3  9

  para los x para los


1
 y  9  x3 3 cuales vale 9  x 3  0 y con y  0 . Se sigue:
2 2

  9  x   3 x  . Resulta que  9  1   3  3  1 8 3  3  8 3  2 2   1


dy 1 
2
dy 1 2  
3 3 2

dx 3 dx x 1 3 3 4
, lo que es el mismo resultado que arriba.

2.6. TEOREMA DE ROLLE Y TEOREMA DEL VALOR MEDIO


Teorema 2.20a: Teorema de Rolle:
Sea f : [ a, b]  R una función tal que
(i) es continua en el intervalo cerrado [ a, b] ,
(ii) es diferenciable en el intervalo abierto ( a, b) ,
(iii) f ( a )  f (b)  0 .
Entonces existe un número c  (a, b) tal que f ' (c )  0 .
Demostración: Se considerarán dos casos:
Caso I: f ( x)  0 x  [a, b] . Entonces f ' ( x)  0 x  (a, b) . Por tanto culaquier número c  (a, b)
puede considerarse como c con f ' (c )  0 .

Página 67 de 114
Caso II: El complemento de caso I:  ci  (a, b) tal que f (ci )  0 (existe al menos un ci,
posiblemente existen más puntos en el intervalo ( a, b) . Como f es continua en [ a, b] , entonces,
por el teorema del valor extremo (ver teorema 2.19) f tiene un valor máximo absoluto en [ a, b]
y un valor mínimo absoluto en [ a, b] . De (iii) vale f ( a )  f (b)  0 y además  ci  (a, b) tal que
f (ci )  0 (i=1 o más grande). En consecuencia, f tendrá un valor máximo absoluto positivo en
c1  (a, b) , se dice en el intervalo abierto, o un valor mínimo absoluto negativo en c2  (a, b) , o
ambos. Así existe un extremo absoluto en un punto interior c  c1 o c  c2 (o más puntos) del
intervalo [ a, b] . Por tanto, el extremo absoluto f (c ) es también un extremo relativo (porque
 
c  (a, b) y por eso existe un -vecindad V (c) : x x  c    (a, b) , ver las definiciones 2.16
y 2.17) y como f ' (c ) existe por hipótesis (ii), se deduce por el teorema 2.18 que vale f ' (c )  0 .
Q.e.d.

El caso más general (teorema 2.20b) se puede reducir a teorema 2.20a:


Teorema 2.20b: Teorema de Rolle:
Sea f : [ a, b]  R una función tal que
(i) es continua en el intervalo cerrado [ a, b] ,
(ii) es diferenciable en el intervalo abierto ( a, b) ,
(iii) f ( a )  f (b) .
Entonces existe un número c  (a, b) tal que f ' (c )  0 .
Si vale f ( a)  f (b)  k  R con k  0 se puede definir una nueva función g ( x ) : f ( x )  k .
Porque f es continua en el intervalo cerrado [ a, b] y diferenciable en el intervalo abierto ( a, b) ,
también g ( x ) : f ( x )  k tiene esas propriedades. Además vale
g (a )  f ( a )  k  k  k  0  g (b) . Asi se cumplen las tres hipótesis del teorema 2.20a y se
concluye que existe un número c  ( a, b) tal que g ' (c )  0 . Porque g ' ( x) : ( f ( x )  k )'  f ' ( x )
vale también f ' (c )  0 . Asi demostramos el teorema 2.20b con la ayuda del teorema 2.20a.

Teorema 2.21: Teorema del valor medio (para derivadas):


Sea f : [ a, b]  R una función tal que
(i) es continua en el intervalo cerrado [ a, b] ,
(ii) es diferenciable en el intervalo abierto ( a, b) ,
f (b)  f (a)
Entonces existe un número c  (a, b) tal que f ' (c)  .
ba
f (b)  f (a)
Interpretación geométrica: Para la gráfica de la función f, es la pendiente del
ba
segmento de recta que une los puntos A  ( a, f (a )) y B  (b, f (b)) . El teorema del valor medio

Página 68 de 114
establece que existe algún punto de la gráfica C  (c, f (c )) entre A y B en el que la recta tangente
f (b)  f (a)
el paralela a la recta secante que pasa por A y B; esto es, existe f ' (c)  .
ba
El teorema del valor medio es una generalización del teorema de Rolle.
Demostración: Una ecuación de la recta que pasa por los puntos A y B (ver arriba) es
f (b)  f (a ) f (b)  f (a)
y ( x  a)  f (a) . Definimos F ( x)  f ( x)  ( x  a)  f (a) . F(x) mide
ba ba
la distancia vertical entre el punto ( x, f ( x )) de la gráfica de la función f y el puinto correspondiente
de la recta secante que pasa por A y B. F satisface las tres condiciones de la hipótesis del teorema
de Rolle: (i) F es continua en el intervalo cerrado [ a, b] , porque es la suma de f (que es continua
por hipótesis) y una función lineal. (ii) F es diferenciable en el intervalo abierto ( a, b) , porque
f (b)  f (a)
F ' ( x)  f ' ( x)  y f es diferenciable por hipótesis.
ba
f (b)  f (a)
F (a)  f (a)  (a  a)  f (a)  f (a)  f (a)  0 y
ba
f (b)  f (a)
F (b)  f (b)  (b  a)  f (a)  f (b)  ( f (b)  f (a))  f (a)  0 . Por tanto, F
ba
satisface la condición (iii) del teorema de Rolle.
Por eso podemos concluir que existe un número c  ( a, b) tal que F ' (c )  0 . Entonces
f (b)  f (a) f (b)  f (a)
F ' (c )  f ' (c )   0  f ' (c )  . q.e.d.
ba ba

Definición 2.20: Una función creciente:


Una función f : I  R es creciente en un intervalo J  I si y sólo si
f ( x1 )  f ( x2 ) x1  x2 , x1 , x2  J .
La función es estrictamente creciente en un intervalo J  I si y sólo si
f ( x1 )  f ( x2 ) x1  x2 , x1 , x2  J .

Definición 2.21: Una función decreciente:


Una función f : I  R es decreciente en un intervalo J  I si y sólo si
f ( x1 )  f ( x2 ) x1  x2 , x1 , x2  J .
La función es estrictamente decreciente en un intervalo J  I si y sólo si
f ( x1 )  f ( x2 ) x1  x2 , x1 , x2  J .
Si una función es creciente o decreciente en un intervalo, entonces se dice que es monótona en ese
intervalo (ver definición 1.5).

Por ejemplo: f ( x)  x : Para cualquier par de argumentos x1, x2  0 con x1  x2 resulta que
2

x1  x1  x2  x1 multiplicando la desigualdad por x1 . Si multiplicamos por x2 se tiene x1  x2  x2  x2


. A causa de la transitividad resulta que x1  x1  x2  x2 . Por lo tanto hemos demostrado: Para
2 2

Página 69 de 114
cualquier par de argumentos x1, x2  0 con x1  x2 se sigue x1  x2  f ( x1)  f ( x2 ) . Es decir,
2 2

según la def. 2.20 una función parábola f ( x)  x es estrictamente creciente en J  R  . Si


2

permitimos que x1  0 , entonces resulta que f es estrictamente creciente en J  R0  [ 0 ,  ) .


Ahora elegimos cualquier par de argumentos x1 , x2  0 con x1  x2 : Multiplicando por x1  0
resulta x1  x1  x2  x1 . Multiplicando por x2  0 resulta x1  x2  x2  x2 . Con la transitividad resulta
que x1  x2 . Según la def. 2.21 resulta entonces que una función parábola f ( x)  x es
2 2 2

estrictamente decreciente en J  R  (o si permitimos x2  0 en J  R0  ( , 0 ] ). (Ver también


el teorema 3.1.)

Teorema 2.22:
Sea f : [ a, b]  R una función continua en el intervalo cerrado [ a, b] y diferenciable en el
intervalo abierto ( a, b) :
(i) Si f ' ( x )  0 x  (a, b) , entonces f es creciente en [ a, b] .
(ii) Si f ' ( x )  0 x  (a, b) , entonces f es estrictamente creciente en [ a, b] .
(iii) Si f ' ( x)  0 x  (a, b) , entonces f es decreciente en [ a, b] .
(iv) Si f ' ( x )  0 x  (a, b) , entonces f es estrictamente decreciente en [ a, b] .
Demostración: Caso (ii): x1  x2 , x1 , x2  [a, b] . Entonces f es continua en [ x1 , x2 ] y diferenciable
en ( x1 , x2 ) , segun el teorema del valor medio existe un número c  ( x1 , x2 ) tal que
f ( x2 )  f ( x1 )
f ' (c )  . x1  x2  x2  x1  0 . f ' ( x)  0 por hipótesis, por tanto
x2  x1
f ( x2 )  f ( x1 )  0  f ( x2 )  f ( x1 ) . Porque x1 , x2  [a, b] son números cualesquiera del intervalo
[ a, b] , f es estrictamente creciente en [ a, b] .
La demostración del inciso (i), (iii) y (iv) es semejante a la del inciso (ii).
Pero se puede demostrar este teorema alternativamente sin aplicar el teorema del valor medio
aplicando el corolario (2) abajo del teorema 2.18: Por hipótesis vale f ' ( x)  0 x  (a, b) , o
f ( x0  h)  f ( x0 ) f ( x)  f ( x0 )
f ' ( x0 )  0  x0  (a, b) , es decir lim  lim  0  x0  (a, b) .
h 0 h x  x0 x  x0
f ( x)  f ( x0 )
Según el corolario (2) existe una -vecindad V ( x0 ) tal que  0  x  V ( x0 ) , x  x0
x  x0
. El signo del númerador debe ser el mismo que el del denominador: Para x  x0  0 resulta que
f ( x)  f ( x0 )  0 y para x  x0  0 resulta que f ( x)  f ( x0 )  0 . Entonces se tiene para todo
x  x0  f ( x)  f ( x0 )  x  V ( x0 ) , x  x0 y  x0  (a, b) y para todo x  x0  f ( x)  f ( x0 )
 x  V ( x0 ) , x  x0 y  x0  (a, b) . Se puede expresar los enunciados arriba de la manera
siguiente: Para todo x2  x0  f ( x2 )  f ( x0 )  x2 V ( x0 ) , x2  x0 y para todo x1  x0
 f ( x1 )  f ( x0 )  x1 V ( x0 ) , x1  x0 , todo esto  x0  (a, b) . Es decir: Para todo x1  x0  x2
 f ( x1 )  f ( x0 )  f ( x2 )  x1 , x2 V ( x0 ) , x1  x2 , todo esto  x0  (a, b) . Según los enunciados
anteriores esto vale también para los casos x1  x0 y x0  x2 , entonces vale de verdad para todas

Página 70 de 114
las combinaciones posibles  x1 , x2 V ( x0 ) , x1  x2 . Esto significa según la def. 2.20
(interpretado estrictamente), que f es estrictamente creciente en cada V ( x0 ) para todo x0  (a, b)
. Si se une todos estos intervalos en donde la función es estrictamente creciente, resulta que f es
estrictamente creciente en ( a, b) . Q.e.d.

Teorema 2.23: Criterio de la primera derivada para extremos relativos:


Sea f : (a, b)  R una función continua en (a, b) con c  (a, b) y existe f ' en (a, b) excepto
posiblemente en c:
(i) Si existen un 1  R  (c  1  a ) y un  2  R  (c   2  b) tal que f ' ( x)  0 x  (c  1 , c) y
f ' ( x)  0 x  (c, c   2 ) , entonces f tiene un valor máximo relativo en c.
(i) Si existen un 1  R  (c  1  a ) y un  2  R  (c   2  b) tal que f ' ( x)  0 x  (c  1 , c) y
f ' ( x)  0 x  (c, c   2 ) , entonces f tiene un valor mínimo relativo en c.
Demostración: Inciso (i): Segun el teorema 2.22 f es estrictamente creciente en [c  1 , c] , es decir
f ( x1 )  f (c) x1  [c  1 , c], x1  c ( x1  c) . Segun el teorema 2.22 f es estrictamente decreciente
en [c, c   2 ] , se dice f (c)  f ( x2 ) x2  [c, c   2 ], x2  c (c  x2 ) . Por lo tanto resulta que
f (c)  f ( x)  x V (c) con  : Min1 ,  2  , y según la definición 2.16 f tiene un valor máximo
relativo en c. La demostración del inciso (ii) es semejante a la del inciso (i). Q.e.d.

Para determinar analíticamente los extremos relativos de f:


1) Calcula f ' ( x) .
2) Determina los números críticos de f, es decir, los valores de xi para los cuales f ' ( xi )  0 .
3) Aplique el criterio de la primera derivada (teorema 2.23).

Página 71 de 114
2.7. DERIVADAS DE ORDEN SUPERIOR

Definición 2.22: Derivada del orden superior:


La función f : [a, b]  R es diferenciable en el intervalo ( a, b) , es decir, existe la función
f ': ( a , b )  R .
Sea la función f ': ( a, b)  R diferenciable en ( a, b) , podemos calcular la segunda derivada de f
d2 f d2 f d  df 
y la denotamos por f ' ' o por 2
: f ' ' : ( f ' )' o 2
:  
dx dx dx  dx 
d n 1 f
Si la función : (a, b)  R diferenciable en ( a, b) ( n  N , n  2 ), podemos calcular la n-
dx n 1
dn f ( n 1) dn f d  d n 1 f 
esima derivada de f y la denotamos por : f (n)
: ( f )' o: :  .
dx n dx n dx  dx n 1 
d0 f
La derivada cero está definida por 0
 f ( 0 ) : f .
dx

d n 1 f
Como la función f también la función : (a, b)  R es diferenciable en ( a, b) y por eso es
dx n 1
continua en (a, b) segun el teorema 2.11.
dk f
Si una función f es n veces diferenciable, su derivada ( k  N ,0  k  n ) es (n-k) veces
dx k
diferenciable.

Ejemplo: f ( x)  x m , m  N . Sea pN . En el caso pm encontramos:

   
p p
d d
p
x m  m  (m  1)  (m  2)    (m  ( p  1))  x m  p . En el caso p  m vale: p
x m  m! . Por
dx dx
 
p
d
tanto vale x m  0 si p  m .
dx p

Teorema 2.24: La generalización de la regla del producto (ver teorema 2.13):


Sean u y v funciones n veces diferenciables en el punto x0. Entonces vale la regla de Leipnitz:
dn n
 n  d n  p   dp 
n
u ( x )  v ( x )    
 

 n p
u ( x )   p v( x)  . (Ver los coeficientes binomiales y el
dx p  0  p  dx   dx 
teorema del binomio 2.12)

d3 3
 3  d 3 p   dp 
Ejemplo: 3
u ( x )  v ( x )    
 

 3 p
u ( x )   p v( x)  
dx p  0  p  dx   dx 
 3  ( 3)  3  3  3
 u  v   u ( 2)  v (1)   u (1)  v ( 2)   u  v (3)  u ( 3)  v  3u ( 2 )  v (1)  3u (1)  v ( 2 )  u  v (3) .
0  1  2  3
Página 72 de 114
Definición 2.23: Concavidad hacia arriba:
(c, f (c ))
Se dice que la gráfica de una función es cóncava hacia arriba en el punto si existen
f ' (c ) xc
y en un intervalo abierto I que contiene a c tal que para todos los valores de en I, el
( x, f ( x )) (c, f (c ))
punto de la gráfica está arriba de la recta tangente a la gráfica en .

Definición 2.24: Concavidad hacia abajo:


Se dice que la gráfica de una función es cóncava hacia abajo en el punto (c, f (c )) si existen
f ' (c ) y en un intervalo abierto I que contiene a c tal que para todos los valores de x  c en I, el
punto ( x, f ( x )) de la gráfica está debajo de la recta tangente a la gráfica en (c, f (c )) .

Teorema 2.25:
Sea f : [ a, b]  R una función diferenciable en el intervalo abierto ( a, b) con c  ( a, b) .
Entonces
(i) si f ' ' (c)  0 , la gráfica de f es cóncava hacia arriba en (c, f (c )) .
(ii) si f ' ' (c )  0 , la gráfica de f es cóncava hacia abajo en (c, f (c )) .
(Sin demostración)

Definición 2.25: Punto de inflexión:


El punto (c, f (c )) es un punto de inflexión de la gráfica de la función f si la gráfica tiene una
recta tangente en ese punto, y si existe un intervalo abierto I que contiene a c tal que si x está en
I, entonces
(i) f ' ' ( x)  0 si x  c y f ' ' ( x)  0 si x  c ; o
(ii) f ' ' ( x )  0 si x  c y f ' ' ( x )  0 si x  c

Teorema 2.26:
Sea f : [ a, b]  R una función diferenciable en el intervalo abierto ( a, b) con c  ( a, b) .
(c, f (c )) es un punto de inflexión de la gráfica de f. Entonces, si f ' ' (c ) existe, f ' ' (c )  0 .
Demostración: g ( x) : f ' ( x) . Entonces g ' ( x)  f ' ' ( x) . Como (c, f (c)) es un punto de inflexión
de la gráfica de la función f entonces f ' ' ( x ) cambia el signo en c, por lo que g ' ( x ) cambia el
signo en c. Segun el criterio de la primera derivada, g tiene un extremo relativo en c y segun el
teorema 2.18 g ' (c )  0  f ' ' (c )  0 (por que f ' ' (c) existe por hipótesis, también existe g ' (c) ).
q.e.d.

Teorema 2.27: Criterio de la segunda derivada para extremos relativos:


Sea f : [ a, b]  R una función 2 veces diferenciable en el intervalo abierto ( a, b) con c  ( a, b) .
Vale f ' (c )  0 .
(i) Si f ' ' (c)  0 , entonces f tiene un valor máximo relativo en c.
(ii) Si f ' ' (c )  0 , entonces f tiene un valor mínimo relativo en c.

Página 73 de 114
f ' ( x )  f ' (c )
Demostración: Inciso (i): Por hipótesis vale f ' ' (c)  lim  0 . Segun el corolario (2)
x c xc
con el teorema 2.18 existe una -vecindad V (c)  [ a , b ] tal que
f ' ( x )  f ' (c )
 0  x  V (c) , x  c . Sean I1 :  x  V (c) x  c  y I 2 :  x  V (c) x  c  .
xc
Entonces si x  I1  x  c  0 vale f ' ( x )  f ' (c )  0  f ' ( x )  f ' ( c ) porque
f ' ( x )  f ' (c )
 0  x  V (c) , x  c , entonces también para x  I1  V (c) . Si
xc
x  I 2  x  c  0 vale f ' ( x )  f ' (c )  0  f ' ( x )  f ' (c) .
Pero como f ' (c)  0 se concluye: x  I1  f ' ( x)  0 y x  I 2  f ' ( x)  0 . Por tanto, f ' ( x)
cambia de signo algebraico de positivo a negativo al pasar por c, conforme x crece. Por el criterio
de la primera derivada (teorema 2.23), f tiene un valor máximo relativo en c. La demostración del
inciso (ii) es semejante a la del inciso (i). Q.e.d.

2.8. INVERSA DE UNA FUNCIÓN

Definición 2.26: Función uno a uno (función inyectiva):


Una función es uno a uno (o inyectiva) si cada número de su contradominio Y f corresponde a
exactamente un número de su dominio X f :
x1  x2  f ( x1 )  f ( x2 ) x1 , x2  X f . Esto es equivalente a
f ( x1 )  f ( x2 )  x1  x2 x1 , x2  X f

Una función es uno a uno (o inyectiva) si y sólo si cada recta horizontal intersecta la gráfica de la
función a lo más en un punto.
Ejemplo: f ( x)  x 2 no es inyectiva, pero f ( x)  x 2 , x  0 sí es inyectiva.

Teorema 2.28:
Si una función es estrictamente monótona en un intervalo, entonces es inyectiva en este
intervalo.
Demostración: f es estrictamente monótona, entonces es estrictamente creciente o estrictamente
decreciente. Mostramos el caso que f es estrictamente creciente en el intervalo I: x1 , x2  I sean
dos números arbitrarios con x1  x2 , entonces vale x1  x2 o x1  x2 . Si x1  x2 vale segun la
definición 2.20 f ( x1 )  f ( x2 ) . Además vale: x1  x2  f ( x1 )  f ( x2 ) . En los dos casos vale
f ( x1 )  f ( x2 ) . Entonces la función es inyectiva en el intervalo I segun la definición 2.26. La
demostración es semejante si f es estrictamente decreciente en el intervalo. q.e.d.

Página 74 de 114
Definición 2.27: La inversa de una función:
f :A B
Si es una función uno a uno (inyectiva) considerada como el conjunto de pares
ordenados
( x, y )
, entonces existe una función
f 1 : B  A , llamada inversa de f, que es el
( y , x)
conjunto de pares ordenados definida por
1
f ( y ) : x si y sólo si y  f (x ) ( x  A ).

f 1 ( y ) es unica para cada valor y  B por que f es inyectiva (ver definición 2.26, especialmente
el enunciado f ( x1 )  f ( x2 )  x1  x2 x1 , x2  X f ). Asi se define para cada función f que es
inyectiva un unica función f 1 . Asi obtenemos una correspondencia A : f  A( f )  f 1 .
f 1 ( y )  x  f 1 ( f ( x))  x , x  A . También vale f ( x)  y  f ( f 1 ( y ))  y , y  B . Como el
símbolo empleado para la variable independiente es abitrario, se puede sustituir y por x para
obtener f ( f 1 ( x))  x , x  B . Asi se ve que si f 1 es la inversa de la función f , entonces f 1
es una función inyectiva y su inversa f 1 es f : f 1   1
 f . Eso significa para la correspondencia
1 1
A : f  A( f )  f , que A( f )  f  A( A( f ))  f ; A es una correspondencia involutoria.

La gráfica de la función f 1 es la gráfica de la función f reflejado a la recta bisectriz (la gráfica


de la función f ( x )  x ).

Teorema 2.29a: (Teorema de la función inversa):


Sea f : [ a , b ]  R una función continua y estrictamente creciente en el intervalo cerrado [a, b] .
Entonces
(i) f tiene una inversa f 1 definida en [ f ( a ), f (b )] ;
(ii) f 1 es estrictamente creciente en [ f ( a ), f (b )] ;
(iii) f 1 es continua en [ f ( a ), f (b )] .

Teorema 2.29b: (Teorema de la función inversa):


Sea f : [ a , b ]  R una función continua y estrictamente decreciente en el intervalo cerrado [ a , b ]
. Entonces
(i) f tiene una inversa f 1 definida en [ f (b), f (a)] ;
(ii) f 1 es estrictamente decreciente en [ f (b ), f ( a )] ;
(iii) f 1 es continua en [ f (b ), f ( a )] .
(La demostración se encuentra en las copias anexadas)

Teorema 2.30: Teorema de la derivada de la función inversa:


Sea f : [ a , b ]  R una función continua y monótona en el intervalo cerrado [a, b] y

Página 75 de 114
diferenciable en el intervalo abierto ( a , b ) con f ' ( x )  0 x  ( a , b ) .

Entonces la derivada de la función inversa f 1 existe y está dada por f 1 ( x )   



1
f ' f 1 ( x0 ) 
x  x0

Demostración: Por hipótesis f es diferenciable en el intervalo abierto ( a , b ) , con el teorema 2.13.i


se sigue entonces que se puede escribir f ( x0  h)  f ( x0 )  f ' ( x0 )  h  r (h)  x0  (a, b) con una
r ( h)
función de resta r (h) con la propiedad lim  0 . Si se define x : x0  h resulta h  x  x0 y
h 0 h

h  0  x  x0 . Con esto se tiene f ( x)  f ( x0 )  f ' ( x0 )  ( x  x0 )  r ( x  x0 )  x0  (a, b) y


f ( x)  f ( x0 ) r ( x  x0 )
entonces resulta  f ' ( x0 )   x  x0 ,  x0  (a, b) talque se puede
x  x0 x  x0
expresar la derivada de la manera siguiente:
f ( x)  f ( x0 ) r ( x  x0 ) r ( x  x0 )
f ' ( x0 )    x  x0 ,  x0  (a, b) con lim  0 (i).
x  x0 x  x0 x  x0 x  x0
.
Si denotamos f ( y )  x, f 1 ( x)  y con f ( y0 ) : x0 , f 1 ( x0 )  y0 , entonces la derivada de f como
función de y según (i) se puede expresar asi:
f ( y )  f ( y0 ) r ( y  y0 ) r ( y  y0 )
f ' ( y0 )    y  y0 ,  y0  (a, b) con lim  0 (ii).
y  y0 y  y0 y  y 0 y  y0
La derivada de la función inversa de f está definida por su cociente de diferencias estandar y con
la ecuación (ii) resulta que
1 1
f ( x)  f ( x0 ) y  y0 1 1
    y  y0 ,  y0  ( a, b)
x  x0 f ( y )  f ( y0 ) f ( y )  f ( y0 ) f ' ( y )  r ( y  y0 )
0
y  y0 y  y0
r ( y  y0 )
con lim  0.
y  y0 y  y0
Segun el teorema 2.29a,b la función f 1 es continua. Por eso vale lim f 1 ( x )  f 1 ( x0 ) , que
x  x0
1 1
significa que si converge x  x0 entonces converge f ( x)  f ( x0 ) o equivalentemente
y  y0 (porque f 1 ( x)  y y f 1 ( x0 ) : y0 ). Entonces se puede escribir
f 1 ( x)  f 1 ( x0 ) 1 1 1 1
lim  lim   
x  x0 x  x0 y  y0
f ' ( y0 ) 
r ( y  y0 )
f ' ( y0 )  lim
r ( y  y0 ) f ' ( y0 ) f ' f 1 ( x0 ) 
y  y0 y  y0 y  y0
. Q.e.d.

Página 76 de 114

 
Parece plausible el siguente: f  f ( x)   x  f  f ( x)  x'  1. Segunla regla de la cadena
1 1

  
   
f 1  f ( x)  f 1 ( f ( x))  f ' ( x) encontramos  
f 1 ( f ( x))  f ' ( x)  1

 
f 1 ( y ) 
1
f ' ( x)
.


Compara el resultado de la demostración arriba, que es equivalente a: f 1 ( x)    1
f '( y)
.

Otra posibilidad de mostrar esa regla de manera plausible es escribir según Leibniz:
 dx
,  f 1  
dy 1
f '  . Pero observa los argumentos diferentes en las formulas más exactas
dx dy dy
dx
arriba!

Ejemplo 1: f ( x)  x 2 , x  0 . y  x 2  x  y ( x  0) . f 1 ( y )  y .
 
 
f 1 ( y ) 
1

1

f ' ( x) 2 x 2 y
1

. Segun teorema 2.30 vale: f 1 ( x)  
1

1
f ' f ( x0 )


1
.
x  x0 2 x0

 12  1  12 1 1
Además la derivada de la función g ( x)  x es: g ' ( x)   x   x  1  (ver
  2 2 x
2x 2
teorema 2.16).

Ejemplo 2: arcsen( x)  1


. Vale sen 2 ( x)  cos2 ( x)  1 x  R
cosarcsen ( x) 
  
 cos( x)  1  sen 2 ( x) . En la region inyectiva de sen(x)  ,  el cos(x) no es negativo,
 2 2
  
entonces vale cos( x)  cos( x) x   ,  . Entonces encontramos cos( x)  1  sen 2 ( x) en
 2 2
el dominio de arcsen(x). Pero hay que excluir los punos x  1 y x  1 : Para x  1 resulta que
1 1
arcsen ( x  1)    2 tal que  no está definida. También para
cos arcsen ( x  1)  cos(  2)
1 1
x  1 resulta que arcsen( x  1)   2 tal que  no está definida. Por
cos arcsen ( x  1)  cos( 2)
d
eso arcsen( x) solamente está definida en el intervalo x  (1,1) . Eso lleva a
dx
arcsen ( x)   1

1
 x (1 ,1) (ver también la Tabla de integrales al
1 - sen arcsen ( x) 
2
1  x2
final del subcapítulo 2.11).

Página 77 de 114
2.9. ANTIDERIVACIÓN
Definición 2.28: Antiderivada:
Una función F se denomina antiderivada de la función f en un intervalo I si
F ' ( x )  f ( x ) x  I .

Ejemplo ilustrativo: F ( x) : 4 x 3  x 2  5  F ' ( x)  12 x 2  2 x . De modo que si f es la función


definida por f ( x)  12 x 2  2 x entonces f es la derivada de F, y F es la antiderivada de f. Si G es
la función definida por G ( x) : 4 x 3  x 2  17  G ' ( x)  12 x 2  2 x , entonces G también es una
antiderivada de f. En realidad, cualquier función determinada por
H ( x) : 4 x  x  C  H ' ( x)  12 x  2 x , donde C es una constante, es una antiderivada de f.
3 2 2

Teorema 2.31:
Si f y g son dos funciones diferenciables definidas en el intervalo I, tales que
f ' ( x )  g ' ( x ) x  I
entonces existe una constante K tal que f ( x )  g ( x )  K x  I .
Demostración: h( x ) : f ( x )  g ( x )  h' ( x )  f ' ( x )  g ' ( x ) (ver teorema 2.13). Por hipótesis vale
f ' ( x )  g ' ( x )  0  h' ( x )  0 x  I . Hay que demostrar que vale h( x )  K x  I . Suponemos
que h no es constante en I, entonces existen dos números diferentes x1 , x2  I , donde x1  x2 , tales
que h( x1 )  h( x2 ) . Por hipótesis h es diferenciable en I y por eso también es continua en [ x1 , x2 ] !
Por tanto, las hipótesis del teorema del valor medio se satisfacen, de modo que existe un número
h( x1 )  h( x2 )
c con x1  c  x2 , tal que h' (c)  (i) (ver teorema 2.21). Pero como
x1  x2
h' ( x)  0 x  [ x1 , x2 ]  h' (c)  0  (i ) h( x1 )  h( x2 ) que es una contradicción a h( x1 )  h( x2 )
que suponemos arriba. Por tanto si h es constante en I.
Entonces vale, si h' ( x )  0 x  I entonces existe una constante K tal que h( x )  K x  I .
Entonces vale h( x )  K  f ( x )  g ( x )  f ( x )  g ( x )  K x  I . q.e.d.

Teorema 2.32:
Si F es una antiderivada particular de f en un intervalo I, entonces cada antiderivada de f en I está
dada por
F ( x)  C (1)
donde C es una constante arbitraria, y todas las antiderivadas de f en I pueden obtenerse a partir
de (1) asignando valores particulares a C.
Demostración: Sea G cualquier antiderivada de f en I. Entonces G ' ( x )  f ( x ) x  I . Como F es
una antiderivada particular de f en I, F ' ( x )  f ( x ) x  I . Entonces vale G ' ( x )  F ' ( x ) x  I .
Por el teorema 2.31 existe una constante K tal que G ( x )  F ( x )  K x  I . Como G representa
cualquier antiderivada de f en I, toda antiderivada de f puede obtenerse a partir de F ( x )  C , donde
C es una constante arbitraria. q.e.d.
Página 78 de 114
La antiderivación o antidiferenciación es el proceso mediante el cual se determina el conjunto
de todas las antiderivadas de una función dada. El símbolo  denota la operación de
antiderivación, y se escribe  f ( x)dx  F ( x)  C (F(x)+C recibe el nombre antiderivada general
de f) donde F ' ( x)  f ( x ) y d ( F ( x ))  f ( x ) dx . Por eso se puede escribir  d ( F ( x))  F ( x)  C .
Por tanto, la antiderivación se considera como la operación para determinar el conjunto de todas
las funciones que tienen una derivada dada. Como la antiderivación es la operación inversa de la
derivación, los teoremas de antiderivación se obtienen de los teoremas de diferenciación.

Teorema 2.33:
 dx  x  C
 dx  1dx  x  C por que ( x  C )'  1 en cada intervalo I  R .

Teorema 2.34:
f y g sean dos funciones definidas en el intervalo I para las cuales existen las antiderivadas.
Entonces vale:
(i)  a f ( x) dx  a  f ( x)dx donde a  R es una constante arbitraria.
(ii)   f ( x)  g ( x)dx   fdx   gdx
Demostración del inciso (i): Si F ' ( x)  f ( x ) , entonces ( aF ( x ))'  a F ' ( x )  a f ( x ) segun
teorema 2.13. Entonces vale  a f ( x) dx  (a F ( x))  K . Además vale
a  f ( x)dx  a  F ( x)  C  porque F ' ( x)  f ( x ) . Entonces vale (aF ( x))  K  a  F ( x)  C  con
K  a C . Porque la antiderivada es única hasta una constante arbitraria (ver arriba la definición de
la antiderivación) vale  a f ( x) dx  a  f ( x)dx .
Inciso (ii): Si F ' ( x)  f ( x ) y G ' ( x)  g ( x) , entonces
( F ( x )  G ( x ))'  F ' ( x)  G ' ( x )  f ( x )  g ( x ) segun teorema 2.13. Entonces vale
  f ( x)  g ( x)dx  F ( x)  G( x)  C 1  F ( x)  K1  G ( x)  K 2   f ( x)dx   g ( x)dx con
C1  K1  K 2 . q.e.d.

Página 79 de 114
Combinamos teorema 2.34 (i) y (ii) podemos demostrar sucesivamente que vale lo siguiente:
Teorema 2.35:
Si existen las antiderivadas para toda f i ( x), i  1,2,, n , definidas en el mismo intervalo I,
entonces:
 n  n

 
i 1
ci  f i ( x ) 

dx  
i 1
ci  f i ( x)dx con ci  R constantes.

Teorema 2.36:
Si n es un número racional, entonces:
x n 1
   C , n  1
n
x dx
n 1
d  x n 1  n  1 n
Demostración:     x  x n segun teorema 2.16 (Ver también la Tabla de
dx  n  1  n  1
integrales al final del subcapítulo 2.11). Q.e.d.

Teorema 2.37:
(1)  sen ( x)dx   cos( x)  C
(2)  cos( x)dx  sen ( x)  C
(3)  sec 2 ( x)dx  tan( x)  C
(4)  csc 2 ( x)dx   cot( x)  C
(5)  sec( x)  tan( x)dx  sec( x)  C
(6)  csc( x)  cot( x)dx   csc( x)  C
(Ver también la Tabla de integrales al final del subcapítulo 2.11)
d
Demostración del inciso (3): tan( x)  sec2 ( x) (ver ejemplo arriba el teorema 2.15).
dx
d
Demostración del inciso (4): cot( x)   csc2 ( x) .
dx
Demostración del inciso (5):
d
sec( x)  d  1     sen2 ( x)  1  sen( x)  sec( x)  tan( x) .
dx dx  cos( x)  cos ( x) cos( x) cos( x)

Demostración del inciso (6):


d
 csc( x)   d   1   cos(2 x)  1  cos( x)  csc( x)  cot( x) .
dx dx  sen( x)  sen ( x) sen( x) sen( x)

Página 80 de 114
 3 sec( x) tan( x)  5 csc ( x)dx  3 sec( x) tan( x)dx  5 csc ( x)dx 
2 2
Ejemplo:
 3 sec( x)  5 cot( x)   K  3 sec( x)  5 cot( x)  K .

Teorema 2.38: Regla de la cadena para antiderivación:


Sea g : A  B una función diferenciable. f : B  C , F sea una antiderivada de f en B. Entonces
 f g ( x)  g ' ( x) dx F g ( x)   C .
Demostración: Por hipótesis vale F ' ( x)  f ( x ) y además F '  g ( x)   f  g ( x)  (i). Por la regla de
d
la cadena para diferenciación (teorema 2.15) vale: F g ( x)  F ' g ( x)  g ' ( x) (ii). Si se
dx
d
sustituye de (i) en esta eciación se obtiene F g ( x)  f g ( x)  g ' ( x) de cual se deduce que
dx
 f g ( x)   g ' ( x)dx  F g ( x)   C . q.e.d.
1
Ejemplo:  3 x  4dx   (3 x  4) 2 dx . Si g ( x)  3 x  4 entonces g ' ( x)dx  3dx . Por tanto, se
necesita un factor 3 junto a dx para obtener g ' ( x) dx . En consecuencia, se escribe
1 1 1
1 1
 (3x  4) 2 dx   (3x  4) 2  3  3dx  3  (3x  4) 2 3dx . Segun el teorema 2.38 vale
3
1 3
1 1 (3 x  4) 2 2
3  (3x  4) 2 3dx 
3 3
 C  (3x  4) 2  C .
9
2

Como un caso particular del teorema 2.38 y del teorema 2.36 se tiene la generalización de la
fórmula de la potencia para antiderivadas, la cual se establece a continuación.
Teorema 2.39:
Si g : A  B es una función diferenciable y n es un número racional, entonces
g ( x)  n 1

 g ( x)  g ' ( x)dx 


n
 C , n  1.
n 1
x (5  2 x 3 )8 dx ; g ( x) : 5  2 x 3  g ' ( x)dx  6 x 2 dx .
2
Ejemplo:
1 1 (5  2 x 3 )9 1
 x (5  2 x ) dx  6  (5  2 x ) 6 x dx  6  9  C  54  (5  2 x )  C .
2 3 8 3 8 2 3 9

d 1  1
Verificación:   (5  2 x 3 )9    9(5  2 x 3 )8  6 x 2  x 2 (5  2 x 3 )8 .
dx  54  54

Página 81 de 114
2.10.INTEGRAL DEFINIDA E INTEGRACIÓN

Definición 2.29: Integrabilidad de una función, integral definida de f de a a b:


f : [ a, b]  R acotada en [ a, b] .
Sea P una partición del intervalo [a,b], es decir una sucesión finita P : ( x0 , x1 , x2 ,, xn ) con
a  x0  x1  x2    xn 1  xn  b . Para cada k con k  1,2,, n se define el ínfimo y el
supremo de f del subintervalo [ xk 1 , xk ] : mk : inf f ( x ) y M k : sup f ( x) . Estos existen
x[ x k 1 , x k ] x[ x k 1 , x k ]

por el axioma 1.1 de completez (ver pag. 10) porque el contradominio de la función f es acotado
por hipótesis. Se define una suma inferior de f para P, L ( P, f ) , y una suma superior de f para
P, U ( f , P ) , (las dos dependen de P y de f):
n
L( f , P ) : m1 ( x1  x0 )  m2 ( x2  x1 )    mn ( xn  xn 1 )   mk ( xk  xk 1 ) .
k 1
n
U ( f , P) : M 1 ( x1  x0 )  M 2 ( x2  x1 )    M n ( xn  xn 1 )   M k ( xk  xk 1 ) .
k 1

Se dice f es integrable en el intervalo [ a, b] de acuerdo a Riemann si existen sup L ( f , P ) y


P

inf U ( f , P ) y son iguales:


P
b
sup L ( f , P )  inf U ( f , P ) . Se define:
P P  f ( x)dx : sup L( f , P)  inf U ( f , P) .
a P P

 f ( x)dx
a
es la integral definida de f sobre [ a, b] con a como límite de integración inferior y b

b
como límite de integración superior.  f ( x)dx
a
recibe el nombre área de R ( f , a, b) cuando

f ( x )  0  x  [ a , b]

Vamos a explicar esta definición paso por paso:


(Para la ilustración de esta teoría de las sumas inferiores y superiores se copió un parte del libro
“Cálculus, Cálculo infinitesimal, segunda edición, Michael Spivak, Editorial Reverté, S.A., 1992”,
paginas 348-363 y se adicinó muchas explicaciones y comentarios! Se recomienda leer capítulo
13 de este libro, empezando en la pagina 345, para estudiar la integración de Riemann!)

L ( P, f ) y U ( f , P ) son aproximaciones para el área de la región R( f , a, b)  R 2 acotado por la


curva de la función f, el eje x y las rectas x  a y x  b . Se define el área de f sobre un subintervalo
I  [ a, b] como positivo en el caso que f ( x )  0 para todo x en I y negativo en el caso que
f ( x )  0 en cualquier subintervalo (área algebraica). El área total de la región R ( f , a, b) es la
suma de todos estos áreas positivas y negativas sobre el intervalo [ a, b] .

(2.10.1): Si P es una partición cualquiera, entonces L ( f , P )  U ( f , P ) .

Página 82 de 114
Esto vale porque para cada k tenemos mk ( xk  xk 1 )  M k ( xk  xk 1 ) . (Siehe Ordner
“Ordnungsrelationen” Blatt [12]!)

Si P1 y P2 son dos particiones cualesquiera de [ a, b] , entonces L( f , P1 )  U ( f , P2 ) (esto es el


teorema 2.40.i)
Debe ser L( f , P1 )  área R ( f , a, b)  U ( f , P2 ) . Para preparar la demostración, primero vamos a
demostrar un lema:

Lema 2.1:
Si una partición Q del intervalo [ a, b] contiene la partición P del intervalo [ a, b] , es decir para
todos los puntos xi  P vale también xi  Q , entonces L ( f , P )  L ( f , Q ) y U ( f , P )  U ( f , Q ) .

Demostración: Consideremos primero el caso que Q contiene exactamente un punto más que P:
P : ( x0  a, x1 , x2 ,, xn  b) , Q : ( x0  a, x1 , x2 ,, xi1 , u, xi , xn  b) donde
a  x0  x1  x2    xi 1  u  xi    xn 1  xn  b . Ver figura 2.2.
n
Sea m': inf f ( x) , m' ': inf f ( x) , entonces L( f , P )   mk ( xk  xk 1 ) y
x[ x i1 , u ] x[ u , xi ]
k 1
i 1 n
L( f , Q)   mk ( xk  xk 1 )  m' (u  xi 1 )  m' ' ( xi  u )   m (x k k  xk 1 ) . Para demostrar
k 1 k  i 1

L( f , P)  L( f , Q) basta probar que mi ( xi  xi 1 )  m' (u  xi 1 )  m' ' ( xi  u ) . El conjunto


 f ( x) xi 1  x  xi  contiene todos los números de  f ( x) xi 1  x  u  (porque
 f ( x) xi 1  x  u    f ( x) xi 1  x  xi ) y posiblemente otros más pequeños (ver figura 2.2.),
de modo que el ínfimo del primer conjunto, inf  f ( x) x i 1  x  xi   inf
x[ x i 1 , x i ]
f ( x) , es menor o

igual que el ínfimo del segundo, inf  f ( x) xi 1  x  u  inf


x[ x i1 , u ]
f ( x) :
inf f ( x)  inf f ( x)  mi  m' . Análogamente sigue inf f ( x)  inf f ( x)
x[ xi 1 , x i ] x[ xi 1 , u ] x[ x i 1 , x i ] x[ u , x i ]

 mi  m' ' .
Por lo tanto mi ( xi  xi 1 )  mi (u  xi 1 )  mi ( xi  u )  m' (u  xi 1 )  m' ' ( xi  u ) . Esto demuestra, en
este caso especial, que L ( f , P )  L ( f , Q ) .
De manera semejante se puede demostrar para este caso especial que U ( f , P )  U ( f , Q ) .

El caso general puede ahora deducirse facilmente: La partición Q más fino que P puede obtenerse
a partir de P añadiendo un punto cada vez. En otras palabras, existe una sucesión de particiones
P  P1 , P2 , P3 ,, Pn  Q tales que Pk 1 contiene exactamente un punto más que Pk . Como ya
demostramos para el caso especial con las particiones P y Q con Q conteniendo un punto más que
P, vale L( f , Pk )  L( f , Pk 1 ) k  1,2,, n  1 . Porque P  P1 y Pn  Q sigue
L( f , P)  L( f , P1 )  L( f , P2 )  L( f , P3 )    L( f , Pn 1 )  L( f , Pn )  L( f , Q) y análogamente
U ( f , P)  U ( f , P1 )  U ( f , P2 )  U ( f , P3 )    U ( f , Pn 1 )  U ( f , Pn )  U ( f , Q) . Entonces es
demostrado que vale L ( f , P )  L ( f , Q ) y U ( f , P )  U ( f , Q ) . q.e.d.

Página 83 de 114
Teorema 2.40.i:
Sean P1 y P2 particiones arbitrarios del intervalo [ a, b] , y sea f : [ a, b]  R acotada en [ a, b] .
Entonces L( f , P1 )  U ( f , P2 ) .

Demostración: Existe una partición P que contiene a la vez a P1 y P2 (P puede ser la que consiste
en los puntos de P1 y P2. Según lema 2.1 y (2.10.1) sigue L( f , P1 )  L( f , P)  U ( f , P)  U ( f , P2 )
. q.e.d.

Del teorema 2.40.i se sigue que cualquier suma superior U ( f , P' ) es una cota superior para el
conjunto de todas las sumas inferiores  L( f , P ) P partición de [a, b]  . Entonces este conjunto de
números reales tiene una cota superior y según el axioma 1.1 de completez (ver pag. 10) existe el
supremo de este conjunto como la mínima cota superior! Entonces
sup L( f , P ) : sup  L( f , P ) P partición de [a, b]   U ( f , P ' )  P ' . Pero esto signifíca a su vez que
P

sup L( f , P ) es una cota inferior para el conjunto de todas las sumas superiores de f
P

U ( f , P' ) P ' partición de [a, b] . Por eso este conjunto tiene una cota inferior y por eso existe su
ínfimo inf U ( f , P) : inf U ( f , P) P partición de [a, b]  como máxima cota inferior (ver
P

axioma 1.1 de completez). Entonces inf U ( f , P ) es mayor que cualquier otra cota inferior,
P

entonces también es mayor que la cota inferior sup L( f , P ) :


P

(2.10.2): inf U ( f , P )  sup L( f , P )


P P

Porque cualquier suma superior U ( f , P ' ) es una cota superior para el conjunto de todas las sumas
inferiores  L( f , P ) P partición de [a, b]  , este conjunto tiene un supremo. Y porque el supremo
es la mínima cota superior sigue sup L ( f , P )  U ( f , P ' )  P ' (ver arriba). Además sup L( f , P ) es
P P

una cota superior para todas las sumas inferiores L( f , P ' ) (  P ' ), entonces
L ( f , P ' )  sup L ( f , P )  P ' . Por lo tanto tenemos:
P

(2.10.3): L ( f , P ' )  sup L ( f , P )  U ( f , P ' )  P ' .


P

Porque cualquier suma inferior L ( f , P ' ) es una cota inferior para el conjunto de todas las sumas
superiores U ( f , P ) P partición de [ a, b]  , este conjunto tiene un ínfimo. Y porque el ínfimo es la
máxima cota inferior sigue L( f , P ' )  inf U ( f , P )  P ' . Además inf U ( f , P) es es una cota
P P

inferior para todas las sumas superiores U ( f , P ' ) (  P ' ), entonces inf U ( f , P )  U ( f , P ' )  P ' .
P
Por lo tanto tenemos:

Página 84 de 114
(2.10.4): L( f , P ' )  inf U ( f , P )  U ( f , P' )  P' .
P

Por completez seguimos análogamente como arriba empezando con L( f , P ' )  inf U ( f , P )  P ' .
P

Esta desigualdad signifíca a su vez que inf U ( f , P ) es una cota superior para el conjunto de todas
P

las sumas inferiores de f  L( f , P ' ) P ' partición de [a, b] . Por eso este conjunto tiene una cota
superior y por eso existe su supremo sup L( f , P ) como mínima cota superior (ver axioma 1.1 de
P

completez). Entonces sup L( f , P ) es menor que cualquier otra cota superior, entonces también
P

es menor que la cota superior inf U ( f , P ) . Entonces sup L ( f , P )  inf U ( f , P ) . Esto es igual a la
P P P

ecuación (2.10.2), que ya demostramos.

Con (2.10.2), (2.10.3) y (2.10.4) sigue


(2.10.5): L( f , P ' )  sup L( f , P )  inf U ( f , P )  U ( f , P ' )  P '
P P

Según (2.10.5) puede occurir muy bien que sup L( f , P )  inf U ( f , P ) . En este caso, éste es el
P P

único número entre la suma inferior L ( f , P ' ) y la suma superior U ( f , P' ) de f para todas las
particiones P', y este número es en consecuencia un candidato ideal para el área de R ( f , a, b) . Esto
se puede explicar de manera siguiente: sup L ( f , P ) es la mínima cota superior de las sumas
P
inferiores para todas las particones P. Según teorema 1.6 para cualquier   0 existe una
partición P' tal que L( f , P' )  sup L( f , P )   , entonces las sumas inferiores pueden acercarse por
P

abajo al sup L( f , P ) arbitrariamente cerca. De manera semejante existe para cualquier   0 una
P

partición P'' tal que U ( f , P ' ' )  inf U ( f , P )   , entonces las sumas superiores pueden acercarse
P

por arriba al inf U ( f , P ) arbitrariamente cerca. Pero no siempre existe una partición P' talque
P

L( f , P ' )  sup L( f , P ) , y no siempre existe una partición P'' talque U ( f , P ' ' )  inf U ( f , P ) . Los
P P

dos miembros de las sumas inferiores y superiores en la desigualdad (2.10.5) son conectados en el
caso en que vale I : sup L( f , P)  inf U ( f , P ) . En este caso las sumas inferiores pueden acercarse
P P

por abajo al valor I arbitrariamente cerca y las sumas superiores pueden acercarse por arriba al
valor I arbitrariamente cerca. Este valor I entonces es un límite, y cada límite es único, si existe
(ver apéndice 2). Por eso vale, si sup L( f , P )  inf U ( f , P ) , entonces podemos definir el número
P P

I : sup L( f , P)  inf U ( f , P ) que es el único número que satisface la desigualdad (2.10.5) entre
P P

las suma inferiores L ( f , P ' ) y las sumas superiores U ( f , P' ) de f para todas las particiones P':
L( f , P ' )  I  U ( f , P' )  P'

Página 85 de 114
Por otra parte, si sup L( f , P )  inf U ( f , P ) , entonces todo número I comprendido entre
P P

sup L ( f , P ) y inf U ( f , P ) satisfará sup L ( f , P' )  I  inf U ( f , P' ) para todas particiones P'. Ver
P P P' P'

también el texto abajo la definición 2.29.i!

Dos ejemplos para casos extremos:


Ejemplo 1) f ( x )  c  x  [ a, b] . Si P : ( x0  a, x1 , x2 ,, xn  b) con x0  x1  x2    xn 1  xn
es una partición cualquiera de [ a, b] , entonces mk : inf f ( x)  c y
x[ x k 1 , x k ]

M k : sup f ( x)  c  k de manera que


x[ x k 1 , x k ]
n n
L( f , P )   mk ( xk  xk 1 )  c  ( xk  xk 1 )  c(b  a ) y
k 1 k 1
n n
U ( f , P )   M k ( xk  xk 1 )  c ( xk  xk 1 )  c(b  a ) .
k 1 k 1
Entonces todas las sumas inferiores y superiores son iguales, por eso sigue
sup L ( f , P )  inf U ( f , P )  c (b  a ) . Esto es igual al área de la región R ( f , a, b) para este caso!
P P

1, si x racional
Ejemplo 2) f ( x)   , x  [ a, b ] .
0, si x irracional
Si P es una partición cualquiera de [ a, b] como en ejemplo 1), entonces mk : inf f ( x)  0 ,
x[ x k 1 , x k ]

puesto que existe un número irracional en [ xk 1 , xk ] y M k : sup f ( x)  1 , puesto que existe


x[ x k 1 , x k ]

un número racional en [ xk 1 , xk ] .
n n
Por lo tanto, L( f , P)   mk ( xk  xk 1 )  0 ( xk  xk 1 )  0 y
k 1 k 1
n n
U ( f , P )   M k ( xk  xk 1 )  1 ( xk  xk 1 )  b  a . Entonces sup L ( f , P)  0 y
k 1 k 1 P

inf U ( f , P )  b  a , sale que no es verdad sup L( f , P )  inf U ( f , P ) ! Para este ejemplo la medida
P P P

del área de R ( f , a, b) podria ser 0 o b  a y la región R ( f , a, b) es tan extraña que sale que el
área de esta region extraña no se puede medir con la teoría de Riemann!

Pero hay otra teoría de integrales, la integración de Lebesgue, que si puede medir el área de una
región R ( f , a, b) parecida! Sale que si una función que es integrable de acuerdo a Riemann,
también es integrable de acuerdo a Lebesgue y las dos integrales son iguales! (Barner-Flohr, Analysis
II, Seite 293,294; 254!). Para los casos de dos dimensiones o más se puede definir una función parecida
que tiene vectores x  R n como argumento:

Página 86 de 114
1, si x  M  Q n
g ( x)   , M  R n . Q n es el conjunto de los números racionales en el R n :
 0, para todo otro x
Q n  Q  Q  Q    Q , Q es el conjunto de todos los números racionales en R. La integral de
  
n veces

g (x ) sobre el conjunto M  R n es cero con respecto a la integración de Lebesgue:  g d  0 .


M

Esto vale porque la medida del conjunto M  Q n es cero:  ( M  Q n )  0 . Aqui se usa la medida
 de Lebesgue. Pero  g d  0 solamente vale en el
M
R n con n  1 .

Por lo tanto se puede motivar la segunda parte de la definición 2.29 que indica la definición de la
integrabilidad de una función f sobre el intervalo [ a, b] de acuerdo a Riemann. Si vale
b
f ( x )  0 x  [ a, b]  f ( x)dx : sup L( f , P)  inf U ( f , P)
a P P
es el área bajo la curva de la función

f entre a y b. Si f es integrable, entonces según esta definición y con la ecuación (2.10.5),


b b
L( f , P ' )   f ( x)dx  U ( f , P ' )  P ' . Además  f ( x)dx es el único número con esta propiedad
a a
(ver el comentario abajo la ecuación (2.10.5)!). Pero todavia no sabemos cuáles funciones son
integrables ni tampoco sabemos cómo hallar la integral de f sobre el intervalo [ a, b] cuando f es
integrable. Los dos ejemplos se puede interpretar de la manera siguiente:
b
Ejemplo 1) f ( x )  c  x  [ a, b] es integrable sobre el intervalo [ a, b] y  f ( x)dx  c(b  a)
a
(ver

teorema 2.42).
1, si x racional
Ejemplo 2) f ( x)   , x  [ a, b] no es integrable sobre el intervalo [ a, b] (de
0, si x irracional
acuerdo a Riemann)

Teorema 2.40.ii:
Si f : [ a, b]  R está acotada sobre [ a, b] , entonces f es integrable sobre [ a, b] si y solo si para
todo   0 existe una partición P de [ a, b] tal que U ( f , P )  L ( f , P )   .

Demostración: Según (2.10.1) siempre vale U ( f , P )  L ( f , P )  0 .


Supongamos en primer lugar que para todo   0 existe una partición P de [ a, b] tal que
U ( f , P)  L( f , P)   .
Al ser inf U ( f , P ' )  U ( f , P ) y sup L( f , P ' )  L( f , P )   sup L( f , P ' )   L( f , P ) se sigue que
P' P' P'

inf U ( f , P' )  sup L( f , P' )  U ( f , P )  L( f , P)      0 . Se sigue que


P' P'

Página 87 de 114
inf U ( f , P ' )  sup L( f , P ' )  0  inf U ( f , P ' )  sup L ( f , P ' ) y por la definición 2.29, f es
P' P' P' P'
integrable.

Ahora supongamos que f es integrable, entonces inf U ( f , P)  sup L ( f , P ) . Esto significa


P P

inf U ( f , P )  sup L ( f , P)  ~  ~  0 . Para el supremo del conjunto de todas las sumas


P P

inferiores vale: Para cualquier  '  0 existe una partición P' tal que L ( f , P ' )  sup L ( f , P )   ' ,
P

porque no hay otra cota superior de todas las sumas inferiores que es menor que sup L( f , P ) , el
P

sup L( f , P ) es la mínima cota superior (ver teorema 1.6 y su demostración en pagina 11). De
P
manera semejante vale para el ínfimo del conjunto de todas las sumas superiores: Para cualquier
 ''  0 existe una partición P'' tal que U ( f , P ' ' )  inf U ( f , P)   ' ' , porque no hay otra cota
P
inferior de todas las sumas superiores que es mayor que inf U ( f , P ) , el inf U ( f , P ) es la máxima
P P
cota inferior. Entonces, del enunciado inf U ( f , P)  sup L( f , P )  ~  ~  0 se sigue
P P

 ( U ( f , P ' ' )   ' ' )  ( L ( f , P ' )   ' )  inf U ( f , P )  sup L ( f , P )  ~  ~,  ' ,  ' '  0 por que
P P

U ( f , P ' ' )   ' '  inf U ( f , P ) y L ( f , P ' )   '  sup L ( f , P )  ( L( f , P ' )   ' )   sup L ( f , P) .
P P P

Esto es equivalente a U ( f , P ' ' )  L ( f , P ' )  ~   ' ' '  ~,  ' ,  ' '  0 . Si definimos
 : ~   ' ' ' esto es equivalente a U ( f , P ' ' )  L( f , P ' )      0 , porque la desigualdad vale
para cualesquier ~,  ' ,  ' '  0 .
Resumiendo podemos decir: De “ inf U ( f , P)  sup L( f , P )  ~  ~  0 ” se sigue el enunciado
P P

“para todo   0 existen particiones P' y P'' con U ( f , P ' ' )  L ( f , P ' )      0 ”.
Sea P una partición que contiene a la vez P' y P''. Entonces sigue según el lema 2.1:
U ( f , P)  U ( f , P' ' ) y L ( f , P )  L ( f , P ' )   L ( f , P )   L ( f , P ' ) . En consecuencia,
U ( f , P )  L ( f , P )  U ( f , P ' ' )  L ( f , P ' )      0 . Entonces si existe una partición P tal que
U ( f , P )  L ( f , P )      0 que demuestra la otra dirección del enunciado del
teorema 2.40.ii. Por lo tanto queda este teorema demostrado. q.e.d.

El teorema 2.40.ii ofrece una otra posibilidad para definir la integrabilidad de una función,
porque el criterio “para todo   0 existe una partición P de [a, b] tal que U ( f , P )  L( f , P )  
” es equivalente a la definición 2.29 de la integrabilidad. Entonces se puede definir la
integrabilidad de manera siguiente:
Definición 2.29.i: Integrabilidad de una función
f : [ a, b]  R acotada en [ a, b] .
Sea P una partición del intervalo [a,b], es decir una sucesión finita P : ( x0 , x1 , x2 ,, xn ) con
a  x0  x1  x2    xn 1  xn  b .
Se dice f es integrable en el intervalo [ a, b] si para todo   0 existe una partición P de [ a, b]
tal que U ( f , P )  L ( f , P )   .

Página 88 de 114
Esta definición equivale solamente a expresar de todo modo la definición de la integrabilidad,
b
pero no da directamente una definición del valor del integral  f ( x)dx como en la definición 2.29.
a
El avantage de la definición 2.29.i en comparison de la definición 2.29 es que la definición 2.29.i
no necesita supremos o ínfimos, que muchas veces son difíciles de manejar.

Como hemos mencionado en el texto abajo de la desigualdad (2.10.5), para el caso


sup L( f , P )  inf U ( f , P ) , que significa que f es integrable según definición 2.29, el número
P P

I : sup L( f , P)  inf U ( f , P ) que satisface la desigualdad (2.10.5) L ( f , P ' )  I  U ( f , P ' )  P '


P P

es único! Según la definición 2.29, este número I es igual a la integral definida de de f sobre [ a, b]
b
: I   f ( x)dx : sup L( f , P )  inf U ( f , P ) .
P P
a

comentario 2.10.1
Esto significa, (1) si vale sup L( f , P )  inf U ( f , P ) (si f es integrable) y (2) si se encuentra un
P P

número I que satisface la desigualdad (2.10.5) L ( f , P ' )  I  U ( f , P ' )  P ' , entonces este
b b
número I es único y tenemos I   f ( x)dx : Entonces L( f , P ' )   f ( x)dx  U ( f , P ' )  P ' .
a a

El próximo ejemplo ilustra este punto:

0 , x  1
Ejemplo 3) f ( x)   x  [0,2] . f no es continua en el punto x  1 y entonces no es
1, x  1
continua en el intervalo [0,2] .
Sea P : ( x0 , x1 , x2 ,, xn ) una partición del intervalo [0,2] con
0  x0  x1  x2   x j 1  1  x j    xn 1  xn  2 . Supongamos que el intervalo [ x j 1 , x j ]
contenga el punto x  1 . Vale mk : inf f ( x)  0  k , M k : sup f ( x)  0  k  j , y por
x[ x k 1 , x k ] x[ x k 1 , x k ]
n
eso M j : sup f ( x)  1 . L( f , P )   mk ( xk  xk 1 )  0 ,
x[ x j 1 , x j ] k 1
n
U ( f , P)   M k ( xk  xk 1 )  x j  x j 1 .
k 1

Entonces U ( f , P)  L( f , P)  x j  x j 1 . Para que f es integrable sobre [0,2] según la


definición 2.29.i debe existir una partición P de [0,2] para todo   0 tal que
U ( f , P )  L ( f , P )   . Podemos elegir nuestra partición P tal que la longitud del intervalo
[ x j 1 , x j ] es menor  para cualquier   0 : x j 1  1  x j con x j  x j 1   , entonces tenemos
U ( f , P)  L( f , P)  x j  x j 1      0 . Por lo tanto sale que f es integrable sobre [0,2] , pero

Página 89 de 114
2
todavia no sabemos el valor de la integral  f ( x)dx . Pero si sabemos que vale
0
L( f , P' )  0  P' y

según (2.10.1) vale L( f , P' )  U ( f , P' )  P' . Entonces es claro que vale
L ( f , P ' )  0  U ( f , P ' )  P' . Entonces encontramos un número I  0 que satisface la
desigualdad (2.10.5) y porque ya hemos demostrado que f es integrable sobre [0,2] sigue que este
2
número es el valor de nuestra integral de f del ejemplo 3):  f ( x)dx  0 (ver comentario 2.10.1
0
abajo de la definición 2.29.i!).

Esta función f del ejemplo 3) no es continua en el punto x  1 y entonces no es continua en el


intervalo [0,2] , pero si se puede calcular su integral definida de acuerdo a Riemann. El mismo
resultado saldría para una función con un número finito de puntos de discontinuidad. Esto se puede
demostrar de manera análoga como para el ejemplo 3). En este caso sale una suma sobre un
número finito de longitudes de intervalos de la partición P que debe ser menor de cualquier   0
. Con esta condición no hay problemas. Entonces se puede concluir que también las funciones con
un número finito de puntos de discontinuidad (no infinita) sí son integrables de acuerdo a
Riemann. También se puede demostrar esto con la ayuda de teorema 2.43 si f no esta contínua en
un punto x1 y g no esta contínua en un punto x2  x1 . También se puede multiplicar el valor 1 con
cualquier constante c y todavía la función es integrable; esto sigue también de teorema 2.43. El
teorema dice que la suma de estas dos funciones no continuas respectivamente en un punto es
integrable y además presenta un valor para la integral de la suma! Asi se puede continuar la
demostración sucesivamente hasta una suma finita de funciones que tienen un punto de
discontinuidad con cualquier valor ci respectivamente! Entonces sigue que cada función con un
número finito de desigualdades (no infinitas) es integrable según definición 2.29 o 2.29.i.
Al contrario la función del ejemplo 2) tiene un número infinito de puntos donde la función no es
continua y por eso salió que no es integrable de acuerdo a Riemann! ((x) Siehe Heuser: Auch bei einer
unendlichen Anzahl von Diskontinuitaeten ist die Funktion integrierbar nach Rieman, siehe ausserdem die Definition
eines Integrales ueber eine Diskontinuitaet einer Funktion, Cálculo II, Ibero!)

Ejemplo 4) f ( x )  x , x  [0, b] con b  0 .


Sea P : ( x0 , x1 , x2 ,, xn ) una partición del intervalo [0, b] con
0  x0  x1  x2    xn 1  xn  b . Las sumas inferiores y superiores son las siguientes:
n n n n
L( f , P)   mk ( xk  xk 1 )   xk 1 ( xk  xk 1 ) , U ( f , P)   M k ( xk  xk 1 )   xk ( xk  xk 1 ) . Se
k 1 k 1 k 1 k 1
puede simplificar estas dos sumas considerablemente usando n subintervalos con longitud igual:
( xk  xk 1 )  b / n  k ; Pn : ( x0  0, x1  b / n, x2  2b / n,, xk  kb / n, xn  nb / n  b) .
2 n
n n
(k  1)b b  b 
Entonces L( f , Pn )   xk 1 ( xk  xk 1 )      (k  1) .
k 1 k 1 n n n k 1

La serie aritmética es la siguiente: sn  a1  a2    an 1  an con


d  an  an 1    a3  a2  a2  a1  const. Entonces a2  a1  d , a3  a2  d  a1  2d ,

Página 90 de 114
a4  a3  d  a2  2d  a1  3d , ..., ak  a1  (k  1) d , ..., an  a1  (n  1) d .
n
2sn  2 ak  a1  a2    an 1  an
k 1

 an  an 1    a2  a1
con a2  an 1  a1  d  an  d  a1  an , a3  an  2  a2  d  an 1  d  a2  an 1  a1  an , ....
n
n n
Entonces 2 sn  2 ak  n(a1  an )  sn  (a1  an )  [2a1  (n  1)d ] .
k 1 2 2
n n
n(n  1)
En nuestro ejemplo  (k  1) tenemos a1  0 , d  1, an  n  1 . Entonces
k 1
 (k  1) 
k 1 2
.
2 n 2
 b  n(n  1) b n  1
2
b
Por lo tanto encontramos L( f , Pn )   
n
 (k  1)   n 
k 1 2

2 n
.
2 n
kb b  b 
n
Análogamente encontramos U ( f , Pn )      k . Tenemos una serie artitmética con
k 1 n n n k 1
2
n
kb b  b  n(n  1) b 2 n  1
a1  1, d  1, an  n , entonces U ( f , Pn )      . El límite de las
k 1 n n n 2 2 n
n 1 n 1 n 1 1  1/ n
sucesiónes y para n   es 1: lim  lim  1 (ver capítulo 1.2). Entonces
n n n   n n   1
b2 n  1 b2 b2 n  1 b2
encontramos lim L( f , Pn )  lim  y lim U ( f , Pn )  lim  . Vale
n  2 n n 2 n  2 n  n 2
b2 2 b2 2
U ( f , Pn )  L( f , Pn )  y lim U ( f , Pn )  L( f , Pn )   lim  0 . Esto es equivalente al
2 n n 2 n  n
enunciado siguiente:
Para cualquier   0 (   R  ) existe un número N   0 ( N  R  ), es decir existe una partición
PN con N como el proximo número natural que sigue el número real N , tal que para todo n  N
( n  N ), es decir para todas las particiones Pn con n  N , entonces U ( f , Pn )  L( f , Pn )   0  
 U ( f , Pn )  L( f , Pn )   .
Por eso la función f es integrable según definición 2.29.i!

Además podemos usar el comentario 2.10 para calcular el número de la integral de f sobre [0, b]
b2 n  1 b2  1 
: Vale L( f , Pn )   1   , n  1,2,3, y por eso encontramos la desigualdad
2 n 2  n
b2
L( f , Pn )   n  N . Además L( f , Pn ) es una sucesión estrictamente creciente. Entonces
2
L( f , Pn ) es una sucesión monótona (creciente) y acotada. Por eso existe un límite y este es igual
al supremo sup L( f , Pn ) , como demostramos en el teorema 1.5! Entonces sabemos que el límite
Pn

Página 91 de 114
b2
que hemos calculado arriba es igual a sup L( f , Pn ) , porque el límite de una sucesión es único
2 Pn

b2
(ver apéndice 2): lim L( f , Pn )   sup L( f , Pn ) .
n  2 Pn

b2
De manera semejante se puede demostrar que lim U ( f , Pn )   inf U ( f , Pn ) :
n  2 Pn

b n 1 b  1 
2 2
b 2
U ( f , Pn )   1   , n  1,2,3, ,  U ( f , Pn )   n  N . Entonces U ( f , Pn ) es
2 n 2  n 2
una sucesión decreciente y acotada por abajo y por eso converge al ínfimo inf U ( f , Pn ) .
Pn

b2
Con estos resultados podemos concluir sup L( f , Pn )  inf U ( f , Pn ) 
. Entonces
Pn Pn 2
L( f , Pm )  sup L( f , Pn )  inf U ( f , Pn )  b 2 2  U ( f , Pm )  Pm o  m  N . Desgraciadamente todo
Pn Pn

esto solamente vale para todas estas ciertas particiones especiales Pn, no lo hemos demostrado
para todas particiones posibles P! Por eso con estas demostraciones sin más razón todavia no se
puede concluir que la integral de f sobre [0, b] es igual a b 2 2 . Pero más adelante vamos a ver que
con la ayuda de las sumas de Riemann (ver teorema 2.41) si se puede concluir esto, ver
comentario 2.10.2! Aquí también vamos a encontrar que al final vale
b
lim L( f , Pn )  lim U ( f , Pn )   f ( x)dx , como explica el texto siguiente:
n n 
a
Aquí podemos solver este problema de la manera siguiente: El teorema 2.40.ii dice que f es
integrable sobre [0, b] si y solo si para todo   0 existe una partición P de [0, b] (es decir al
menos una partición especial o más particiones) tal que U ( f , P )  L ( f , P )   . Pero esto es
precisamente el caso en nuestro ejemplo; las particiones especiales son los Pn  n  N ! Entonces
b
f es integrable y esto es equivalente a  f ( x) dx : sup L( f , P)  inf U ( f , P)
0 P P
según

definición 2.29! Por eso vale con (2.10.5) además:


b
L( f , P ' )  I  sup L( f , P )   f ( x) dx  inf U ( f , P )  U ( f , P ' )  P ' y el número I que satisface
P P
0
esta desigualdad es único! Esto significa: Si vale esta desigualdad para todas las particiones P'
(como en nuestro caso) con un número único I, entonces también tiene que valer para ciertas
particiones Pn especiales con este mismo número I. Es decir, el número I' para lo cual vale
L( f , Pm )  I '  sup L( f , Pn )  inf U ( f , Pn )  b 2 2  U ( f , Pm )  Pm (para estas ciertas particiones
Pn Pn

especiales), que también es único porque es un límite, debe ser el mismo número que aparece en
la desigualdad L ( f , P ' )  I  sup L ( f , P )  inf U ( f , P )  U ( f , P ' )  P ' . Entonces: I' = I! Todo
P P

esto bajo la condición, que si vale esta ultima desigualdad para todas particiones posibles! Pero
esta si vale porque arriba podimos demostrar que f es integrable! Entonces: Bajo estas condiciones
si se puede concluir de la desigualdad

Página 92 de 114
L( f , Pm )  sup L( f , Pn )  inf U ( f , Pn )  b 2 2  U ( f , Pm )  Pm que los dos números I '  b 2 2 y I
Pn Pn

b
son iguales, es decir que vale I ´ I   x dx  b 2 2 . (Ver también el texto abajo la demostración
0

de “teorema 2.40.iii  teorema 2.41” con el cálculo de esta integral con la ayuda de sumas de
Riemann y el comentario 2.10.2!)
Este número b 2 2 si es el área debajo de la curva de la función f ( x )  x entre x  0 y x  b
como se puede demostrar facilmente con la formula del área A de un triángulo: A  g  h / 2 con g
como base ( g  b ) y h altura ( h  f (b)  b ) del rectangulo. Entonces: A  b  b / 2  b 2 / 2 .

Teorema 2.40:
Si una función f : [ a, b]  R es continua en el intervalo cerrado [ a, b] , entonces f es integrable
(según definición 2.29 o definición 2.29.i) en [ a, b] .

Demostración:
f : [ a, b]  R es continua en el intervalo cerrado [ a, b] , por eso es acotada (ver teorema 2.19).
Para demostrar que f es integrable en [ a, b] usamos el teorema teorema 2.40.ii: Hay que demostrar
que para todo   0 existe una partición P de [ a, b] tal que U ( f , P )  L ( f , P )   . Porque
f : [ a, b]  R es continua en el intervalo cerrado [ a, b] que es compácto (acotado y completo),
también es uniformemente continua en [ a, b] . Es decir, para todo   0 existe algún   0 tal que,
para todos x e y en [ a, b] con x  y   se cumple f ( x)  f ( y )   . Por que esto vale para todo
  0 , entonces también para  : ~ /[ 2(b  a )] .
El   0 depende en el caso de la continuidad en un punto x0  [a, b] normalmente de   0 y de
x0 :    ( , x0 ) . En el caso de la continuidad uniforme el valor de   0 se puede eligir
independientemente del punto x0  [a, b] (uniformemente para todo x0  [a, b] ):    ( ) !

Anmerkungen in eigener Sache: Barner-Flohr Bd. II, Seite 26:


Kompakte Mengen: M  R n . Folgende 3 Aussagen sind äquivalent: 1.) M ist beschränkt und abgeschlossen. 2.) Aus jeder
~ ~
Überdeckung U von M mit offenen Mengen ( U ( x,  ( x)) ist eine -Umgebung von x, System U all dieser Umgebungen
überdeckt M: M U ( x,  ( x)) ) lassen sich endlich viele dieser Mengen so auswählen, dass M von ihnen überdeckt wird,
xM
r
d.h. man kann endlich viele Punkte x1 , x 2 ,  , x r  M derart auswählen, dass auch schon M  ,  ( x k )) gilt („M hat
U ( x
k 1
k

die Heine-Borel-Überdeckungseigenschaft“).(Siehe Abh. Mathematik II, Blatt (10c))R und „Überdeckungssatz von Heine-Borel“
in Barner-Flohr Bd. I, Seite 228.) 3.) Jede unendliche Teilmenge von M besitzt einen Häufungspunkt in M.

Def. der gleichmässigen Stetigkeit in Abh. Zusammenfassung Variationsrechnung, Blatt (1b)). Satz: Ist die Definitionsmenge einer
stetigen Abbildung kompakt, so ist die Abbildung gleichmässig stetig. Beweis dort und in Barner-Flohr Bd. I, Seite 243! Beweis
erfolgt unter Ausnutzung der Heine-Borel-Eigenschaft der kompakten Definitionsmenge! Beweis auch in Cálculo infinitesimal,
seg. Edición, pag. 191,192!

Página 93 de 114
Se elige una Partición P : ( x0 , x1 , x2 ,, xn ) con a  x0  x1  x2    xn 1  xn  b tal que se
cumpla xk  xk 1    k  1,2,, n para un   0 que depende de f y de  : ~ /[ 2(b  a )] , con
cualquier ~  0 . Este   0 existe porque f es uniformemente continua en [ a, b] . Por eso se tiene
para todo k: f ( x )  f ( y )  ~ /[2(b  a )] para todos los x, y  [ xk 1 , xk ] . Además tenemos para la
diferencia de los suprémos y ínfimos M k  mk 
 sup f ( x)  inf ~ ~
f ( x)   /[2(b  a )]   /(b  a )  k  1,2,, n . Entonces
x[ x k 1 , x k ] x[ x k 1 , x k ]

n
~ n
~
U ( f , P )  L( f , P )   ( M k  mk )( xk  xk 1 )  (b  a )  ~ .
 ( xk  xk 1 ) 
k 1 b  a k 1 b  a
Denotamos ahora ~ como  que ahora tomará el papel del ~ y que no tiene nada que ver con el
 que usamos arriba:
Entonces existe para todo   0 una partición P de [ a, b] (esta que definimos arriba) tal que
U ( f , P )  L( f , P )   . Por eso f es integrable sobre [ a, b] según definición 2.29.i! q.e.d.

Página 94 de 114
Las sumas de Riemann:

Suponemos que P : ( x0 , x1 , x2 ,, xn ) es una partición del intervalo [ a, b] con


a  x0  x1  x2    xn 1  xn  b y que para cada k elegimos cualquier punto t k  [ xk 1 , xk ]  k
. De esta manera se define el conjunto TP : t1 , t2 ,, tn . Se tiene entonces
mk  inf f ( x)  f (tk )  M k  sup f ( x)  k . Por eso sigue
x[ x k 1 , x k ] x[ x k 1 , x k ]
n n n
L( f , P )   mk ( xk  xk 1 )   f (tk )( xk  xk 1 )  U ( f , P )   M k ( xk  xk 1 ) , entonces
k 1 k 1 k 1
n
(2.10.6): L( f , P)   f (tk )( xk  xk 1 )  U ( f , P) para este P y este conjunto TP elegido.
k 1

n
Una suma tal como  f (t
k 1
k )( xk  xk 1 ) recibe el nombre de suma de Riemann de f para la

partición P. La interpretación geométrica de una suma de Riemann: Se trata del área total de n
rectángulos que quedan en parte por encima y en parte por debajo de la gráfica de f. Debido al
modo arbitrario en que se han tomado las alturas de los rectángulos no se puede establecer con
b
suguridad si una determinada suma de Riemann es mayor o es menor que la integral  f ( x)dx .
a

Si las bases de los rectángulos son suficientemente estrechas, la suma de Riemann tendría que
aproximarse a la integral. El siguiente teorema 2.40.iii establece esto con precisión.

Teorema 2.40.iii:
Si una función f : [ a, b]  R es continua en [ a, b] .
Entonces para todo   0 existe algún   0 tal que, si P : ( x0 , x1 , x2 ,, xn ) con
a  x0  x1  x2    xn 1  xn  b es una partición cualquiera de [ a, b] con todas las longitudes
xk  xk 1    k  1,2,, n y si definimos un conjunto TP : t1 , t2 ,, tn  con xk 1  tk  xk  k
n
y si se define una suma de Riemann  f (t ) ( x
k 1
k k  xk 1 ) ,

n b
Entonces  f (t )( x
k 1
k k  xk 1 )   f ( x)dx   para la suma de Riemann formada usando
a

cualquier conjunto TP .


La demostración es para funciones continuas, pero el teorema vale en realidad para cualquier
función integrable! (Omitimos esta demostración más general porque es más bien intrincada y no
muy instructiva.)

Demostración (Comparar la demostración de teorema 2.40):

Página 95 de 114
Porque f : [ a, b]  R es continua en el intervalo cerrado [ a, b] que es compácto (acotado y
completo), también es uniformemente continua en [ a, b] . Es decir, para todo   0 existe algún
  0 tal que, para todos x e y en [ a, b] con x  y   se cumple f ( x )  f ( y )   . Por que esto
vale para todo   0 , entonces también para  : ~ /[ 2(b  a )] . Se elige una Partición
P : ( x0 , x1 , x2 ,, xn ) a  x0  x1  x2    xn 1  xn  b
con tal que se cumpla
~
xk  xk 1    k  1,2,, n para un   0 que depende de f y de  :  /[ 2(b  a )] , con cualquier
~  0 . Este   0 existe porque f es uniformemente continua en [ a, b] . Se elige cualquier tk en el
intervalo k ( xk 1  tk  xk ) de TP . Como ya hemos visto en la demostración de teorema 2.40 sigue
U ( f , P )  L( f , P )  ~ .
 

Entonces existe para todo   0 una partición P de [a, b] (esta que definimos arriba) tal que
U ( f , P )  L( f , P )   (i).
Entonces f es integrable sobre [ a, b] según definición 2.29.i como lo hemos demostrado (ver
teorema 2.40) y por eso podemos usar el enunciado de comentario 2.10.1: Si f es integrable y si
se encuentra un número I que satisface la desigualdad (2.10.5) L ( f , P ' )  I  U ( f , P ' )  P ' ,
b
entonces este número I es único y tenemos I   f ( x)dx . Por eso vale
a
b
L( f , P ' )   f ( x)dx  U ( f , P ' )  P ' y naturalmente vale también para P '  P :
a
b
L( f , P )   f ( x)dx  U ( f , P ) (ii).
a
n
Pero también tenemos L( f , P )   f (tk )( xk  xk 1 )  U ( f , P ) (iii) (para el P y TP elegído)
k 1
según (2.10.6).

Con las tres ecuaciones (i) , (ii) y (iii) sigue la desigualdad buscada: Con (i) tenemos
b
U ( f , P )  L( f , P )   , con (ii) sigue   f ( x)dx   L( f , P ) y con (iii) sigue
a
n n b


k 1
f (tk )( xk  xk 1 )  U ( f , P ) . Entonces  f (t )( x k k  xk 1 )   f ( x)dx  U ( f , P )  L( f , P )   .
k 1 a
n b
Pero hay que demostrar que vale  f (t )( x
k 1
k k  xk 1 )   f ( x)dx   . Existen dos casos posibles
a

n b
para la norma: Para el caso que  f (t )( x
k 1
k k  xk 1 )   f ( x)dx  0 no tenemos que hacer nada más:
a
n b n b
Vale  f (t )( x
k 1
k k  xk 1 )   f ( x)dx   f (tk )( xk  xk 1 )   f ( x)dx  
k 1
como ya hemos
a a

Página 96 de 114
n b
demostrado arriba. El otro caso posible es  f (t )( x
k 1
k k  xk 1 )   f ( x)dx  0 . En este caso vale
a
n b b n

 f (t )( x
k 1
k k  xk 1 )   f ( x)dx   f ( x)dx   f (tk )( xk  xk 1 ) .
k 1
Ahora usamos otras
a a
b
desigualdades: De la desigualdad (ii) usamos  f ( x)dx  U ( f , P )
a
y de la desigualdad (iii)
n
elegimos   f (tk )( xk  xk 1 )   L( f , P ) . Con estas dos igualdades tenemos entonces
k 1
b n n

 f ( x)dx   f (tk )( xk  xk 1 )  U ( f , P )   f (tk )( xk  xk 1 )  U ( f , P )  L( f , P )   . Por lo


a k 1 k 1

n b
tanto ya demostramos que vale  f (tk )( xk  xk 1 )   f ( x)dx   .
k 1 a

Entonces hemos demostrado que para cualquier   0 existe un   0 (que depende de f y de  )


con que podemos definir una partición P de [ a, b] y un conjunto TP tal que
n b

 f (tk )( xk  xk 1 )   f ( x)dx   . q.e.d.


k 1 a

Con este teorema se puede demostrar el teorema teorema 2.41:


Sea Pn : ( x0( n ) , x1( n ) , x2( n ) ,, xn( n ) ) con a  x0( n )  x1( n )  x2( n )    xn( n)1  xn( n )  b  n  N una
partición del intervalo [ a, b] con n  1 puntos que definen n subintervalos. Definimos el longitud
maximo de todos los subintevalos de Pn como  ( Pn ) : max( xk  xk 1 ) . Con un conjunto
( n) (n)
k


TPn : t , t , , t
(n)
1
(n)
2
(n)
n  con x( n)
k 1 t ( n)
k x ( n)
k k se define la suma de Riemann
n
Sn (TPn , f ) :  f (tk( n ) )( xk( n )  xk( n)1 ) con L( f , Pn )  S (TPn , f )  U ( f , Pn )  n .
k 1

Teorema 2.41:
La función f : [a, b]  R sea continua en [a, b] .
Para cada sucesión de particiones Pn : ( x0( n ) , x1( n ) , x2( n ) ,, xn( n ) ) con
a  x0( n )  x1( n )  x2( n )    xn( n)1  xn( n )  b , n  N del intervalo [ a, b] con lim  ( Pn )  0 y para
n
b
cada seleción de conjuntos TPn vale lim S n (TPn , f )   f ( x)dx (según definición 2.29).
n
a

(Como teorema 2.40.iii este teorema 2.41 vale en realidad para cualquier función integrable!)

Página 97 de 114
b n
Eso significa:  f ( x)dx  lim S n (TPn , f )  lim  f (tk( n ) )( xk( n )  xk( n)1 ) ( ( xk( n )  xk( n)1 ) :  k( n ) es el
n n 

 
a k 1

longitud del k-esima intervalo de la n-esima partición) con lim max( xk  xk 1 )  0 . Definimos
( n) ( n)
n  k

max( xk( n )  xk( n)1 )  max  k( n ) :  ( Pn ) podemos escribir más compacto lim  ( Pn )  0 .
k k n

Según el teorema 2.40.iii existe para todo  0 algún  0 tal que


n b

 f (t )( x
k 1
k k  xk 1 )   f ( x)dx   para cualquier suma de Riemann formada usando un conjunto
a

TPn , . La partición usado fue definida como P : ( x0 , x1 , x2 ,, xn ) con todas las longitudes
xk  xk 1    k  1,2,, n   ( Pn ) : max( xk( n )  xk( n)1 )   . El conjunto TP fue definido
k

solamente con las condiciones xk 1  tk  xk  k .


La hipótesis del teorema 2.41 es que se define una sucesión de particiones
Pn : ( x0( n ) , x1( n ) , x2( n ) ,, xn( n ) ) con lim  ( Pn )  0 . Este última propiedad es equivalente a
n

   N  , tal que  ( Pn )    n  N   que es equivalente a

     N , tal que max( xk( n )  xk( n)1 )    n  N  que también es equivalente a


 k 

   N , tal que xk  xk 1    n  N  k
(n) (n)

Entonces se usa subintervalos que satisfacen la condición xk( n )  xk( n)1    n  N ,  k para
cualquier   0 , entonces también para el   0 que sale en la hipótesis del teorema 2.40.iii que
depende de f y de  : ~ /[ 2(b  a )] , con cualquier ~  0 y que existe porque f es uniformemente
continua en [ a, b] . Además sigue el enunciado del teorema 2.40.iii para todas estas particiones
que satisfacen esta condición ( xk  xk 1    k ), entonces también para todas las particiones
Pn  n  N ! Entonces sigue con la hipótesis del teorema 2.41 con la ayuda del teorema 2.40.iii:
n b
~
Para todo   0 existe un número N  N  , tal que  f (tk( n) )( xk( n)  xk( n)1 )   f ( x)dx   , o en
k 1 a
b
~
otra forma S n (TPn , f )   f ( x)dx   , para todas las particiones Pn  n  N  N  y para
a

cualesquiera conjuntos TPn relacionados! El número N depende también de  , porque 


depende de  , entonces podemos escribir en lugar de N el mismo número dependiente de  ,
~
N  . Finalmente vemos que este enunciado es exactamente equivalente al enunciado del
b
teorema 2.41: lim S n (TPn , f )   f ( x)dx !
n
a
Todo esto es la demostración que del teorema 2.40.iii sigue el teorema 2.41
(teorema 2.40.iii  teorema 2.41)! Se puede demostrar también la dirección inversa,

Página 98 de 114
teorema 2.41  teorema 2.40.iii, entonces los dos teoremas son equivaentes. La demostración
de teorema 2.41  teorema 2.40.iii se puede demostrar indirectamente con
non[teorema 2.40.iii]  non[teorema 2.41] (non[enunciado] es la negación del enunciado).
Pero esto es más sofisticado sobre todo respectivo la negación de enunciados intercalados y por
eso no lo vamos a demostrar en este texto.

Ejemplo:
b
Un ejemplo para la convergencia de sumas de Riemann tal que lim S n (TPn , f )   f ( x)dx ya hemos
n
a

visto en ejemplo 4) con f ( x )  x , x  [0, b] con b  0 . Podemos usar la partición


Pn : ( x (n)
0  0, x (n)
1  b / n, x
(n)
2  2b / n,, x (n)
k  kb / n, x
( n)
n  nb / n  b) y un conjunto TPn con
k 1 k  1/ 2 k
xk( n)1  b  tk( n )  b  xk( n )  b  k
n n n
y construir la sucesión de las sumas de Riemann
2 n
n n n
k  1/ 2 b  b 
S n (TPn , f )   f (tk( n ) )( xk( n )  xk( n)1 )   tk( n ) ( xk( n )  xk( n)1 )   b    ( k  1 / 2) . Según
k 1 k 1 k 1 n n n k 1
n
Para nuestra serie aritmética  ( k  1 / 2)
k 1
tenemos a1  1 / 2 , d  1, an  n  1 / 2 , entonces
2 n
n
n2 b b2
 (k  1 / 2)  . Por lo tanto encontramos S n (TPn , f )   
2 n
 ( k  1 / 2) 
k 1 2
 n . Entonces
k 1
b 2
b
sigue lim S n (TPn , f )   x dx 
. Este límite también hemos encontrado como límite de las
n 
0
2
sumas inferiores y de las sumas superiores para este ejemplo usando las mismas particiones Pn!
Estas particiones si satisfacen los hipótesis de teorema 2.41: lim  ( Pn )  0
n

 x (n)
k x (n)
k 1    n  N ,  k para cualquier   0 , porque vale En el texto preparando el
teorema 2.41 hemos visto que siempre vale L( f , Pn )  S (TPn , f )  U ( f , Pn )  n . En el texto
b2
calculando la integral para ejemplo 4) hemos visto que vale lim L( f , Pn )  lim U ( f , Pn ) 
. Con
n  n  2
esto también se puede concluir, usando teorema 1.3, que la sucesión S (TPn , f ) es convergente y
b2
tiene el mismo límite , como salió arriba!
2

Página 99 de 114
Entonces hemos encontrado el enunciado siguiente:
Comentario 2.10.2:
Si existe el límite de las sumas inferiores lim L( f , Pn )  sup L( f , Pn ) y si también existe el límite
n  Pn

de las sumas superiores limU ( f , Pn )  inf U ( f , Pn ) y los dos límites son iguales, entonces sigue
n Pn

debido a L( f , Pn )  S (TPn , f )  U ( f , Pn )  n con teorema 1.3 que la sucesión de las sumas de


Riemann para la misma partición Pn tiene el mísmo límite. Pero el límite de las sumas de Riemann
para una sucesión de particiones Pn que satisface los hipótesis del teorema 2.41 ( lim  ( Pn )  0
n

 x (n)
k x(n)
k 1    n  N ,  k para cualquier   0 ) siempre es igual a la integral de f sobre
b
[ a, b] : lim S n (TPn , f )   f ( x)dx . Entonces si se puede concluir que en el caso de
n
a

lim L( f , Pn )  lim U ( f , Pn ) que este límite es igual a la integral de f sobre [ a, b] :


n  n 
b
lim L( f , Pn )  lim U ( f , Pn )   f ( x)dx !
n n 
a

Definición 2.30:
b
Si  f ( x)dx
a
(ver def. 2.29) existe con a  b , entonces

a b


b
f ( x)dx :   f ( x)dx .
a

Definición 2.31:
Si f (a ) existe, entonces
a

 f ( x)dx :0 .
a

0 3
9
Ejemplo:  x dx    x dx  
3 0
2
(El area bajo la gráfica f ( x )  x entre a=0 y b=3 es el area de

1 1
un triángulo: Atriang  gh  3  3 (con base g y altura h del triángulo).
2 2

Página 100 de 114


Teorema 2.42:
b
Si c  R es cualquier constante, entonces  c dx  c(b  a) .
a

Esto ya demostramos con el ejemplo 1 (ver arriba).

Teorema 2.43: Reglas para el integral definida:


(i) f : [ a, b]  R es integrable (ver def. 2.29) en el intervalo [a, b] , c  R es cualquier constante,
entonces c  f es integrable en [a, b] y
b b

 c f ( x) dx  c  f ( x) dx
a a

(ii) f : [ a, b]  R y g : [a, b]  R son integrables (ver def. 2.29) en el intervalo [a, b] , entonces
f  g es integrable en [a, b] y:
b b b

  f ( x)  g ( x) dx   f ( x) dx   g ( x) dx
a a a

Con la ayuda del teorema 2.41 y de las reglas para sumas infinitas (teorema 1.8) se puede
demostrar el teorema 2.43.

Teorema 2.44:
Sea a  c  b , f : [ a, b]  R acotada en el intervalo [ a, b] . Entonces vale: f es integrable en
[ a, b] si y sólo si f es integrable en [ a, c ] y [c, b] . Finalmente, si f es integrable en [ a, b] ,
entonces
b c b


a
f ( x) dx   f ( x) dx   f ( x) dx (i).
a c
Más general vale la ecuación (i) para una función integrable en un intervalo cerrado I sin
importar el orden de los tres números a,b y c  I .
Este teorema se puede demostrar con la ayuda del teorema 2.40.iii (teorema 2.41).

Más general vale la ecuación (i) del teorema 2.44 sin importar el orden a,b y c: Por ejemplo sea
c b c
a  b  c : Según teorema 2.44 vale 
a
f ( x) dx   f ( x) dx   f ( x) dx . De la def. 2.30
a b
c b c b b c b b

 f ( x) dx   f ( x) dx .   f ( x) dx   f ( x) dx   f ( x) dx
b c a a c
  f ( x) dx   f ( x) dx   f ( x) dx lo
a c a
cual es el resultado deseado (teorema 2.44 sin importar el orden a,b y c). Las demostraciones para
los otros órdenes posibles de estos tres números a,b,c son semejantes. Otra posibilidad consiste en
c a
que dos números son iguiales: Por ejemplo a  c  b . Entonces 
a
f ( x) dx   f ( x) dx  0 (ver def.
a

Página 101 de 114


b b c b b
2.31). También como a  c  f ( x) dx   f ( x) dx . Por tanto  f ( x) dx   f ( x) dx  0   f ( x) dx
c a a c a
lo cual es el resultado deseado. (q.e.d.)

Página 102 de 114


Teorema 2.45:
f : [ a, b]  R y g : [a, b]  R son integrables (ver def. 2.29) en el intervalo [a, b] con
f ( x )  g ( x ) x  [ a, b] , entonces
b b


a
f ( x) dx   g ( x) dx .
a

Eso teorema se pude demostrar con la ayuda del teorema 2.41 y de las reglas para sumas
infinitas.

Teorema 2.46:
f : [ a, b]  R continua en [a, b] , m : min f ( x) , M : max f ( x) (ver def. 2.18, 2.19 y
x[ a , b ] x[ a , b ]

teorema 2.19) de modo que m  f ( x)  M  x  [a, b] . Entonces


b
m(b  a )   f ( x) dx  M (b  a ) .
a

Demostración: Como f es continua en [a, b] , el teorema 2.19, el teorema del valor extremo,
b
garantiza la existencia de m y M. Por el teorema 2.42 vale  m dx  m(b  a)
a
(i) y

 M dx  M (b  a) (ii). Porque f es continua en [a, b] según teorema 2.40 es integrable en [a, b]


a
b b
. Entonces, como f ( x )  m x  [ a, b] , se tiene, por el teorema 2.45  f ( x) dx   m dx de donde,
a a
b
al sustituir de (i), se obtiene  f ( x) dx  m(b  a) .
a
De manera semejante se deduce con

b
M  f ( x ) x  [a, b] y (ii) que M (b  a )   f ( x) dx . Si se combina estas desigualdades se
a
b
obtiene m(b  a )   f ( x) dx  M (b  a ) . q.e.d.
a
Este teorema también vale para funciones que solamente deben ser integrables:
Teorema 2.46.i:
f : [a, b]  R sea integrable sobre [a, b] , y se suponga que m  f ( x)  M  x  [a, b] .
Entonces
b
m(b  a )   f ( x) dx  M (b  a ) .
a

La demostración para este hipótesis más general es la siguente:

Página 103 de 114


n
L( f , P )   mk ( xk  xk 1 )  m(b  a )  P porque los ínfimos mk siempre son la máxima cota
k 1
inferior en todos los intervalos k!
n
U ( f , P)   M k ( xk  xk 1 )  M (b  a )  P porque los supremos M k siempre son la mínima cota
k 1
superior en todos los intervalos k! Porque f es integrable por hipótesis, entonces
b b

 f ( x)dx  sup L( f , P)  inf U ( f , P) .


a P P
De  f ( x)dx  sup L( f , P)  L( f , P)  m(b  a)
a P
y

b b


a
f ( x)dx  inf U ( f , P )  U ( f , P )  M (b  a ) sigue m(b  a )   f ( x) dx  M (b  a ) . q.e.d.
P
a

Teorema 2.47: Teorema del valor medio para integrales


f : [ a, b]  R continua en [a, b] , entonces existe un número c  [ a, b] tal que
b

 f ( x) dx  f (c)  (b  a) .
a

Demostración: Segun el teorema 2.19, el teorema del valor extremo, la función f tiene un valor
máximo absoluto y un valor mínimo absoluto en [ a, b] porque f es continua en [ a, b] :
m : min f ( x)  f ( xm ) (ver def. 2.19) con a  xm  b , M : max f ( x)  f ( xM ) (ver def. 2.18)
x[ a , b ] x[ a , b ]

con a  xM  b . Por tanto, m  f ( x )  M x  [ a, b] . Por el teorema 2.46 vale


b b b
1 1
m(b  a )   f ( x) dx  M (b  a )  m   b  a a
f ( x) dx  M  f ( xm )  f ( x) dx  f ( xM )
a
ba a
con b  a .
Del teorema 2.10, teorema del valor intermedio, existe un c  [ xm , xM ] tal que
b b
1
b  a a
f (c )  f ( x) dx  f (c)  (b  a )   f ( x) dx con a  c  b . q.e.d.
a
(El valor c de este teorema no es necesariamente único. Además el teorema afirma la existencia
de un valor c, pero no proporciona un méthodo para obtener c. Pero como en la demostración del
teorema 2.10, se puede aplicar el principio de los intervalos encajados para encontrar el número
c como c  lim cn .)
n

Página 104 de 114


2.11. TEOREMAS FUNDAMENTALES DEL CÁLCULO

Teorema 2.48: Primer teorema fundamental del Cálculo


f : [ a, b]  R continua en [a, b] y sea x cualquier número x  [ a, b] . Si F es la función definida
x
por F ( x) :  f (t )dt (1) (ver definición 2.29, integral definida de f de a a x, y teorema 2.41)
a
entonces
x
d
dx a
F ' ( x)  f ( x ) (2)  f (t )dt  f ( x) x  [a, b] (3).

(Si x  a , la derivada en (2) y (3) puede ser una derivada por la derecha, y si x  b , puede ser
una derivada por la izquierda (ver def. 2.15).)
x1 x1  x

Demostración: x1 y x1  x  [a, b] : F ( x1 )   f (t )dt


a
y F ( x1  x)   f (t )dt .
a
x1  x x1

 F ( x1  x)  F ( x1 )   f (t )dt   f (t )dt .


a a
Por el teorema 2.44 (más general),

x1 x1  x x1  x x1  x x1 x1  x

 f (t )dt   f (t )dt   f (t )dt .


a x1 a
Entonces  f (t )dt   f (t )dt   f (t )dt .
a a x1
x1  x

 F ( x1  x)  F ( x1 )   f (t )dt . Por el teorema del valor medio para integrales (teorema 2.47)
x1
x1  x

existe algun número c  [ x1 , x1  x] tal que  f (t )dt  f (c)  x .


x1

F ( x1  x)  F ( x1 )
 F ( x1  x)  F ( x1 )  f (c)  x   f (c)
x
F ( x1  x)  F ( x1 )
 lim  lim f (c)  F ' ( x1 )  lim f (c) . c  [ x1 , x1  x]  x1  c  x1  x ,
x  0 x x  0 x 0

como lim x1  x1 y lim  x1  x   x1 se deduce (ver teorema 1.3, eso también vale para
x  0 x  0

funciones) lim c  x1 . Asi se tiene lim f (c)  lim f (c)  f ( x1 ) por que f es continua. Por tanto
x  0 x  0 c  x1

vale  F ' ( x1 )  lim f (c)  f ( x1 ) .


x  0

Si f no está definida para x  a pero es continua por la derecha en a (ver def. 2.12a), entonces si
F ( x1  x)  F ( x1 )
x1  a en lim  lim f (c) , x debe aproximarse a 0 por la derecha. Por tanto
x  0 x x  0

obtenemos F ' ( x1 )  f ( x1 ) (ver def. 2.15). De manera semejante, si f no está definida para x  b
F ( x1  x)  F ( x1 )
pero es continua por la izquierda en b, entonces si x1  b en lim  lim f (c) ,
x  0 x x  0

x debe aproximarse a 0 por la izquierda. Por tanto obtenemos F ' ( x1 )  f ( x1 ) q.e.d.

Página 105 de 114


Con la ecuación (2) y (3) del teorema 2.48 definimos, con la ayuda de la definición 2.29, integral
x
definida de f de a a x, ver teorema 2.41), una función F(x), F ( x) :  f (t )dt , que también
a
satisface la definición de la antiderivada (ver def. 2.28 y teorema 2.32), porque
F ' ( x)  f ( x)   f ( x)dx  F ( x)  C . Entonces esta función F definida con la integral definida es
la antiderivada de f con una constante arbitraria C. Por tanto se encuentra la misma función F(x)
(más una constante arbitraria) con las dos definiciones. Por eso utilizamos el mismo símbolo 
b
en los dos casos. La definición del integral definida  f ( x)dx
a
necesita dos límites a y b (ver el

segundo teorema fundamental del Cálculo), los cuales faltan en la definición de la antiderivada.
Por eso aperece una constante arbitraria en el caso de la antiderivada.  f ( x)dx se llama integral
indefinida.
En el segundo teorema fundamental del Cálculo encontramos, que para cada función
g ' ( x)  f ( x) x  [a, b] (  g es una antiderivada de f:  f ( x)dx  g ( x)  C (i)) vale
b

 f (t )dt  g (b)  g (a) (ii). Las dos ecuaciones (i) y (ii) son muy parecidas, pero los símbolos 
a
originalmente están definidos de maneras diferentes!
Por tanto podemos calcular un integral definida de una función f con la ayuda de su antiderivada
y no tenemos que calcular el area bajo la curva de la función entre a y b, R( f , a, b)  R 2 , segun la
def. 2.29 o el teorema 2.41!
El proceso de evaluación de una integral indefinida o una integral definida se denomina
integración.

d x 1 1
dx 1 t 3  1
Ejemplo 1: dt . f (t )  , segun teorema 2.48, ecuación (3) vale
t 1
3

d x 1 1

dx 1 t  1
3
dt  3
x 1
.

d x2
Ejemplo 2:
dx  3
cos(t )dt . Con u  x 2 en la regla de la cadena se obtiene
d dh du
h(u ( x))  (u )  ; h es una función arbitraria, por eso vale por el operador de
dx du dx
d du d d x2 du d u
dx 3 dx du 3
diferenciación:   . Entonces obtenemos cos(t )dt   cos(t )dt 
dx dx du
 2 x  cos(u )  2 x  cos( x 2 ) .

Página 106 de 114


Teorema 2.49: Segundo teorema fundamental del Cálculo
f : [ a, b]  R continua en [a, b] y sea g una función diferenciable en [a, b] tal que
g ' ( x)  f ( x) x  [a, b] (1). Entonces
b

 f (t )dt  g (b)  g (a) .


a

(Si x  a , la derivada en (1) puede ser una derivada por la derecha, y si x  b , la derivada en (1)
puede ser una derivada por la izquierda.)
Demostración: Por el teorema 2.48 (Primer teorema fundamental del Cálculo) se sabe que la
x
integral definida  f (t )dt , con límite superior variable en x, define una función F(x) cuya derivada
a

en [a, b] es f(x). Como por hipótesis g ' ( x )  f ( x) en [a, b] , se deduce, por el teorema 2.31, que
g ' ( x )  f ( x )  F ' ( x ) x  [ a, b]  g ( x )  F ( x )  K x  [ a, b]
x
 g ( x)   f (t )dt  K x  [a, b] donde K es alguna constante.
a
b a a
 x  b : g (b)   f (t )dt  K x  [a, b] y  x  a : g (a )   f (t )dt  K  K (  f (t )dt  0 segun
a a a
b
def. 2.29 y teorema 2.41; comparar def. 2.31). Entonces vale g (b)  g (a )   f (t )dt .
a

Si f no está definida para x  a pero es continua por la derecha en a (ver def. 2.12a), entonces la
derivada g ' ( x )  f ( x) es una derivada por la derecha , y se tiene g ' (a)  F ' (a) de donde se
a
concluye x  a : g (a )   f (t )dt  K  K .
a

Si f no está definida para x  b pero es continua por la izquierda en b (ver def. 2.12b), entonces
la derivada g ' ( x )  f ( x) es una derivada por la izquierda , y se tiene g ' (b)  F ' (b) de donde
b
se concluye x  b : g (b)   f (t )dt  K . q.e.d.
a

Ahora se puede obtener el valor exacto de una integral definida aplicando el segundo teorema
b

 f (t )dt  g (b)  g (a)  g ( x)


b
fundamental del Cálculo. Se denota a
.
a

2 x5

4
Ejemplo: x 4 dx . Como una antiderivada de x es , por el segundo teorema fundamental del
1 5
2
2 x5 32 1 31
 x dx     .
4
Cálculo se tiene
1 51 5 5 5

Página 107 de 114


Finalmente podemos escribir la regla de la cadena para antiderivación (ver teorema 2.38) para
integrales definidas:
b
f ( x)dx   f  g (t )   g ' (t ) dt . Podemos sustituir en la integral de lado derecho x  g (t ) ,
g (b )
g (a)
a

dg
entonces tenemos dx  dt  g ' dt . Entonces la frontera inferior (t  a ) es g (a ) y la frontera
dt
superior (t  b) es g (b) . Con esta transformación obtenemos la integral por el lado izquierdo.

1 dx
Ejemplo: 0
t 2  2  t dt . Si sustituimos x  t 2  2 tenemos dx  2t dt  dt 
2t
y las fronteras

0  2  2 y 12  2  3 .
2

Entonces vale
3
3
1 2  2  1
 
1 3 3
1 3 1 1 3 1 x 2

 t  2  t dt   x  dx    x 2 dx      2
 3 27  8 .
2
3 2
0 2 2 2 2 2 3 2  3  
2 2

Página 108 de 114


Tabla de integrales:

 1a) Si s es un número real, s  1 , entonces:


x s 1
d  x  n 1 n
n 1

 x dx  s  1  C , C  R . Demostración para s  n  N \  1 : dx  n  1   n  1  x


s

 x n segun teorema 2.16. q.e.d.


Para x s ( x  0 , x s : R   R  ) con s  R \  1  (definida con la ayuda de una sucesión de
números racionales que converge a s, ver “Función exponencial y logaritmo” (1) y [1b)])
se puede expresar x s con la ayuda de las funciones exponencial y logaritmo de esta manera
 
(ver “Función exponencial y logaritmo” (6)): x s  exp ln( x s ) porque x s  0 . Según las
reglas para las funciones del logaritmo vale x  exp s  ln( x)  ( x  0 ). Según las reglas
s

d
para la diferenciación y con exp( x)  exp( x) (ver “Función exponencial y logaritmo”
dx
d s d d
[2]) resulta que x  exp s  ln( x)   exp s  ln( x)    s  ln( x) 
dx dx dx
 
 exp ln( x s )  s 
1
x
 xs  s 
1
x
 s  x s 1 según las reglas para exponentes reales (ver
“Función exponencial y logaritmo” (1)). Q.e.d. (comparar también la demostración de la
integral 2))
1 d 1 x 1
 1b) Si s  1 , entonces  dx  ln x  C . Demostración: ln x  C   
x dx x x x
(ver “Función exponencial y logaritmo [2]).
 2)  a x dx 
ax
ln(a )
 C , a  R  , x  R , a x  R  . Demostración:
d x d
dx
a  exp ln a x
dx
  
  
x
d x d d a
a  exp x  ln a    exp ln a x  lna   a x  lna  .   a x . Q.e.d.
dx dx dx ln(a )
ex
Caso especial: a  e  0 (número de Euler):  e x dx   C  ex  C , x  R .
ln(e)
d
 3a)  sen ( x) dx   cos( x)  C . Demostración: ( cos( x))  sen ( x) .
dx
d
 3b)  cos( x) dx  sen ( x)  C . Demostración: sen ( x)  cos( x) .
dx
d d e x  e x e x  e x
 4a)  senh ( x) dx  cosh( x)  C . Demostración: cosh( x)  
dx dx 2 2
 senh( x) .
d d e x  e x e x  e x
 4b)  cosh( x) dx  senh( x)  C . Demostración:
dx
senh ( x) 
dx 2

2
 cos h ( x) .

Página 109 de 114


1 d d cos( x)
 5a)  sen 2
( x)
dx   cot( x)  C . Demostración:
dx
cot( x) 
dx sen ( x)
 sen ( x)  cos 2 ( x)
2
1 d 1
  2
.   cot( x)   .
2
sen ( x) sen ( x) dx sen 2 ( x)
1 d d sen ( x) cos 2 ( x)  sen 2 ( x)
 5b)  cos 2 ( x) d x  tan( x )  C . Demostración:
dx
tan( x ) 
dx cos( x)

cos 2 ( x)
1
 .
cos 2 ( x)
tan( x) 1 d 1  sen ( x) 1 sen ( x)
 5c)  dx   C . Demostración  2
 
cos( x) cos( x) dx cos( x) cos ( x) cos( x) cos( x)
1
  tan( x) .
cos( x)
cot( x) 1 d 1  cos( x)
 5d)  sen ( x) dx   sen ( x)  C . Demostración   
dx  sen( x)  sen 2 ( x)
1 cos( x) 1
    cot( x) . (Ver
sen ( x) sen ( x) sen ( x)
también teorema 2.37)
1
 6)  dx  arctan( x)  C . Demostración (ver teorema 2.30 y “Cálculo Diferencial
1  x2
2016, wichtige Detailergänzungen zum Manuskript”, hoja [2f)], [2g)] ):
d 1 1
arctan(x)    cos 2  arctan( x)  . De
dx d 1
tan( y ) y  arctan( x ) y  arctan( x )
dy cos 2 ( y )
1 2z  1
sen 2 ( y )  cos 2 ( y )  1  y  2
para todo y   , z  Z ( cos2 ( y )  0)
cos ( y ) 2
1 2z  1
 tan 2 ( y )  1  2
y   ,zZ , se sigue
cos ( y ) 2
1
cos 2  arctan( x)   . Porque arctan( R)  (  2 ,  2 ) el denominador
tan  arctan( x)   1
2

de esta derivada está definida para todo x  R . Entonces resulta que


d 1
arctan( x)  2  x  R . Q.e.d.
dx x 1
1
 7)  dx  arcsen ( x)  C1  arccos( x)  C2  x (1 ,1) .
1  x2
Demostración de la primera antiderivada (ver teorema 2.30 y ejemplo 2 para este teorema
y “Cálculo Diferencial 2016, wichtige Detailergänzungen zum Manuskript”, hoja [2d)],
d 1 1 1
[2e)] ): arcsen( x)    . De
dx d cos( y ) y  arcsen ( x ) cos arcsen ( x) 
sen( y ) y  arcsen( x )
dy

Página 110 de 114


sen 2 ( y )  cos2 ( y )  1  y se sigue cos2 ( y )  1  sen 2 ( y )  y
 cos( y )  1  sen 2 ( y )  y . Entonces resulta que
cos arcsen( x)   1  sen 2  arcsen( x)  . arcsen  [1,1]   [  2 ,  2 ] , pero para
1 1
x  1 resulta que arcsen ( x  1)    2 tal que  no
cos arcsen ( x  1)  cos(  2)
está definida. También para x  1 resulta que arcsen ( x  1)   2 tal que
1 1 d
 no está definida. Por eso arcsen( x) solamente está
cos arcsen ( x  1)  cos( 2) dx
definida en el intervalo x  (1,1) . Porque arcsen  (1,1)   (  2 ,  2 ) resulta que
cos arcsen ( x)   0  x  (1,1) y por eso resulta que
cos arcsen ( x)   cos arcsen ( x)   x (1 ,1) y por lo tanto se tiene
cos arcsen ( x)   1  sen 2  arcsen ( x)   1  x 2  x (1 ,1) . Entonces resulta que
d 1 1
arcsen ( x)    x (1 ,1) . Q.e.d.
dx cos arcsen ( x)  1  x2

De manera senejante se puede demostrar la segunda antiderivada


1
 1  x2
dx  arccos( x)  C2  x (1 ,1) (ver teorema 2.30 y “Cálculo Diferencial

2016, wichtige Detailergänzungen zum Manuskript”, hoja [2d)2.)], [2d)3.)] )):


d 1 1 1
arccos( x)    . De
dx d  sen ( y ) y  arccos( x )  sen  arccos( x) 
cos( y ) y  arccos( x )
dy
sen 2 ( y )  cos2 ( y )  1  y se sigue sen 2 ( y )  1  cos2 ( y )  y
 sen( y )  1  cos2 ( y )  y . Entonces resulta que
sen arccos( x)   1  cos2  arccos( x)  . arccos [1,1]   [ 0 ,  ] , pero para x  1
1 1
resulta que arccos( x  1)   tal que  no está definida.
 sen  arccos( x  1)   sen( )
1 1
También para x  1 resulta que arccos( x  1)  0 tal que 
 sen  arccos( x  1)   sen(0)
d
no está definida. Por eso arccos( x) solamente está definida en el intervalo x  (1,1) .
dx
Porque arccos (1,1)   ( 0 ,  ) resulta que sen  arccos( x)   0  x  (1,1) y por eso
resulta que sen  arccos( x)   sen  arccos( x)   x (1 ,1) y por lo tanto se tiene

Página 111 de 114


sen  arccos( x)   1  cos 2  arccos( x)   1  x 2  x (1 ,1) . Entonces resulta que
d 1 1
arccos( x)    x (1 ,1) . Q.e.d.
dx  sen  arccos( x)  1  x2

3. FUNCIONES ESPECIALES

3.1. DEFINICIÓN DE UNA FUNCIÓN POTENCIA CON EXPONENTES RACIONALES


Repetimos algunos teoremas de la relación de orden entre números reales “<” (o “>”
respectivamente) (ver “Ordnungsrelationen [1]ff, H. Heuser, Lehrbuch der Analysis Bd.1, S. 77):

Teorema 3.1: Algunas reglas para la relación del orden “<” entre números reales:

Según el axioma básico de monotonía vale para cualquier par de números reales a y b:
1) a  b  a  c  b  c  c  R

2) a  b  a  c  b  c  c  R ( c  0)
Se puede deducir de los tres axiomas del orden entre otros:

3) a  b  a  c  b  c  c  R ( c  0)
4) 0  a  b  0  c  d  a  c  b  d

5) para cualquier x, y  0 ( x, y  R ) : x  y  x  y  p  N
p p

La propiedad 5) significa que la función f ( x)  x con cualquier potencia p  N es


p

estrictamente creciente en R  .
Si se permite que x  0 (con 0 p  0 ), vale para todo y  x  0 : 0  y  0  y  p  N
p

(porque y  0  y  0 , ver abajo). Entonces f ( x)  x es estrictamente creciente en


p p

R0  [ 0 ,  ) (ver la def. 2.20).

Con respecto a la regla 5) ver también el ejemplo abajo de la def. 2.21.

Teorema 3.2: La definición de una función única inversa de f ( x)  x , p  N :


p

Si y  0 ( a  R0 ) y p  N , entonces la ecuación x  y tiene exactamente una solución única


p

~ 1p
x  0 . Esta solución se denota como y o p y y se llama la p-esima raíz de y.
Casos especiales: 2 y : y . 1 y  y1 1  y  x1  y .

Según el teorema 3.2 vale entonces: y  x  x  y para x, y  0 . Según la def. 2.27 la


p 1p

1
función f ( x)  x es la función inversa de la función f ( x)  x , p  N .
1p p

Página 112 de 114


La demostración del teorema 3.2 es muy larga y un poco complicada, pero vamos a mencionar
algunas de las ideas más importantes de esta demostración.
Si x  y  0 la única solución es x  0 , solamente 0 p  0 para cualquier p  N : x  0 y x  0
p

, es decir x  0 , entonces multiplicando por x resulta que x 2  0 , multiplicando p veces por x


resulta que x p  0 . Siempre vale x  0  x p  0 , y acabamos de demostrar que especialmente
vale x  0  x p  0 . De la negación de la consecuencia se sigue la negación de la hipótesis, y
porque tenemos x  0 y x p  0 esto es equivalente a: x p  0  x  0 . Y porque 0 p  0 se sigue
además x p  0  x  0 (equivalentemente x p  0  x  0 ). Entonces ya hemos demostrado que
la solución única del problema x  y  0 es x  0 .
p

Entonces podemos suponer x  y  0 : Para el caso especial p  1 se tiene x  y  0 como


p

solución, que es única. Entonces ya hemos demostrado que existe una solución única para el caso
p  1 . Para la busqueda de una solución general única se puede suponer que p  1 .
La unicidad de la solución del problema x  y  0 resulta directamente de la regla 5) del
p

teorema 3.1: Si existen dos soluciones x1  y  0 y x2  y  0 con x1  x2 , entonces tenemos


p p

dos posibilidades: 1) x1  x2 y 2) x2  x1 . Del caso 1) resulta según regla 5) del teorema 3.1:
x1p  x2p , lo que es una contradicción a la hipótesis x1p  y y x2p  y , es decir x1p  x2p . De manera
semejante el caso 2) tiene como consecuencia x2  x1 , es decir x1  x2 . Entonces, si x1  x2 ,
p p p p p p

entonces debe valer x1  x2 , es decir la solución es única!


La unicidad de la solución del problema x  y  0 es equivalente con la existencia de una función
p

1
inversa de la función f ( x)  x , p  N : y  f ( x)  x  x  f ( y)  y , p  N . La función
p p 1p

inversa existe en este caso según el teorema 2.29a. f ( x)  x , p  N es contiuna para todo x (ver
p

el teorema 2.9a), también en R0  [ 0 ,  ) . Entonces según el teorema 2.29a resulta que f tiene
1
una función inversa f ( x) definida en R0  [ 0 ,  ) que es estrictmante creciente y continua en
R0  [ 0 ,  ) . Además existe la derivada de esta función inversa según el teorema 2.30:
f ( x)  x p , p  N es continua y monótona en R0  [ 0 ,  ) . Además f es diferenciable para todo x

(ver la demostración de
d n
dx
 
'
x  x n  n  x n 1 , x  R , n  N arriba del teorema 2.12). f es

diferenciable y desigual a cero  x  R  ( 0 ,  ) . Entonces según el teorema 2.30 existe la
1 d 1 1
derivada de la función inversa f ( x)  x y vale:
1p
f ( x)  . Entonces se tiene:
dx 
df 1
dx
f ( x)

 
x1 p ' 
1
p( x1 p ) p 1
.

Ahora no se puede hacer nada más, porque se necesita la definición de una potencia racional
p 1

como x 1 p
y las leyes para este tipo de potencias, por ejemplo que vale x  
1 p p 1
x p
 x11 p .
1
Por eso en todas maneras hay que definir bien la función f ( x)  x , lo que queremos hacer
1p

en el texto siguiente. Supongamos que valen las leyes de las potencias (esto sí se puede demostrar),

Página 113 de 114


1

 
entonces resulta x ' 
1 p 1

p x11 p p
1  (11 p ) 1 p 1
x  x . Esto es exactamente la regla análoga a la
p
regla para exponentes naturales.

Falta todavía teminar este capítulo, lo siento!

Página 114 de 114

También podría gustarte